SlideShare a Scribd company logo
1 of 76
ĐẠI HỌC QUỐC GIA HÀ NỘI
TRƯỜNG ĐẠI HỌC KHOA HỌC TỰ NHIÊN HÀ NỘI
VŨ THỊ HẢI THANH
PHƯƠNG PHÁP GIẢI BÀI TOÁN CỰC TRỊ
VÀ ỨNG DỤNG
LUẬN VĂN THẠC SĨ KHOA HỌC
Chuyên ngành: Phương pháp toán sơ cấp
Mã số: 60 46 40
NGƯỜI HƯỚNG DẪN KHOA HỌC
PGS.TS Nguyễn Đình Sang
Hà Nội - 2012
Mục lục
Mục lục . . . . . . . . . . . . . . . . . . . . . . . . . . . . . i
Lời nói đầu . . . . . . . . . . . . . . . . . . . . . . . . . . . iii
Lời cảm ơn . . . . . . . . . . . . . . . . . . . . . . . . . . . . iv
Bảng kí hiệu . . . . . . . . . . . . . . . . . . . . . . . . . . v
1 Cực trị hàm số 1
1.1 Kiến thức chuẩn bị . . . . . . . . . . . . . . . . . . . . . 1
1.2 Các phương pháp tìm cực trị . . . . . . . . . . . . . . . . 2
1.2.1 Áp dụng điều kiện cần, điều kiện đủ . . . . . . . 2
1.2.2 Phương pháp sử dụng bất đẳng thức . . . . . . . 9
1.3 Một số bài toán tổng quát và ứng dụng . . . . . . . . . . 11
1.3.1 Bài toán tổng quát . . . . . . . . . . . . . . . . . 11
1.3.2 Bài tập tham khảo . . . . . . . . . . . . . . . . . 14
2 Giá trị lớn nhất, giá trị nhỏ nhất 19
2.1 Các khái niệm cơ bản . . . . . . . . . . . . . . . . . . . . 19
2.1.1 Giá trị lớn nhất, giá trị nhỏ nhất của hàm số . . . 19
2.1.2 Giá trị lớn nhất, giá trị nhỏ nhất của một tập hợp 20
2.1.3 Một số tính chất của giá trị lớn nhất, giá trị nhỏ
nhất . . . . . . . . . . . . . . . . . . . . . . . . . 22
2.1.4 Một số định lý về giá trị lớn nhất, giá trị nhỏ nhất 24
2.2 Các phương pháp tìm giá trị lớn nhất, giá trị nhỏ nhất . 26
2.2.1 Phương pháp sử dụng đạo hàm . . . . . . . . . . 26
2.2.2 Phương pháp tập giá trị . . . . . . . . . . . . . . 31
2.2.3 Phương pháp lượng giác . . . . . . . . . . . . . . 37
2.2.4 Phương pháp hình học . . . . . . . . . . . . . . . 44
i
MỤC LỤC
2.2.5 Phương pháp sử dụng bất đẳng thức . . . . . . . 51
2.2.6 Một số bài tập vận dụng . . . . . . . . . . . . . . 63
Kết luận . . . . . . . . . . . . . . . . . . . . . . . . . . . . . 69
Tài liệu tham khảo . . . . . . . . . . . . . . . . . . . . . . . 70
ii
Lời nói đầu
Bài toán cực trị địa phương và cực trị tuyệt đối là những bài toán
rất quan trọng trong giải tích toán học và có nhiều ứng dụng khác nhau
trong toán học cũng như trong nhiều ngành khoa học khác như: Kinh
tế, Khoa học công nghệ, ...v.v.
Để giải bài toán cực trị, có nhiều phương pháp khác nhau. Mục đích
của luận văn là giới thiệu các phương pháp giải dạng toán này, cho bình
luận về các phương pháp đó đồng thời đưa ra một số ứng dụng. Những
ứng dụng của bài toán cực trị có rất nhiều, nhưng vì giới hạn trong
phương pháp toán sơ cấp và hạn chế trong một luận văn thạc sĩ nên bản
luận văn chỉ nêu ra một số ứng dụng cơ bản.
Bản luận văn gồm 2 chương:
Chương 1: Cực trị hàm số.
Trình bày bài toán cực trị địa phương, đưa ra điều kiện cần, điều kiện
đủ để có cực trị. Cho những ví dụ không thỏa mãn điều kiện đủ nhưng
vẫn có cực trị. Trình bày các phương pháp khác nhau để giải bài toán
cực trị, tổng quát hóa một số bài toán về cực trị với mong muốn đưa ra
cách giải nhanh gọn cho các bài toán dạng này.
Chương 2: Giá trị lớn nhất, giá trị nhỏ nhất.
Phần đầu của chương trình bày định nghĩa giá trị lớn nhất, giá trị nhỏ
nhất của hàm số trên một tập, điều kiện đủ để tồn tại giá trị lớn nhất,
giá trị nhỏ nhất của hàm một biến và các tính chất của giá trị lớn nhất,
giá trị nhỏ nhất. Trong phạm vi chương trình phổ thông, hàm số nhiều
biến không được nghiên cứu. Vì vậy để tìm giá trị lớn nhất, giá nhỏ nhất
của hàm nhiều biến, ta phải quy về bài toán tìm giá trị lớn nhất, giá trị
nhỏ nhất của một tập hợp số. Phần tiếp theo luận văn trình bày một
số phương pháp khác nhau để giải bài toán tìm giá trị lớn nhất, giá trị
nhỏ nhất trong đó dành nhiều thời gian cho phương pháp bất đẳng thức.
Phần cuối chương là một số bài toán vận dụng phối hợp nhiều phương
pháp.
Lời cảm ơn
Hoàn thành được luận văn này, ngoài sự nỗ lực của bản thân, tôi đã
nhận được sự chỉ bảo, giúp đỡ từ nhiều phía của các thầy, cô giáo, gia
đình và bạn bè.
Tôi xin bày tỏ lòng biết ơn sâu sắc tới người thầy kính mến PGS.TS.
Nguyễn Đình Sang, người đã trực tiếp truyền thụ kiến thức, quyết định
hướng nghiên cứu và tận tình hướng dẫn cho tôi hoàn thành bản luận
văn.
Tôi xin chân thành cảm ơn các thầy, cô giáo khoa Toán - Cơ - Tin học,
Trường Đại học Khoa học tự nhiên - Đại học Quốc gia Hà Nội, những
người đã trực tiếp giảng dạy và giúp đỡ tôi trong quá trình học tập tại
trường cùng toàn thể bạn bè và người thân đã đóng góp ý kiến, giúp đỡ,
động viên tôi trong quá trình học tập, nghiên cứu và hoàn thành luận
văn này.
Ứng dụng của bài toán cực trị có rất nhiều, nhưng vì giới hạn trong
phương pháp toán sơ cấp và hạn chế trong một luận văn thạc sĩ nên bản
luận văn mới chỉ trình bày được một phần nào đó. Do thời gian có hạn
và năng lực có phần hạn chế nên chắc chắn luận văn không tránh khỏi
những thiếu sót. Kính mong nhận được ý kiến đóng góp của các thầy cô
và bạn bè đồng nghiệp để bản luận văn được hoàn chỉnh hơn.
Xin chân thành cảm ơn.
Hà Nội, ngày 20 tháng 11 năm 2012
Học viên
Vũ Thị Hải Thanh
Bảng kí hiệu
N tập các số tự nhiên
N∗
tập các số tự nhiên khác không
Z tập các số nguyên
Z+ tập số nguyên không âm
Z∗
+ tập số nguyên dương
R tập số thực
R∗
tập số thực khác không
R+ tập số thực không âm
R∗
+ tập số thực dương
C tập số phức
[a; b] = {x ∈ R|a ≤ x ≤ b}
(a; b) = {x ∈ R|a < x < b}
(a; b] = {x ∈ R|a < x ≤ b}
v
Chương 1
Cực trị hàm số
Trong chương này, chúng ta sẽ trình bày khái niệm về cực trị hàm số.
Điều kiện để có cực trị hàm số, đưa ra một số ví dụ minh họa điều kiện
cần, điều kiện đủ cũng như giới thiệu các phương pháp tìm cực trị kèm
theo các ví dụ và bài tập.
1.1 Kiến thức chuẩn bị
Định nghĩa 1.1. Cho khoảng (a; b) ⊂ R và hàm số f : (a; b) → R.
Ta nói rằng, hàm f đạt cực đại địa phương(tương ứng cực tiểu địa
phương) tại x0 ∈ (a; b) nếu: ∃δ sao cho (x0 − δ; x0 + δ) ⊂ (a; b) và
f(x0) ≥ f(x) (tương ứng f(x0) ≤ f(x)), với mọi x ∈ (x0 − δ; x0 + δ) và
f không phải là một hằng số trong một lân cận nào đó của x0.
Điểm x0 mà tại đó hàm đạt cực đại địa phương hoặc cực tiểu địa
phương được gọi chung là điểm cực trị của hàm số.
Định lý 1.1. (Định lý Fermat - Điều kiện cần để hàm số có cực trị)
Cho khoảng (a; b) ⊂ R và hàm số f : (a; b) → R.
Nếu điểm c ∈ (a; b) là điểm cực trị của hàm số f và nếu tồn tại f (c)
thì f (c) = 0.
Điểm x0 mà tại đó f (x0) = 0 hoặc đạo hàm không xác định được gọi
là điểm dừng của hàm f.
Nhận xét: Nếu hàm f : (a; b) → R là hàm khả vi trên (a; b) thì những
điểm cực trị của f phải nằm trong số các điểm dừng của f.
Định lý 1.2. (Điều kiện đủ để hàm số đạt cực trị)
Giả sử hàm số f liên tục trên (a; b) chứa điểm x0 và có đạo hàm trên
1
Chương 1. Cực trị hàm số
các khoảng (a; x0) và (x0; b).
- Nếu f (x) đổi dấu từ âm sang dương khi x qua điểm x0 thì hàm số
đạt cực tiểu tại điểm x0.
- Nếu f (x) đổi dấu từ dương sang âm khi x qua điểm x0 thì hàm số
đạt cực đại tại điểm x0.
Định lý 1.3. Giả sử hàm số f(x) xác định trên khoảng (a; b), x0 là một
điểm dừng của f(x). Hàm f(x) khả vi cấp 1 và cấp 2 tại x0. Khi đó:
- Nếu f (x0) < 0 thì hàm số f đạt cực đại tại x0.
- Nếu f (x0) > 0 thì hàm số f đạt cực tiểu tại x0.
1.2 Các phương pháp tìm cực trị
1.2.1 Áp dụng điều kiện cần, điều kiện đủ
Dựa vào điều kiện cần, điều kiện đủ để hàm số đạt cực trị, ta xây
dựng các quy tắc tìm cực trị của hàm số f(x) liên tục trên khoảng (a; b)
sau đây:
Quy tắc 1.
- Tìm f (x) ;
- Tìm các điểm xi, (i = 1, 2, 3, ...) mà tại đó f có đạo hàm bằng 0
hoặc hàm số liên tục nhưng không có đạo hàm;
- Xét dấu f (x).Nếu f (x) đổi dấu khi x qua điểm xi thì hàm số đạt
cực trị tại xi.
Ví dụ 1.1. Tìm cực trị của hàm số:
y = 3
x(1 − x)2
Lời giải.
Hàm y xác định và liên tục trên R.
Với mọi x = 0 và x = 1
y =
1 − 3x
3 3
x2(1 − x)
2
Chương 1. Cực trị hàm số
y = 0 ⇔ x =
1
3
Lập bảng biến thiên của hàm y:
x
y
y
−∞ 0 1
3 1 +∞
+ + 0 − +
−∞−∞
3
√
4
3
3
√
4
3
00
+∞+∞
0
Từ bảng biến thiên ta thấy:
Hàm số đạt cực đại tại x = 1
3, giá trị cực đại của hàm số là y(1
3) =
3
√
4
3 .
Hàm số đạt cực tiểu tại x = 1, giá trị cực tiểu của hàm số là y(1) = 0.
Chú ý: Khi qua điểm x = 0 đạo hàm y không đổi dấu nên hàm số đã
cho không có cực trị tại điểm x = 0.
Ví dụ 1.2. Tìm cực trị của hàm số:
y = −x2
+ 2x + 3
Lời giải.
Hàm y xác định trên R.
Ta có
y = −x2
+ 2x + 3 = (|−x2 + 2x + 3|)2
⇔ y =
(−x2
+ 2x + 3)(−2x + 2)
|−x2 + 2x + 3|
=
f(x)
|−x2 + 2x + 3|
Xét f(x) = (−x2
+ 2x + 3)(−2x + 2) = 0 ⇔
x = ±1
x = 3
Lập bảng biến thiên của hàm y
x
y
y
−∞ −1 1 3 +∞
− + 0 − +
+∞+∞
00
44
00
+∞+∞
3
Chương 1. Cực trị hàm số
Từ bảng biến thiên ta suy ra
Giá trị cực đại của hàm số y(1) = 4
Giá trị cực tiểu của hàm số y(−1) = 0; y(3) = 0
Ví dụ 1.3. Tìm cực trị của hai hàm số sau:
f(x) =
xe−1
x với x = 0
0 với x = 0
g(x) =
e− 1
x2
với x = 0
0 với x = 0
Lời giải.
Ta có:
f (x) = e− 1
x2
+
2
x2
e− 1
x2
, ∀x = 0
Nhận thấy f (x) > 0, ∀x = 0
Mặt khác, do
lim
x→0−
xe−1
x = −∞
Nên hàm f(x) không liên tục tại x = 0.
Từ đó suy ra hàm f(x) không có cực trị.
Hàm g(x) liên tục với mọi x,vì lim
x→0
e− 1
x2
= 0.
Ta thấy với mọi x = 0
g (x) =
2
x3
e− 1
x2
Lập bảng biến thiên của hàm g(x):
x
g (x)
g(x)
−∞ 0 +∞
− +
+∞+∞
00
+∞+∞
4
Chương 1. Cực trị hàm số
Từ bảng biến thiên suy ra, giá trị cực tiểu của hàm số là g(0) = 0.
Nhận xét: Hai hàm f(x) và g(x) đều có đạo hàm không xác định tại
điểm x = 0. Nhưng khi qua điểm x = 0: hàm g(x) có đạo hàm đổi dấu
nên g(x) mới có cực trị, còn hàm f(x) thì đạo hàm không đổi dấu nên
không tồn tại cực trị.
Ví dụ 1.4. Tìm cực trị của hàm số:
y = (1 + x +
x2
2!
+ ... +
xn
n!
)e−x
, n ∈ N∗
.
Lời giải.
Hàm số đã cho xác định trên R.
Ta có
y = −
xn
n!
e−x
, n ∈ N∗
• Với n = 2k, k ∈ N
Khi đó
y = −
xn
n!
e−x
< 0, ∀x ∈ R
Do đó hàm số không có cực trị.
• Với n = 2k + 1, k ∈ N
Ta có y = 0 ⇔ x = 0
Lập bảng biến thiên của hàm y :
x
y
y
−∞ 0 +∞
+ 0 −
−∞−∞
00
−∞−∞
Vậy giá trị cực đại của hàm y là y(0) = 0.
Ví dụ 1.5. Tìm cực trị của hàm số sau:
f(x) =
2 − x2
(2 + sin 1
x) với x = 0
2 với x = 0
5
Chương 1. Cực trị hàm số
Lời giải.
Hàm số đã cho xác định và liên tục trên R vì lim
x→0
f(x) = 2.
Nhận thấy: f(x) − f(0) = −x2
(2 + sin 1
x) < 0, ∀x = 0
Mặt khác
f (x) = −2x(2 + sin
1
x
) + cos
1
x
Với xk = 1
kπ , k ∈ Z, ta có:
cos
1
xk
=
1 với k chẵn
−1 với k lẻ
Từ đó suy ra:
f (x) > 0 với k chẵn
f (x) < 0 với k lẻ.
Như vậy f (x) đổi dấu trong khoảng (0; 1
kπ )
Tuy vậy ta vẫn kết luận được giá trị cực đại của hàm f(x) là f(0) = 2.
Nhận xét: Như vậy không phải hàm số nào cũng có đạo hàm không
đổi dấu về một lân cận ở phía phải (hay phía trái) của điểm cực trị. Hàm
số trong ví dụ (1.5) không thỏa mãn điều kiện đủ nhưng vẫn có cực trị.
Các hàm sơ cấp thường không có tình trạng này.
Quy tắc 2.
- Tìm f (x) ;
- Tìm các nghiệm xi, (i = 1, 2, 3, ...) của phương trình f (x) = 0;
- Tìm f (x) và tính f (xi) :
Nếu f (xi) > 0 thì hàm số đạt cực tiểu tại điểm xi.
Nếu f (xi) < 0 thì hàm số đạt cực đại tại điểm xi.
Ví dụ 1.6. Tìm cực trị của hàm số:
y = x − sin2x + 2
Lời giải.
Hàm số đã cho xác định trên R.
Ta có y = 1 − 2 cos 2x;
y = 0 ⇔ x = ±
π
6
+ kπ (k ∈ Z)
6
Chương 1. Cực trị hàm số
• Vì y = (π
6 + kπ) = 4 sin(π
3 + k2π) > 0
Nên hàm số đạt cưc tiểu tại x = π
6 + kπ và giá trị cực tiểu của hàm
số là y(π
6 + kπ) = π
6 −
√
3
2 + 2 + kπ, k ∈ Z.
• Vì y (−π
6 + kπ) = 4sin(π
3 + k2π) < 0
Nên hàm số đạt cực đại tại x = −π
6 + kπ và giá trị cực đại của hàm
số là y(−π
6 + kπ) = −π
6 +
√
3
2 + 2 + kπ, k ∈ Z.
Ví dụ 1.7. Tìm cực trị của hàm số:
y =
ex
+ e−x
2
Lời giải.
Hàm số đã cho xác định trên R.
Ta có
y =
ex
− e−x
2
y = 0 ⇔ ex
= e−x
⇔ x = 0
Lại có
y =
ex
+ e−x
2
⇒ y (0) = 1 > 0
Như vậy y (0) = 0; y (0) > 0
Vậy giá trị cực tiểu của hàm số là y(0) = 1
Ví dụ 1.8. Tìm cực trị của hàm số:
y = x2 + x + 1 + x2 − x + 1
Lời giải.
Hàm số đã cho xác định trên R.
Ta có
y =
2x + 1
2
√
x2 + x + 1
+
2x − 1
2
√
x2 − x + 1
y = 0 ⇔ (2x + 1) x2 − x + 1 − (1 − 2x) x2 + x + 1 = 0
⇔ 2x[( x2 − x + 1 + x2 + x + 1) − 1] = 0
⇔
x = 0√
x2 − x + 1 +
√
x2 + x + 1 = 1
7
Chương 1. Cực trị hàm số
Do
x2 − x + 1+ x2 + x + 1 = (x +
1
2
)2 +
3
4
+ (x −
1
2
)2 +
3
4
≥ 2
√
3
2
> 1, ∀x
Nên y = 0 ⇔ x = 0
Mặt khác
y =
4
√
x2 + x + 1 − (2x+1)2
√
x2+x+1
4(x2 + x + 1)
+
4
√
x2 − x + 1 − (2x−1)2
√
x2−x+1
4(x2 − x + 1)
Suy ra y (0) = 3
2 > 0
Vậy hàm số đạt cực tiểu tại x = 0 và giá trị cực tiểu của hàm số là
y(0) = 2.
Ví dụ 1.9. Cho hàm số y = (x − m)3
− 3x.
Tìm m để hàm số đạt cực đại tại điểm có hoành độ x = 0.
Lời giải.
Hàm số đã cho xác định trên R.
Ta có y = 3(x − m)2
− 3, y = 6(x − m).
Hàm số đạt cực đại tại điểm có hoành độ x = 0 khi:
y (0) = 0
y (0) < 0
⇔
3m2
− 3 = 0
−6m < 0
⇔



m = −1
m = 1
m > 0
⇔ m = 1
Vậy m = 1 hàm số đã cho đạt cực đại tại điểm x = 0.
Ví dụ 1.10. Cho hàm số y = −2x + k
√
x2 + 1.
Tìm k để hàm số có cực tiểu.
Lời giải.
Hàm số đã cho xác định và liên tục với mọi x ∈ R.
Ta có
y = −2 +
kx
√
x2 + 1
=
kx − 2
√
x2 + 1
√
x2 + 1
8
Chương 1. Cực trị hàm số
y =
k
(x2 + 1)
√
x2 + 1
Do đó y và y xác định và liên tục với mọi x ∈ R
Xét các trường hợp:
• Nếu k = 0 thì y = −2x, nên hàm số không có cực tri.
• Nếu k < 0 thì y < 0, ∀x ∈ R, nên hàm số hoặc không có cực trị, hoặc
chỉ có cực đại.
• Nếu k > 0 thì y > 0, ∀x ∈ R, nên hàm số đã cho có cực tiểu khi và
chỉ khi phương trình y = 0 có nghiệm.
y = 0 ⇔ 2 x2 + 1 = kx
⇔
kx ≥ 0
4(x2
+ 1) = k2
x2
⇔
x ≥ 0 (1)
(k2
− 4)x2
= 4 (2)
Phương trình y = 0 có nghiệm khi và chỉ khi (2) có nghiệm.
Suy ra k2
− 4 > 0 hay k > 2.
Vậy k > 2 hàm số đã cho có cực tiểu.
Nhận xét: Quy tắc 2 tìm cực trị hàm số được áp dụng trong trường
hợp hàm số chứa tham số hoặc khó xét dấu đạo hàm bậc nhất.
1.2.2 Phương pháp sử dụng bất đẳng thức
Khi tìm cực trị của các hàm số không tính được đạo hàm hoặc tính
được đạo hàm nhưng việc tìm nghiệm của phương trình y = 0 gặp nhiều
khó khăn, ta có thể sử dụng phương pháp bất đẳng thức để tìm cực trị
của hàm số. Nội dung của phương pháp như sau:
Cho hàm số y = f(x) xác định trên D.
+ Nếu ta tìm được giá trị x0 mà f(x) ≥ f(x0), ∀x0 ∈ (x0 −δ, x0 +δ)
với (x0 − δ, x0 + δ) ⊂ D, ∀δ > 0 và f không phải là hằng số trong một
lân cận của x0 thì hàm số đạt cực tiểu tại x0.
+ Nếu ta tìm được giá trị x1 mà f(x) ≤ f(x1), ∀x1 ∈ (x1 −δ, x1 +δ)
với (x1 − δ, x1 + δ) ⊂ D, ∀δ > 0 và f không phải là hằng số trong một
lân cận của x1 thì hàm số đạt cực đại tại x1.
9
Chương 1. Cực trị hàm số
Sau đây ta sẽ xét một số ví dụ vận dụng phương pháp bất đẳng thức
để tìm cực trị hàm số.
Ví dụ 1.11. Tìm cực trị của hàm số:
y =
√
sin x − 4
√
cos x
Trong trường hợp này, nếu sử dụng phương pháp xét dấu đạo hàm
thì sẽ phức tạp hơn rất nhiều. Mặc dù đây là hàm lượng giác nhưng ta
cũng không vận dụng được quy tắc 2 cho ví dụ này, do đó ta cần vận
dụng một phương pháp khác.
Lời giải.
Với mọi x thuộc tập xác định của hàm số, ta luôn có:
0 ≤
√
sin x ≤ 1
−1 ≤ − 4
√
cos x ≤ 0
Suy ra −1 ≤ y =
√
sin x − 4
√
cos x ≤ 1
Vậy hàm số đạt cực đại tại x = π
2 + k2π, k ∈ Z, giá trị cực đại của hàm
số là y = 1.
Hàm số đạt cực tiểu tại x = k2π, giá trị cực tiểu của hàm số là
y = −1.
Ví dụ 1.12. Xét hàm số:
f(x, y) = (x2
+ y2
)e−(x2
+y2
)
Tìm cực trị của hàm số f.
Đây là một hàm số 2 biến, trong phạm vi chương trình phổ thông ta
không sử dụng được hai quy tắc tìm cực trị trong trường hợp này. Ví dụ
này được giải bằng phương pháp bất đẳng thức như sau.
Lời giải.
Hàm số đã cho xác định trên R.
Ta thấy
f(x; y) ≥ 0, ∀(x; y)
f(0; 0) = 0
Suy ra f(x; y) ≥ f(0; 0), ∀(x; y)
Vậy hàm số đạt cực tiểu tại một điểm duy nhất khi x = y = 0, và giá
trị cực tiểu của hàm số là f(0; 0) = 0.
10
Chương 1. Cực trị hàm số
Ví dụ 1.13. Tìm cực tiểu của hàm số
f(x) = x3 + 2(1 + x3 + 1) + x3 + 2(1 − x3 + 1) với x ≥ 0
Lời giải.
Hàm số đã cho xác định với mọi x ≥ 0 Ta có
f(x) = ( x3 + 1 + 1)2 + ( x3 + 1 − 1)2
= x3 + 1 + 1 + x3 + 1 − 1 ≥ 2 x3 + 1
Suy ra f(x) ≥ 2
Dấu bằng xảy ra khi x = 0
Vậy giá trị cực tiểu của hàm số f(0) = 2
1.3 Một số bài toán tổng quát và ứng dụng
1.3.1 Bài toán tổng quát
Bài toán 1. Cho hàm số f(x) > 0, ∀x ∈ TXĐ và hàm F(x) = cf2
(x),
với c > 0 bất kỳ.
Chứng minh rằng F(x) và f(x) có cùng các điểm cực trị.
Chứng minh.
• Giả sử x0 là điểm cực đại của hàm f(x), tức là
0 < f(x) < f(x0), x ∈ {0 < |x − x0| < δ}
Từ đó suy ra cf2
(x) < cf2
(x0), với c > 0
Hay F(x) < F(x0), x ∈ {0 < |x − x0| < δ}
Vậy x0 cũng là điểm cực đại của hàm F(x).
Tương tự với trường hợp x1 là điểm cực tiểu.
• Giả sử x0 là điểm cực đại của hàm F(x), tức là
0 < F(x) < F(x0), x ∈ {0 < |x − x0| < δ}
Lại có F(x) = cf2
(x), với c > 0, f(x) > 0, x ∈ {0 < |x − x0| < δ}
Từ đó suy ra f(x) < f(x0), x ∈ {0 < |x − x0| < δ}
Vậy x0 cũng là điểm cực đại của hàm f(x).
11
Chương 1. Cực trị hàm số
Tương tự với trường hợp x1 là điểm cực tiểu.
Kết luận: f(x) và F(x) có cùng cực trị.
Nhận xét: Tổng quát hóa bái toán trên, ta được kết quả sau đây:
Cho hàm số f(x) > 0, ∀x ∈ TXĐ và hàm F(x) = cf2n
(x), với c > 0
bất kỳ và n ∈ N∗
.
Khi đó hai hàm F(x) và f(x) có cùng các điểm cực trị.
Ví dụ 1.14. Tìm cực trị của hàm số:
f(x) =
4
x2 + x + 1
Lời giải.
Hàm số đã cho xác định trên R.
Đặt F(x) = f4
(x) = x2
+ x + 1 ⇒ F (x) = 2x + 1
F (x) = 0 ⇔ x = −
1
2
Lập bảng biến thiên của hàm F(x):
x
F (x)
F(x)
−∞ −1
2 +∞
− 0 +
+∞+∞
3
4
3
4
+∞+∞
Từ bảng biến thiên ta thấy hàm F(x) đạt cực tiểu tại x = −1
2
Lại có: f(x) > 0, ∀x ∈ R và hàm F(x) = f4
(x).
Nên hàm F(x) và f(x) có cùng các điểm cực trị.
Do đó hàm f(x) cũng đạt cực tiểu tại x = −1
2, giá trị cực tiểu của hàm
f(x) là f(−1
2) = 4 3
4.
Bài toán 2. Cho hàm số f(x) xác định và liên tục trên D ⊂ R và ϕ(x)
là một hàm đồng biến, liên tục với mọi x ∈ R.
Chứng minh rằng hàm f(x) có cùng cực trị với hàm ϕ[f(x)].
Chứng minh.
Giả sử hàm f(x) đạt cực đại tại điểm x0, tức là f(x) ≤ f(x0), ∀x ∈ D.
12
Chương 1. Cực trị hàm số
Vì ϕ(x) là hàm đồng biến với mọi x ∈ R.
Nên ϕ[f(x)] ≤ ϕ[f(x0)], ∀x ∈ D
Như vậy hàm ϕ[f(x)] cũng đạt cực đại tại điểm x0
Chứng minh tương tự với trường hợp cực tiểu.
Kết luận: hàm f(x) và ϕ[f(x)] có cùng các điểm cực trị.
Ví dụ 1.15. Tìm cực trị của hàm số:
y = 2
x
x2+1
Lời giải.
Hàm số đã cho xác định với mọi x ∈ R.
Vì hàm y = log2 x là hàm đồng biến với x > 0, nên hàm y = 2
x
x2+1 có
cùng các điểm cực trị với hàm f(x) = log2 2
x
x2+1 = x
x2+1.
Ta có f (x) = 1−x2
(x2+1)2 .
f (x) = 0 ⇔ x = ±1
Lập bảng biến thiên của hàm f(x):
x
f (x)
f(x)
−∞ −1 1 +∞
− 0 + 0 −
+∞+∞
−1
2−1
2
1
2
1
2
−∞−∞
Từ bảng biến thiên ta thấy, hàm f(x) đạt cực tiểu tại x = −1 và đạt
cực đại tại x = 1.
Do đó hàm y cũng đạt cực tiểu tại x = −1, giá trị cực tiểu của hàm y
là y(−1) = 1√
2
.
Hàm y đạt cực đại tại x = 1, giá trị cực đại của hàm y là y(1) =
√
2.
Bài toán 3. Cho hàm số ϕ(x) là hàm liên tục, đồng biến với mọi x ∈ R
và f(X) là hàm số đạt cực tiểu tại X0 = ϕ(x0).
Chứng minh rằng hàm f[ϕ(x)] cũng đạt cực tiểu tại x0.
Chứng minh.
Giả sử hàm f(X) đạt cực tiểu tại điểm X0 = ϕ(x0), tức là
f(X) ≥ f(X0), ∀x ∈ (X0 − δ; X0 + δ) với δ > 0
13
Chương 1. Cực trị hàm số
Giả sử ϕ(x0 − α) = X0 − δ , ϕ(x0 + α) = X0 + δ, .
Nên ϕ[f(x)] ≤ ϕ[f(x0)], ∀x ∈ D
Như vậy hàm ϕ[f(x)] cũng đạt cực tiểu tại điểm x0
Chứng minh tương tự với trường hợp cực đại.
Kết luận: hàm f(x) và ϕ[f(x)] có cùng các điểm cực trị.
Ví dụ 1.16. Tìm cực trị của hàm số:
y = sin ln x
Lời giải.
Vì hàm y = sin X là hàm tuần hoàn nên ta chỉ cần xét trong −π
2 ; π
2
Nhận thấy: hàm y = sin X đạt cực đại tại X = π
2 .
Áp dụng kết quả bài toán 3 ta suy ra hàm y = sin ln x đạt cực đại tại
ln x = π
2 ⇔ x = e
π
2 .
Tương tự: y = sin X đạt cực tiểu tại X = −π
2 .
Suy ra hàm y = sin ln x đạt cực tiểu tại ln x = −π
2 ⇔ x = e−π
2 .
1.3.2 Bài tập tham khảo
Bài tập 1.1. Tìm cực trị của hàm số
y = (x − 1)e
x2−5x+6
x−1
Lời giải.
Hàm số dã cho xác định với mọi x = 1
Ta có
y = e
x2−5x+6
x−1 +
x2
− 2x − 1
x − 1
e
x2−5x+6
x−1
y = 0 ⇔
x2
− x − 2
x − 1
e
x2−5x+6
x−1 = 0
⇔
(x + 1)(x − 2)
x − 1
e
x2−5x+6
x−1 = 0
⇔
x = −1
x = 2
Lập bảng biến thiên của hàm số y
14
Chương 1. Cực trị hàm số
x
y
y
−∞ −1 1 2 +∞
− 0 + − 0 +
00
−2e−6
−2e−6
0 +∞
11
+∞+∞
Từ bảng biến thiên ta suy ra:
Giá trị cực tiểu của hàm số y(−1) = −2e−6
, y(2) = 1
Bài tập 1.2. Tìm cực trị của các hàm số sau:
a) y = |x| e−|x−1|
b) y = |x|
1√
2 |1 − x|1− 1√
2
c) y = 1
2(cos x + |cos x|)
Lời giải.
a) Xét các trường hợp:
• Với x < 0, ta có
y =
−x
e1−x
y =
−1 − x
e1−x
; y = 0 ⇔ x = −1
Lập bảng biến thiên của hàm y trong (−∞; 0)
x
y
y
−∞ −1 0
+ 0 −
−∞−∞
1
e2
1
e2
00
Do đó hàm số đạt cực đại tại x = −1, giá trị cực đại của hàm số là
y(−1) = 1
e2 .
• Với 0 ≤ x < 1, ta có
y =
x
e1−x
; y =
x + 1
e1−x
15
Chương 1. Cực trị hàm số
Nhận thấy y > 0, ∀x [0; 1)
Suy ra y ≥ y(0), ∀x [0; 1)
Vậy hàm số đạt cực tiểu tại x = 0, giá trị cực tiểu của hàm số là
y(0) = 0.
• Với x ≥ 1, ta có
y =
x
ex−1
y =
1 − x
ex−1
; y = 0 ⇔ x = 1
Lập bảng biến thiên của hàm y trong (1; +∞)
x
y
y
1 +∞
0 +
11
+∞+∞
Do đó hàm số đạt cực đại tại x = 1, giá trị cực đại của hàm số là
y(1) = 1.
b) Chứng minh tương tự phần a), ta được kết quả
• Tại x = 0 hàm số đạt giá trị cực tiểu y = 0
• Tại x = 1 hàm số đạt giá trị cực tiểu y = 1
• Tại x = 1 + 1√
2
hàm số đạt giá trị cực tiểu
y = (1 +
1
√
2
)
1√
2 (
1
√
2
)1− 1√
2
c) Giá trị cực tiểu của hàm số là y = 0, tại điểm x = π
2 .
Giá trị cực đại của hàm số là y = 1, tại điểm x = 0
Bài tập 1.3. Cho f(x) = xn
+ (a − x)n
với a > 0 và n ∈ Z, n ≥ 2
Tìm cực trị của hàm f(x).
Lời giải.
Hàm số dã cho xác định với mọi x ∈ R
Ta có
f (x) = nxn−1
− n(a − x)n−1
= n xn−1
− (a − x)n−1
16
Chương 1. Cực trị hàm số
f (x) = 0 ⇔ xn−1
= (a − x)n−1
(∗)
• Nếu n chẵn, khi đó
(∗) ⇔ x = a − x ⇔ x =
a
2
• Nếu n lẻ, khi đó
(∗) ⇔
x = a − x
x = x − a
⇔
2x = a
a = 0
⇔ x = a
2
Từ đó suy ra f (x) = 0 ⇔ x = a
2.
Lại có
f (x) =n(n − 1) xn−2
+ (a − x)n−2
⇒ f (
a
2
) =n(n − 1) (
a
2
)n−2
+ (
a
2
)n−2
=2n(n − 1)(
a
2
)n−2
> 0,
a > 0
2 ≤ n ∈ Z
.
Do f (a
2) = 0 và f (a
2) > 0 nên giá trị cực tiểu của hàm số f(a
2) = 2(a
2)n
.
Bài tập 1.4. Cho m, n là các số tự nhiên. Tìm cực trị của hàm số
y = xm
(1 − x)n
Gợi ý:
• Tại x = 0: khi m chẵn, hàm số đạt cực tiểu. Giá trị cực tiểu của hàm
số là: y(0) = 0
Khi m lẻ, hàm số không có cực trị.
• Tại x = 1: khi n chẵn, hàm số đạt cực tiểu. Giá trị cực tiểu của hàm
số là: y(1) = 0
Khi n lẻ, hàm số không có cực trị.
• Tại x = m
m+n hàm số đạt cực đại, giá trị cực đại của hàm số là:
y(
m
m + n
) =
mm
nn
(m + n)m+n
Bài tập 1.5. Tìm cực trị của các hàm số sau:
a)f(x) = e−π2
x2
(
√
2 + sin π2
x2 ) với x = 0
0 với x = 0
17
Chương 1. Cực trị hàm số
b)g(x) =
e− 1
|x| (
√
2 + cos 1
x) với x = 0
0 với x = 0
Lời giải.
a) Hàm số đã cho xác định và liên tục trên R, vì lim
x→0
f(x) = 0
Do
f (x) =
2π2
x3
e−π2
x2
(
√
2 + sin
π2
x2
− cos
π2
x2
)
Nên
f (x) > 0 với x > 0
f (x) < 0 với x < 0
Lại có f(x) − f(0) = e−π2
x2
(
√
2 + sin π2
x2 ) > 0, ∀x = 0
Từ đó suy ra giá trị cực tiểu của hàm số là f(0) = 0
b) Chứng minh tương tự phần a), ta có:
Giá trị cực tiểu của hàm g(x) là g(0) = 0.
18
Chương 2
Giá trị lớn nhất, giá trị nhỏ nhất
2.1 Các khái niệm cơ bản
Trong phần này ta sẽ nêu ra một số khái niệm cơ bản về giá trị lớn
nhất và giá trị nhỏ nhất của hàm số một biến, hàm số nhiều biến và của
một tập hợp.
2.1.1 Giá trị lớn nhất, giá trị nhỏ nhất của hàm số
Định nghĩa 2.1. Cho hàm số y = f(x) xác định trên D ⊂ R. Khi đó:
• Số M được gọi là giá trị lớn nhất của hàm số f(x) trong miền D nếu
đồng thời xảy ra hai điều kiện:
1) f(x) ≤ M với mọi x ∈ D;
2) Tồn tại x1 ∈ D sao cho f(x1) = M.
Ký hiệu: M = max
x∈D
f(x).
• Số m được gọi là giá trị nhỏ nhất của hàm số f(x) trong miền D
nếu đồng thời xảy ra hai điều kiện:
1) f(x) ≥ m với mọi x ∈ D;
2) Tồn tại x2 ∈ D sao cho f(x2) = m.
Ký hiệu: m = min
x∈D
f(x).
Định nghĩa 2.2. Cho hàm số n biến
F = f(x1, x2, ..., xn) : D ⊂ Rn
→ R
19
Chương 2. Giá trị lớn nhất, giá trị nhỏ nhất
Khi đó:
• Số M được gọi là giá trị lớn nhất của hàm số F trong miền D, ký
hiệu M = max
D
F nếu đồng thời xảy ra hai điều kiện:
1) f(x1, x2, ..., xn) ≤ M với mọi (x1, x2, ..., xn) ∈ D;
2) Tồn tại (x0
1, x0
2, ..., x0
n) ∈ D sao cho f(x0
1, x0
2, ..., x0
n) = M.
• Số m được gọi là giá trị nhỏ nhất của hàm số F trong miền D, ký
hiệu m = min
D
F nếu đồng thời xảy ra hai điều kiện:
1) f(x1, x2, ..., xn) ≥ m với mọi (x1, x2, ..., xn) ∈ D;
2) Tồn tại (x1
1, x1
2, ..., x1
n) ∈ D sao cho f(x1
1, x1
2, ..., x1
n) = m.
2.1.2 Giá trị lớn nhất, giá trị nhỏ nhất của một tập hợp
Trong một số bài toán việc tìm giá trị lớn nhất, nhỏ nhất của một
hàm số được đưa về tìm giá trị lớn nhất, giá trị nhỏ nhất của một tập
hợp số.
Định nghĩa 2.3. Cho tập A ⊂ R
• Một số thực α được gọi là giá trị nhỏ nhất của tập A nếu nó đồng
thời thỏa mãn hai điều kiện:
1) α ≤ a với mọi a ∈ A;
2) α ∈ A.
Khi đó ký hiệu: α = min A
Ta phân tích rõ thành hai điều kiện để khi chứng minh α = minA
thì phải chứng minh đủ cả hai điều kiện đó.
• Một số thực β được gọi là giá trị lớn nhất của tập A nếu nó đồng
thời thỏa mãn hai điều kiện:
1) β ≥ a với mọi a ∈ A;
2) β ∈ A.
Khi đó ký hiệu: β = max A
20
Chương 2. Giá trị lớn nhất, giá trị nhỏ nhất
Ví dụ 2.1. Cho A = {c ∈ R|c = (3 − x)(4 − y)(2x + 3y), 0 ≤ x ≤ 3, 0 ≤ y ≤ 4}
Tìm max A.
Lời giải.
Do
0 ≤ x ≤ 3
0 ≤ y ≤ 4
⇔
0 ≤ 3 − x ≤ 3
0 ≤ 4 − y ≤ 4
Áp dụng bất đẳng thức giữa trung bình cộng và trung bình nhân,ta có
c =
1
6
(6−2x)(12−3y)(2x+3y) ≤
1
6
(
6 − 2x + 12 − 3y + 2x + 3y
3
)3
= 36
Dấu bằng xảy ra khi và chỉ khi
6 − 2x = 12 − 3y
6 − 2x = 2x + 3y
⇔
2x − 3y = −6
4x + 3y = 6
⇔
x = 0
y = 2
Suy ra
c ≤ 36, ∀c ∈ A
c1 = (3 − 0)(4 − 2)(0 + 6) = 36 ∈ A
Vậy max A = 36
Ví dụ 2.2. Cho A = {a0, a1, ..., a12}, trong đó các aj là hệ số trong khai
triển
(1 + 2x)12
= a0 + a1x + a2x2
+ ... + a12x12
Tìm max A, min A.
Lời giải.
Ta biết rằng theo khai triển nhị thức Newton ak = Ck
122k
với 0 ≤ k ≤ 12
Giải bất phương trình
ak ≤ ak+1 ⇔ Ck
122k
≤ Ck+1
12 2k+1
⇔ k ≤ 7
Như vậy:
ak ≤ a8, ∀k ≤ 7
ak ≥ a8, ∀k = 9, 10, 11, 12
Vậy max A = a8 = C8
1228
.
Mặt khác a0 = 1, a12 = 212
. Do đó min A = a0 = 1.
21
Chương 2. Giá trị lớn nhất, giá trị nhỏ nhất
2.1.3 Một số tính chất của giá trị lớn nhất, giá trị nhỏ nhất
Tính chất 1. Nếu hàm số y = f(x) có đạo hàm và có hữu hạn các điểm
dừng x1, x2, ..., xn trong (a; b) thì
max
[a;b]
= max {f(x1); f(x2); ...; f(xn); f(a); f(b)}
min
[a;b]
= min {f(x1); f(x2); ...; f(xn); f(a); f(b)}
Tính chất 2. Cho hàm số f(x) xác định trên miền D
(i) Nếu hàm f(x) đồng biến trong [a; b] ⊆ D thì:
min
x∈[a;b]
f(x) = f(a)
max
x∈[a;b]
f(x) = f(b)
(ii) Nếu hàm f(x) nghịch biến trong [a; b] ⊆ D thì:
min
x∈[a;b]
f(x) = f(b)
max
x∈[a;b]
f(x) = f(a)
Tính chất 3. Giả sử hàm số f(x) xác định trên D và A, B là các tập
con của D, trong đó A ⊆ B.
Giả sử tồn tại max
x∈A
f(x), max
x∈B
f(x), min
x∈A
f(x), min
x∈B
f(x).
Khi đó ta có:
max
x∈A
f(x) ≤ max
x∈B
f(x)
min
x∈A
f(x) ≥ min
x∈B
f(x)
Tính chất 4. Giả sử hàm số f(x) xác định trên miền D và D = D1 ∪D2.
Nếu tồn tại các giá trị:
m1 = min
x∈D1
f(x); m2 = min
x∈D2
f(x)
M1 = max
x∈D1
f(x); M2 = max
x∈D2
f(x)
Thì:
min
x∈D
f(x) = min {m1; m2}
max
x∈D
f(x) = max {M1; M2}
22
Chương 2. Giá trị lớn nhất, giá trị nhỏ nhất
Tính chất 5. Giả sử hàm số f(x) xác định trên D và tồn tại min
x∈D
f(x), max
x∈D
f(x).
Khi đó ta có:
max
x∈D
f(x) = − min
x∈D
(−f(x))
min
x∈D
f(x) = − max
x∈D
(−f(x))
Nếu f(x) > 0 với mọi x ∈ D thì:
max
x∈D
(
1
f(x)
) =
1
min
x∈D
f(x)
min
x∈D
(
1
f(x)
) =
1
max
x∈D
f(x)
Tính chất 6. Giả sử hàm số f(x) xác định trên miền D.
Khi đó với mọi n nguyên dương, ta có:
max
x∈D
f(x) = 2n+1 max
x∈D
(f2n+1(x))
min
x∈D
f(x) = 2n+1 min
x∈D
(f2n+1(x))
Nếu f(x) ≥ 0, ∀x ∈ D. Khi đó với mọi n nguyên dương, ta có:
max
x∈D
f(x) = 2n max
x∈D
(f2n(x))
min
x∈D
f(x) = 2n min
x∈D
(f2n(x))
Tính chất 7. Giả sử f(x) là hàm số xác định trên D và tồn tại max
x∈D
f(x), min
x∈D
f(x).
Khi đó ta có:
max
x∈D
|f(x)| = max max
x∈D
f(x) ; min
x∈D
f(x)
min
x∈D
f(x) =



0 nếu f(x) = 0
min max
x∈D
f(x) ; min
x∈D
f(x) nếu f(x) = 0
Tính chất 8. Cho các hàm số f1(x), f2(x), ..., fn(x) cùng xác định trên
miền D.
Đặt f(x) = f1(x) + f2(x) + ... + fn(x). Giả sử tồn tại max
x∈D
f(x), min
x∈D
f(x)
23
Chương 2. Giá trị lớn nhất, giá trị nhỏ nhất
và max
x∈D
fi(x), min
x∈D
fi(x) với mọi i = 1, n. Khi đó ta có:
max
x∈D
f(x) ≤ max
x∈D
f1(x) + max
x∈D
f2(x) + ... + max
x∈D
fn(x) (1)
min
x∈D
f(x) ≥ min
x∈D
f1(x) + min
x∈D
f2(x) + ... + min
x∈D
fn(x) (2)
Dấu bằng trong (1) xảy ra khi và chỉ khi tồn tại x0 ∈ D sao cho
max
x∈D
fi(x) = fi(x0), ∀i = 1, n.
Dấu bằng trong (2) xảy ra khi và chỉ khi tồn tại x1 ∈ D sao cho
min
x∈D
fi(x) = fi(x1), ∀i = 1, n.
2.1.4 Một số định lý về giá trị lớn nhất, giá trị nhỏ nhất
Định lý 2.1. (Điều kiện đủ để tồn tại giá trị lớn nhất, giá trị nhỏ nhất)
Nếu hàm số f(x) liên tục trên [a; b] thì f(x) đạt giá trị lớn nhất, giá trị
nhỏ nhất trên đoạn đó.
Định lý 2.2. Cho hàm số f(x) có đạo hàm cấp một liên tục trên [a; b],
có đạo hàm cấp hai tại mọi điểm x ∈ (a; b) sao cho f (x) ≥ 0 với mọi
x ∈ (a; b). Khi đó, với mọi x, y ∈ [a; b] ta luôn có:
f(x) ≥ f(y) + f (y)(x − y) (2.2)
Hay f(x) = max
a≤y≤b
(f(y) + f (y)(x − y))
Chứng minh.
1) Xét x = y thì dấu đẳng thức của (2.2) xảy ra.
2) Xét x > y thì
(2.2) ⇔
f(x) − f(y)
x − y
≥ f (y) (1)
Theo nguyên lý Lagrange suy ra: ∃y0 ∈ (x; y) sao cho
f(x) − f(y)
x − y
= f (y0)
Vậy (1) có dạng f (y0) ≥ f (y) với y < y0 < x (2)
Ta thấy (2) là hiển nhiên vì f (x) ≥ 0 nên f (x) là hàm đơn điệu
tăng.
24
Chương 2. Giá trị lớn nhất, giá trị nhỏ nhất
3) Xét x < y thì
(2.2) ⇔
f(x) − f(y)
x − y
≤ f (y) (3)
Theo nguyên lý Lagrange suy ra: ∃y1 ∈ (x; y) sao cho
f(x) − f(y)
x − y
= f (y1)
Vậy (3) có dạng f (y1) ≤ f (y) với y < y1 < x (4)
Ta thấy (4) là hiển nhiên vì f (x) ≥ 0 nên f (x) là hàm đơn điệu
tăng. 2
Khi f (x) ≤ 0, chứng minh tương tự ta có định lý sau đây:
Định lý 2.3. Cho hàm số f(x) có đạo hàm cấp một liên tục trên [a; b],
có đạo hàm cấp hai tại mọi điểm x ∈ (a; b) sao cho f (x) ≤ 0 với mọi
x ∈ (a; b). Khi đó, với mọi x, y ∈ [a; b] ta luôn có:
f(x) ≤ f(y) + f (y)(x − y) (2.3)
Hay f(x) = min
a≤y≤b
(f(y) + f (y)(x − y))
Ví dụ 2.3. Cho tam giác ABC bất kỳ.Tìm giá trị lớn nhất của các biểu
thức
M = cos
A
2
+ cos
B
2
+ cos
C
2
Lời giải.
Xét hàm số f(x) = cos x, x ∈ (0; π
2 ) có f (x) = − cos x < 0, ∀x ∈ (0; π
2 ).
Theo định lý (2.3) với x = A
2 và y = π
6 ta được
cos
A
2
≤ cos
π
6
− sin
π
6
(
A
2
−
π
6
)
Dấu đẳng thức xảy ra khi A
2 = π
6
Chứng minh tương tự, ta có:
cos
B
2
≤ cos
π
6
− sin
π
6
(
B
2
−
π
6
)
cos
C
2
≤ cos
π
6
− sin
π
6
(
C
2
−
π
6
)
25
Chương 2. Giá trị lớn nhất, giá trị nhỏ nhất
Cộng các bất đẳng thức trên vế theo vế
M ≤ 3 cos
π
6
− sin
π
6
(
A + B + C
2
−
π
2
)
Dấu đẳng thức xảy ra khi A
2 = B
2 = C
2 = π
6
Vậy max M = 3
√
3
2 khi ABC là tam giác đều.
2.2 Các phương pháp tìm giá trị lớn nhất, giá trị
nhỏ nhất
Để tìm giá trị lớn nhất, giá trị nhỏ nhất của hàm số ta có thế sử
dụng nhiều phương pháp. Trong thực tế, chúng ta có thể bắt gặp những
bài toán chỉ có thể giải bằng một phương pháp, nhưng cũng có những
bài toán có thể giải bằng nhiều phương pháp hay những bài toán cần
phối hợp nhiều phương pháp mới giải quyết được.Điều đó nói lên tính
đa dạng, phong phú của các phương pháp giải toán nói chung và phương
pháp tìm giá trị lớn nhất, giá trị nhỏ nhất nói riêng. Vì vậy, việc nắm
vững các phương pháp sẽ giúp người làm toán giải quyết được nhiều
dạng bài tập.
Trong phần này, luận văn sẽ trình bày một số phương pháp tìm giá
trị lớn nhất, giá trị nhỏ nhất và các ví dụ áp dụng.
2.2.1 Phương pháp sử dụng đạo hàm
a. Cơ sở lý thuyết.
Dựa vào tính chất 1 của giá trị lớn nhất, giá trị nhỏ nhất, ta xây dựng
quy tắc tìm giá trị lớn nhất, giá trị nhỏ nhất của hàm số sử dụng đạo
hàm như sau:
Quy tắc:
1) Tìm các điểm dừng của hàm số đã cho;
Điểm dừng là điểm thuộc tập xác định của hàm số mà tại đó đạo
hàm của hàm số bằng không hoặc không xác định.
2) Tính các giá trị của hàm số tại các điểm dừng và hai đầu mút;
3) So sánh các giá trị tìm được, rồi kết luận giá trị lớn nhất, giá trị nhỏ
nhất của hàm số.
26
Chương 2. Giá trị lớn nhất, giá trị nhỏ nhất
b. Các ví dụ áp dụng.
Ví dụ 2.4. (Đề thi tuyển sinh Đại học và Cao đẳng khối B năm 2004)
Tìm giá trị lớn nhất, giá trị nhỏ nhất của hàm số:
f(x) =
ln2
(x)
x
trên 1; e3
Lời giải.
Nhận thấy f(x) xác định và liên tục với mọi x ∈ 1; e3
.
Ta có
f (x) =
lnx(2 − lnx)
x2
f (x) = 0 ⇔ x = e2
Lập bảng biến thiên của hàm f(x):
x
f (x)
f
1 e2
e3
+ 0 −
00
4
e2
4
e2
9
e3
9
e3
Từ bảng biến thiên ta suy ra:
max
x∈[1;e3]
f(x) = f(e2
) =
4
e2
min
x∈[1;e3]
f(x) = f(1) = 0
Ví dụ 2.5. Tìm giá trị lớn nhất, giá trị nhỏ nhất (nếu có) của hàm số:
y =
1
cos x
+
1
sin x
với x ∈ 0;
π
2
Lời giải.
Nhận thấy:
y =
1
cos x
+
1
sin x
=
sin x + cos x
sin x cos x
Đặt sin x + cos x = t. Suy ra t ∈ 1;
√
2 .
Khi đó hàm số đã cho có dạng:
g(t) =
2t
t2 − 1
27
Chương 2. Giá trị lớn nhất, giá trị nhỏ nhất
Nhận xét: hàm g(t) xác định và liên tục với mọi t ∈ 1;
√
2 .
Ta có
g (t) = −
2 + 2t2
(t2 − 1)2
< 0, ∀t ∈ 1;
√
2
Lập bảng biến thiên của hàm g(t):
x
g (x)
g(x)
1
√
2
−
+∞
2
√
22
√
2
Từ bảng biến thiên ta suy ra min
t∈(1;
√
2]
g(t) = 2
√
2; và không tồn tại
max
t∈(1;
√
2]
g(t).
Vậy không tồn tại max
x∈(0;π
2 )
f(x);
min
x∈(0;π
2 )
f(x) = 2
√
2 ⇔ x = π
2 .
Ví dụ 2.6. Cho x, y là các số thực không âm, thỏa mãn x + y = 1. Tìm
giá trị lớn nhất và nhỏ nhất của biểu thức:
M = 22x
+ 2y
Lời giải.
Từ giả thiết x + y = 1, ta suy ra y = 1 − x
Khi đó biểu thức M có dạng:
M = 22x
+ 21−x
= 22x
+
2
2x
Đặt 2x
= t ⇒ 1 ≤ t ≤ 2
Ta có:
M = f(t) = t2
+
2
t
, với 1 ≤ t ≤ 2
f (t) = 2t − 2
t2 = 2t3
−2
t2 xác định và liên tục trên [1; 2]
f (t) = 0 ⇔ t = 1
Lập bảng biến thiên của hàm f(t) trong [1; 2]
28
Chương 2. Giá trị lớn nhất, giá trị nhỏ nhất
t
f (t)
f(t)
1 2
0 +
33
55
Từ bảng biến thiên ta thấy
max
1≤t≤2
f(t) = 5 ; min
1≤t≤2
f(t) = 3
Kết luận:
max M = 5 ⇔
x + y = 1
2x
= 2
⇔
x = 1
y = 0
min M = 3 ⇔
x + y = 1
2x
= 1
⇔
x = 0
y = 1
c. Bài tập tham khảo.
Bài tập 2.1. (Đề thi học sinh giỏi Toán Quốc gia năm 1991)
Chứng minh rằng
sin x + 2 cos x
2
cos x + 2 sin x
2
≥
2 +
√
2
2
với 0 ≤ x ≤
π
2
Lời giải.
Xét sự biến thiên của hàm số
f(x) =
sin x + 2 cos x
2
cos x + 2 sin x
2
trong 0;
π
2
Nhận thấy với x ∈ 0; π
2 thì cos x ≥ 0 và sin x
2 ≥ 0 nên cos x+2 sin x
2 > 0
Đạo hàm f (x) có tử thức
(cos x − sin
x
2
)(cos x + 2 sin
x
2
) − (sin x + 2 cos
x
2
)(− sin x + cos
x
2
)
= sin x. cos
x
2
+ sin
x
2
. cos x − 1
= sin(x +
x
2
) − 1 ≤ 0
29
Chương 2. Giá trị lớn nhất, giá trị nhỏ nhất
Vậy hàm số f(x) nghịch biến trong 0; π
2
Do đó f(x) ≥ f(π
2 ) = 2+
√
2
2
Bài tập 2.2. Tìm giá trị lớn nhất, giá trị nhỏ nhất của hàm số
y =
2 cos2
x + |cos x| + 1
|cos x| + 1
trong đoạn [0; 2]
Gợi ý: Đặt |cos x| = t, t ∈ [0; 1]
Đáp số:
max y = 2 ⇔ x = kπ
min y = 1 ⇔ x =
π
2
+ kπ (k ∈ Z)
Bài tập 2.3. Tìm giá trị lớn nhất, giá trị nhỏ nhất của hàm số
y = x3
+ 3x2
− 72x + 90 trong đoạn [−5; 5]
Gợi ý: Đặt f(x) = x3
+ 3x2
− 72x + 90
Lập bảng biến thiên, tìm giá trị lớn nhất và nhỏ nhất của hàm f(x) rồi
dựa vào tính chất 7 để suy ra giá trị lớn nhất và nhỏ nhất của hàm y.
Đáp số:
max y = f(−5) = 400
min y = 0
Bài tập 2.4. Cho x; y là các số thực không âm, thỏa mãn x + y = 1.
Tìm giá trị lớn nhất và nhỏ nhất của biểu thức:
P =
x
y + 1
+
y
x + 1
Đáp số:
min P =
2
3
⇔
x = 1
2
y = 1
2
max P = 1 ⇔
x = 0; y = 1
x = 1; y = 2
Bài tập 2.5. Cho xyz = 1 và 1
4 ≤ x ≤ 1; y, z ≥ 1. Tìm giá trị nhỏ nhất
của biểu thức
P =
1
1 + x
+
1
1 + y
+
1
1 + z
30
Chương 2. Giá trị lớn nhất, giá trị nhỏ nhất
Gợi ý: Đặt t =
√
xy; 1 ≤ t ≤ 2
Khi đó
P = f(t) =
2t
(t2 + 1)2
−
2
(1 + t)2
Lập bảng biến thiên, tìm giá trị nhỏ nhất của hàm f(t) ta được
min P = min
1≤t≤2
f(t) =
22
15
khi x =
1
4
; y = z = 2.
2.2.2 Phương pháp tập giá trị
a. Cơ sở lý thuyết.
Định nghĩa 2.4. Cho hàm số y = f(x) xác định trên tập D ⊂ R. Khi
x biến thiên trên tập D thì tương ứng y biến thiên trên tập T ⊂ R.
Khi đó tập T được gọi là tập giá trị của hàm số tương ứng với tập
xác định D của nó.
Nhận xét: Giả sử hàm số f(x) xác định và liên tục trong miền D và
phương trình f(x) = y có nghiệm x ∈ D khi và chỉ khi y ∈ T. Khi đó:
Nếu T = [a; b] thì min
x∈D
f(x) = a; max
x∈D
f(x) = b
Nếu T = (a; b] thì max
x∈D
= b
Nếu T = [a; b) thì min
x∈D
= a
Sau đây ta sẽ xét một số bài toán tìm giá trị lớn nhất, nhỏ nhất của
hàm số sử dụng phương pháp tập giá trị.
b. Các ví dụ áp dụng.
Ví dụ 2.7. Tìm giá trị lớn nhất, giá trị nhỏ nhất của hàm số:
f(x) =
x2
− 2x + 2
x2 + 2x + 2
Lời giải.
Vì x2
+ 2x + 2 > 0 với mọi x ∈ R nên hàm số đã cho xác định và liên
tục với mọi x ∈ R.
Xét phương trình ẩn x, tham số y
x2
− 2x + 2
x2 + 2x + 2
= y (2.7)
31
Chương 2. Giá trị lớn nhất, giá trị nhỏ nhất
Ta có
(2.7) ⇔ (y − 1)x2
+ 2(y + 1)x + 2(y − 1) = 0
1) Xét y = 1 ⇒ x = 0
2) Xét y = 0. Để phương trình có nghiệm:
∆ = (y + 1)2
− 2(y − 1)2
≥ 0
⇔ −y2
+ 6y − 1 ≥ 0
⇔ 3 − 2
√
2 ≤ y ≤ 3 + 2
√
2
So sánh với giá trị y = 1 ta được
max f(x) = 3 + 2
√
2 ⇔ x = −
√
2
min f(x) = 3 − 2
√
2 ⇔ x =
√
2
Ví dụ 2.8. Tìm giá trị lớn nhất, giá trị nhỏ nhất của hàm số:
y = sin 2x + 3 cos2
x − sin2
x
Lời giải.
Viết hàm số đã cho dưới dạng:
y =
− sin2
x + 2 sin x cos x + 3 cos2
x
sin2
x + cos2 x
1) Xét cos x = 0 ⇒ y = −1
2) Xét cos x = 0. Viết y dưới dạng:
y =
−t2
+ 2t + 3
t2 + 1
, với t = tan x (2.8)
Ta có
(2.8) ⇔ (y + 1)t2
− 2t + y − 3 = 0
Cần xác định y để phương trình trên có nghiệm.
Ta thấy
∆ = 1 − (y + 1)(y − 3) ≥ 0
⇔ −y2
+ 2y + 4 ≥ 0
⇔ 1 −
√
5 ≤ y ≤ 1 +
√
5
32
Chương 2. Giá trị lớn nhất, giá trị nhỏ nhất
Vậy
max y = 1 +
√
5 ⇔ x = arctan
1
2 +
√
5
min y = 1 −
√
5 ⇔ x = arctan
1
2 −
√
5
Ví dụ 2.9. Cho 2x2
+ y2
+ xy ≥ 1. Tìm giá trị nhỏ nhất của biểu thức:
M = x2
+ y2
Lời giải.
Đặt 2x2
+ y2
+ xy = a, a ≥ 1
Khi đó:
M
a
=
x2
+ y2
2x2 + y2 + xy
1) Nếu y = 0thì
M
a
=
1
2
2) Nếu y = 0 thì
M
a
=
t2
+ 1
2t2 + t + 1
, t =
x
y
Cần xác định giá trị của M
a sao cho phương trình
M
a
=
t2
+ 1
2t2 + t + 1
có nghiệm.
⇔ (2
M
a
− 1)t2
+
M
a
t +
M
a
− 1 = 0 có nghiệm
⇔ ∆ = (
M
a
)2
− 4(2
M
a
− 1)(
M
a
− 1) ≥ 0
⇔ M ≥
6 − 2
√
2
7
a ≥
6 − 2
√
2
7
= M0
Vậy min M = 6−2
√
2
7 khi
x
y = − M0
2(2M0−1)
2x2
+ y2
+ xy = 1
⇔



x = M1y
y = ±
√
2(1−2M0)
√
2−7M0+7M2
0
với M1 = −
M0
2(2M0 − 1)
33
Chương 2. Giá trị lớn nhất, giá trị nhỏ nhất
c.Bài tập tham khảo.
Bài tập 2.6. Tìm giá trị lớn nhất, giá trị nhỏ nhất của hàm số
f(x) =
2 sin x − 3 cos x + 1
sin x + 2 cos x − 3
Đáp số:
max f(x) = 2 ⇔ x = 2π
min f(x) = −
3
2
⇔ x =
π
2
Bài tập 2.7. Tìm giá trị lớn nhất, giá trị nhỏ nhất của hàm số
f(x) =
x2
cos α − 2x + cos α
x2 − 2x cos α + 1
(α là một hằng số thực) Đáp số:
max f(x) = 1 ⇔ x = −1
min f(x) = −1 ⇔ x = 1
Bài tập 2.8. Tìm giá trị nhỏ nhất của hàm số
f(x) = x + x2 +
1
x
với x > 0
Lời giải.
Hàm số đã cho xác định và liên tục với mọi x > 0.
Xét phương trình ẩn x, tham số m
x + x2 +
1
x
= m (∗)
Phương trình (∗) có nghiệm x > 0 khi và chỉ khi
0 < x ≤ m
x2
+ 1
x = (m − x)2
⇔
0 < x ≤ m
2mx2
− m2
x + 1 = 0 (∗∗)
Ta thấy (∗∗) có nghiệm khi và chỉ khi
m4
− 8m ≥ 0 ⇔ m ≥ 2
34
Chương 2. Giá trị lớn nhất, giá trị nhỏ nhất
Lại có: P =
1
2m
> 0; S =
m2
2m
=
m
2
> 0
Do đó (∗∗) có hai nghiệm dương x1 ≥ x2.
Vì x1 + x2 = m
2 nên m > x1 ≥ x2
Vậy m ≥ 2, phương trình (∗) có nghiệm x > 0
Suy ra f(x) ≥ 2, ∀x > 0
Kết luận: min
x>0
f(x) = 2 ⇔ x = 1
2.
Bài tập 2.9. (Đề thi tuyển sinh Đại học, Cao đẳng khối B năm 2008)
Giả sử x, y là hai số thực thỏa mãn x2
+ y2
= 1. Tìm giá trị lớn nhất,
nhỏ nhất của biểu thức
P =
2(x2
+ 6xy)
1 + 2xy + 2y2
Lời giải.
Do x2
+ y2
= 1 nên ta có
P =
2(x2
+ 6xy)
x2 + 2xy + 3y2
• Nếu y = 0 ⇒ x = 1
Khi đóP = 2
• Nếu y = 0, chia cả tử thức và mẫu thức của P cho y2
= 0 ta được
P =
2 (x
y )2
+ 6x
y
(x
y )2 + 2x
y + 3
Đặt x
y = t, t ∈ R
Bài toán quy về tìm giá trị lớn nhất, nhỏ nhất của hàm số
f(t) =
2t2
+ 12t
t2 + 2t + 3
, t ∈ R
Xét phương trình ẩn t, tham số một
2t2
+ 12t
t2 + 2t + 3
= m (1)
Vì t2
+ 2t + 3 > 0, ∀t nên
(1) ⇔ (m − 2)t2
+ 2(m − 6)t + 3m = 0 (2)
35
Chương 2. Giá trị lớn nhất, giá trị nhỏ nhất
+) Nếu m = 2: (2) ⇔ t = 3
4
+) Nếu m = 2: (2) có nghiệm khi và chỉ khi
m2
+ 3m − 18 ≤ 0
⇔ −6 ≤ m ≤ 3
Vậy
max f(t) = 3 ⇔ t = 3
min f(t) = −6 ⇔ t = −
3
2
Kết luận:
max P = 3 ⇔
x = 3√
10
; y = 1√
10
x = − 3√
10
; y = − 1√
10
min P = −6 ⇔
x = − 3√
13
; y = 2√
13
x = 3√
13
; y = − 2√
13
Bài tập 2.10. Tìm giá trị lớn nhất, nhỏ nhất của hàm số:
y =
3
√
x + 3 + 4
√
1 − x + 1
4
√
x + 3 + 3
√
1 − x + 1
Gợi ý:
Từ điều kiện − 3 ≤ x ≤ 1 và do (
√
x + 3)2
+ (
√
1 − x)2
= 4,
đặt
√
x + 3 = 2.2t
1+t2
√
1 − x = 2(1−t2
)
1+t2
0 ≤ t ≤ 1
Khi đó
y =
−7t2
+ 12t + 9
−5t2 + 16t + 7
Đáp số:
max y =
9
7
⇔ x = −3
min y =
7
9
⇔ x = 1
36
Chương 2. Giá trị lớn nhất, giá trị nhỏ nhất
2.2.3 Phương pháp lượng giác
Một số bài toán tìm giá trị lớn nhất, nhỏ nhất có thể giải bằng phương
pháp lượng giác. Bằng các phép biến đổi lượng giác, đánh giá lượng giác
ta đưa biểu thức và điều kiện của bài toán về dạng lượng giác.
a.Cơ sở lý thuyết.
Định lý 2.4. Nếu đại lượng x biến thiên trên miền D thì luôn đặt được
x = ϕ(t) với t ∈ D1.
Trong đó: ϕ(t) là một hàm số lượng giác nào đó; miền D1 được chọn
sao cho ánh xạ ϕ : D1 → D là song ánh.
Quy tắc.
- Đặt x = ϕ(t). Chuyển điều kiện x ∈ D về điều kiện tương đương
t ∈ D1;
- Chuyển bài toán đại số đã cho về bài toán lượng giác mới;
- Giải bài toán lượng giác mới thu được;
- Kết luận cho bài toán đại số ban đầu.
b.Ví dụ áp dụng.
Ví dụ 2.10. Tìm giá trị lớn nhất, giá trị nhỏ nhất của hàm số
y =
1 + x4
(1 + x2)2
Lời giải.
Hàm số đã cho xác định với mọi x ∈ R.
Đặt x = tan t với t ∈ (−π
2 ; π
2 ).
Khi đó:
y =
1 + tan4
t
(1 + tan2 t)2
= sin4
t + cos4
t = 1 −
1
2
sin2
2t
Suy ra 1
2 ≤ y ≤ 1
Vậy
min y =
1
2
⇔ x = 1
max y = 1 ⇔ x = 0
37
Chương 2. Giá trị lớn nhất, giá trị nhỏ nhất
Ví dụ 2.11. (Đề thi tuyển sinh Đại học, Cao đẳng khối D năm 2008)
Cho x ≥ 0; y ≥ 0. Tìm giá trị lớn nhất, giá trị nhỏ nhất của biểu thức
P =
(x − y)(1 − xy)
(1 + x)2(1 + y)2
Lời giải.
Biến đổi biểu thức P về dạng
P =
x
(1 + x)2
−
y
(1 + y)2
Do x ≥ 0; y ≥ 0.
Đặt
x = tan2
α, 0 ≤ α < π
2
y = tan2
β, 0 ≤ β < π
2
Khi đó
P =
tan2
α
(1 + tan2 α)2
−
tan2
β
(1 + tan2 β)2
= sin2
α cos2
α − sin2
β cos2
β
=
1
4
sin2
2α −
1
4
sin2
2β
Suy ra −1
4 ≤ P ≤ 1
4, ∀α, β ∈ 0; π
2
Vậy
max P =
1
4
⇔ x = 1; y = 0
min P = −
1
4
⇔ x = 0; y = 1
Ví dụ 2.12. Cho x2
+ y2
= 1. Tìm giá trị lớn nhất, giá trị nhỏ nhất của
biểu thức
M = 16(x5
+ y5
) − 20(x3
+ y3
) + 5(x + y)
Lời giải.
38
Chương 2. Giá trị lớn nhất, giá trị nhỏ nhất
Do x2
+ y2
= 1, nên ta có thể đặt
x = sin α
y = cos α
α ∈ [0; 2π]
Ta có
sin 5α = sin 3α cos 2α + sin 2α cos 3α
= (3 sin α − 4 sin3
α)(1 − 2 sin2
α) + 2 sin α cos α(4 cos3
α − 3 cos α)
= (3 sin α − 4 sin3
α)(1 − 2 sin2
α) + 2 sin α cos2
α(4 cos2
α − 3)
= 16 sin5
α − 20 sin3
α + 5 sin α
Biến đổi tương tự ta được
cos 5α = 16 cos5
α − 20 cos3
α + 5 cos α
Suy ra
M = sin5
α + cos5
α =
√
2 cos(5α −
π
4
)
Vậy
max M =
√
2 ⇔ x = sin
π
20
; y = cos
π
20
min M = −
√
2 ⇔ x =
√
2
2
; y =
√
2
2
Nhận xét: Một số dấu hiệu nhận biết bài toán có thể giải bằng phương
pháp lượng giác:
1) Tập biến thiên D của một biến số là tập giá trị của một hàm lượng
giác nào đó.
2) Trong giả thiết có một bộ phận tương tự với công thức lượng giác
nào đó:
• Bộ phận 1 + x2
tương tự với 1 + tan2
t = 1
cos2 t.
• Bộ phận 4x3
− 3x tương tự với 4 cos3
t − 3 cos t = cos 3t.
• Bộ phận 2x2
− 1 tương tự với 2 cos2
t − 1 = cos t
• Bộ phận 2x
1−x2 tương tự với tan 2t = 2 tan t
1−tan2
t
• Bộ phận 2x
1+x2 tương tự với sin 2t = 2 tan t
1+tan2
t
• Bộ phận x+y
1−xy tương tự với tan(α + β) = tan α+tan β
1−tan α tan β
39
Chương 2. Giá trị lớn nhất, giá trị nhỏ nhất
c.Bài tập tham khảo.
Bài tập 2.11. Cho x, y, z ∈ [0; 1]. Tìm giá trị lớn nhất của biểu thức
M =
√
xyz + (1 − x)(1 − y)(1 − z)
Lời giải.
Theo giả thiết x, y, z ∈ [0; 1], do đó ta đặt
x = sin2
A, y = sin2
B, z = sin2
C vớiA, B, C ∈ 0;
π
2
Suy ra 0 ≤ sin A ≤ 1; 0 ≤ sin B ≤ 1; 0 ≤ sin C ≤ 1; 0 ≤ cos A ≤ 1; 0 ≤
cos B ≤ 1; 0 ≤ cos C ≤ 1
Khi đó
M = sin2
A sin2
B sin2
C +
√
cos2 A cos2 B cos2 C
= sin A sin B sin C + cos A cos B cos C (1)
Nhận thấy:
+) sin A sin B sin C ≤ sin A sin B (2)
Dấu bằng xảy ra ⇔
sin C = 1
sin A sin B = 0
⇔


z = 1
x = 0
y = 0
(∗)
+) cos A cos B cos C ≤ cos A cos B (3)
Dấu bằng xảy ra ⇔
cos C = 1
cos A cos B = 0
⇔


z = 0
x = 1
y = 1
(∗∗)
Từ (1), (2), (3) suy ra
M ≤ sin A sin B + cos A cos B ⇒ M ≤ cos(A − B) (4)
Vì cos(A − B) ≤ 1 nên M ≤ 1
Kết luận
min M = 1 ⇔
x = y = z = 1
x = y = z = 0
40
Chương 2. Giá trị lớn nhất, giá trị nhỏ nhất
Bài tập 2.12. Cho x, y, z là các số thực dương và thỏa mãn điều kiện
x + y + z = xyz. Tìm giá trị lớn nhất của biểu thức
P =
1
√
1 + x2
+
1
1 + y2
+
1
√
1 + z2
Lời giải.
Từ giả thiết x + y + z = xyz ta suy ra
1
xy
+
1
yz
+
1
zx
= 1
Do x, y, z là các số thực dương, nên đặt x = cot A; y = cot B; z = cot C
với A, B, C ∈ 0; π
2 .
Khi đó ta có:
1
xy
+
1
yz
+
1
zx
= 1 ⇔ tan A tan B + tan B tan C + tan C tan A = 1
⇔ tan A(tan B + tan C) = 1 − tan B tan C
⇔ tan A =
1 − tan B tan C
tan B + tan C
= cot(B + C)
⇔ A = B + C =
π
2
Do đó P có dạng
P =
1
√
1 + cot2 A
+
1
√
1 + cot2 B
+
1
√
1 + cot2 C
= sin A + sin B + sin C
Mặt khác
sin A + sin B + sin C
3
≤ sin
A + B + C
3
= sin
π
6
=
1
2
Suy ra sin A + sin B + sin C ≤ 3
2
Vậy
min P =
3
2
⇔ A = B = C =
π
6
⇔ x = y = z =
√
3
41
Chương 2. Giá trị lớn nhất, giá trị nhỏ nhất
Bài tập 2.13. (Đề thi học sinh giỏi Toán Quốc gia năm 1999)
Cho a, b, c > 0 và abc + a + c = b. Tìm giá trị lớn nhất của biểu thức
M =
2
a2 + 1
−
2
b2 + 1
+
3
c2 + 1
Lời giải.
Từ giả thiết ta có
ac +
a
b
+
c
b
= 1 (1)
Vì a, b, c > 0 nên đặt
a = tan
A
2
,
1
b
= tan
B
2
, c = tan
C
2
với A, B, C ∈ 0;
π
2
Khi đó (1) có dạng:
tan
A
2
tan
C
2
+ tan
A
2
tan
B
2
+ tan
B
2
tan
C
2
= 1
Suy ra A
2 + B
2 + C
2 = π
2 hay A + B + C = π
Ta có: 1 + a2
= 1 + tan2 A
2
=
1
cos2 A
2
1 + b2
= 1 + cot2 B
2
=
1
sin2 B
2
1 + c2
= 1 + tan2 C
2
=
1
cos2 C
2
Vậy P = 2 cos2 A
2
− 2 sin2 B
2
+ 3 cos2 C
2
= 1 + cos A − (1 − cos B) + 3 cos2 C
2
= 2 cos
A + B
2
cos
A − B
2
+ 3 cos2 C
2
= −3 sin2 C
2
+ 2 sin
C
2
cos
A − B
2
+ 3
= 3 − 3(sin2 C
2
−
2
3
sin
C
2
cos
A − B
2
)
= 3 − 3 (sin
C
2
−
1
3
sin
C
2
cos
A − B
2
)2
−
1
9
cos2 A − B
2
= 3 +
1
3
cos2 A − B
2
− 3(sin
C
2
−
1
3
cos
A − B
2
)2
≤
10
3
42
Chương 2. Giá trị lớn nhất, giá trị nhỏ nhất
Dấu bằng xảy ra ⇔
cos A−B
2 = 1
sin C
2 = 1
3
Kết luận max P = 10
3 ⇔ x = 1√
2
; y =
√
2; z = 1
2
√
2
.
Bài tập 2.14. Cho |a| ≥ 1. Tìm giá trị lớn nhất và giá trị nhỏ nhất của
biểu thức
P =
5 − 12
√
a2 − 1
a2
Lời giải.
Đặt x = 1
cos α với α ∈ 0; π
2 ∪ π; 3π
2
Khi đó
5 − 12
√
a2 − 1
a2
= (5 − tan α) cos2
α = 5 cos2
α − 12 sin α cos α
=
5
2
(1 + cos 2α) − 6 sin 2α =
5
2
+
13
2
(
5
13
cos 2α −
12
13
sin 2α)
=
5
2
+
13
2
cos(2α + β) với cos β =
5
13
, sin β =
12
13
Vì −1 ≤ cos(2α + β) ≤ 1 nên
−4 ≤
5 − 12
√
a2 − 1
a2
≤ 9
Vậy
max P = 9 ⇔ a =
1
cos β
2
,
min y = 2 ⇔ a =
1
cos π−β
2
với cos β =
5
13
, sin β =
12
13
Bài tập 2.15. Cho x ∈ [−1; 1] và n ≥ 2. Tìm giá trị lớn nhất và giá trị
nhỏ nhất của
y = (1 + x)n
+ (1 − x)n
Gợi ý: Đặt x = cos 2t, 0 ≤ t ≤ π
2
Đáp số:
max y = 2n
⇔ x = 1
min y = 2 ⇔ x = 0
43
Chương 2. Giá trị lớn nhất, giá trị nhỏ nhất
2.2.4 Phương pháp hình học
a. Cơ sở lý thuyết.
Khi bài toán tìm giá trị lớn nhất, giá trị nhỏ nhất có tiềm ẩn các yếu
tố hình học, ta có thể sử dụng các đánh giá hình học để giải quyết bài
toán.
Một số đánh giá hình học quen thuộc:
1) Với các vectơ −→u , −→v bất kì ta luôn có
• |−→u + −→v | ≤ |−→u | + |−→v |
Dấu bằng xảy ra khi −→u , −→v cùng hướng.
• |−→u .−→v | ≤ |−→u | |−→v |
Dấu bằng xảy ra khi −→u , −→v cùng phương.
• −→u 2
≥ 0
Dấu bằng xảy ra khi −→u =
−→
0
2) Với ba điểm A, B, C bất kì ta luôn có AB + BC ≥ AC
Dấu bằng xảy ra khi A, B, C thẳng hàng theo thứ tự trên.
3) Cho điểm M nằm ngoài đường thẳng d cho trước. Khi đó, độ dài
đường vuông góc hạ từ M xuống đường thẳng d luôn ngắn hơn mọi
đường xiên kẻ từ M xuống d.
b.Các ví dụ áp dụng.
Ví dụ 2.13. Tìm giá trị nhỏ nhất của hàm số
f(x) = x2 − x + 1 + x2 −
√
3x + 1, với x ∈ R
Lời giải.
Hàm số đã cho xác định với mọi x ∈ R
Ta có
f(x) = (x −
1
2
)2 + (
√
3
2
)2 + (x −
√
3
2
)2 + (
1
2
)2
Xét hệ trục Oxy, với: A(1
2;
√
3
2 ), B(
√
3
2 ; −1
2) và C(x; 0), x ∈ R
Khi đó f(x) = CA + CB.
44
Chương 2. Giá trị lớn nhất, giá trị nhỏ nhất
Lại có
CA + CB ≥ AB = (
√
3
2
−
1
2
)2 + (−
1
2
−
√
3
2
)2 =
√
2
Như vậy f(x) ≥
√
2, ∀x ∈ R
Vậy min f(x) =
√
2 ⇔ x =
√
3 − 1
Nhận xét: Ví dụ trên xuất phát từ một bài toán đại số đã được quy
về bài toán hình học: tìm trên trục Ox một điểm C sao cho tổng khoảng
cách CA + CB là nhỏ nhất.Từ đây ta suy ra:
Nếu A, B nằm về hai phía của trục Ox, thì điểm C là giao điểm của
đường thẳng AB với trục Ox.
Nếu A, B nằm cùng phía với trục Ox, thì điểm C là giao điểm của
đường thẳng AB với trục Ox, trong đó B là điểm đối xứng của B qua
Ox.
Ví dụ 2.14. Tìm giá trị lớn nhất, giá trị nhỏ nhất của hàm số
f(x) = cos2 x − 2 cos x + 3 + cos2 x + 4 cos x + 6
Lời giải.
Ta có
f(x) = (1 − cos x)2 + 2 + (2 + cos x)2 + 2
45
Chương 2. Giá trị lớn nhất, giá trị nhỏ nhất
Trong mặt phẳng Oxy xét điểm M(
√
2; 1 − cos x) và N(2
√
2; 3).
Khi đó
f(x) = (
√
2)2 + (1 − cos x)2 + (2
√
2 −
√
2)2 + [3 − (1 − cos x)]2
= OM + MN
Do 0 ≤ 1−cos x ≤ 2 nên M chạy trên đoạn AB với A(
√
2; 0) và B(
√
2; 2).
Từ bất đẳng thức OM + MN ≥ ON, ta suy ra
f(x) ≥ (2
√
2)2 + 32
⇔ f(x) ≥
√
17
Dấu bằng xảy ra khi và chỉ khi M ≡ C, với C là giao điểm của ON và
AB. Khi đó:
cos x = −
1
2
⇔ x = ±
2π
3
+ k2π, k ∈ Z
Mặt khác:
OM + MN ≤ max(OA + AN; OB + BN) = max(
√
2 +
√
11;
√
6 +
√
3)
Do đó f(x) ≤
√
2 +
√
11
Dấu bằng xảy ra khi và chỉ khi M ≡ A ⇔ cos x = 1
Vậy
min f(x) =
√
17 ⇔ x = ±
2π
3
+ k2π, k ∈ Z
max f(x) =
√
2 +
√
11 ⇔ x = 2kπ, k ∈ Z
46
Chương 2. Giá trị lớn nhất, giá trị nhỏ nhất
Ví dụ 2.15. Cho các số thực dương a, b, c thỏa mãn ab + bc + ca = abc.
Tìm giá trị nhỏ nhất của biểu thức
P =
√
b2 + 2a2
ab
+
√
c2 + 2b2
bc
+
√
a2 + 2c2
ca
Lời giải.
Biến đổi P về dạng
P =
1
a2
+
2
b2
+
1
b2
+
2
c2
+
1
c2
+
2
a2
Đặt −→u (1
a;
√
2
b ), −→v (1
b ;
√
2
c ), −→w (1
c ;
√
2
a ).
Khi đó
−→u + −→v + −→w = (
1
a
+
1
b
+
1
c
;
√
2
a
+
√
2
b
+
√
2
c
)
Mặt khác, từ giả thiết ab + bc + ca = abc suy ra 1
a + 1
b + 1
c = 1, nên
−→u + −→v + −→w = (1;
√
2) ⇒ |−→u + −→v + −→w | =
√
1 + 2 =
√
3
Nhận thấy
P = |−→u | + |−→v | + |−→w | ≥ |−→u + −→v + −→w |
Suy ra P ≥
√
3. Dấu bằng xảy ra khi a = b = c = 3
Vậy min P =
√
3 ⇔ a = b = c = 3.
c. Bài tập tham khảo.
Bài tập 2.16. (Đề thi học sinh giỏi Toán Toàn Quốc năm 1998)
Tìm giá trị giá trị nhỏ nhất của biểu thức
F(x; y) = (x + 1)2 + (y − 1)2+ (x − 1)2 + (y + 1)2+ (x + 2)2 + (y + 2)2
Trong đó x, y là các số thực.
Gợi ý: Trong hệ trục Oxy lấy các điểm A(0; −1), B(0; 3). Bài toán
trở thành: Tìm giá trị nhỏ nhất của tổng khoảng cách từ điểm M(x; y)
trên đường thẳng y = 2x − 2 đến hai điểm A và B, nghĩa là tìm giá trị
nhỏ nhất của
F(x; y) = F(M) = MA + MB
47
Chương 2. Giá trị lớn nhất, giá trị nhỏ nhất
Giải bài toán này bằng hình học:
Lấy điểm B đối xứng với điểm B qua đường thẳng y = 2x − 2, được
B (4; 1). Khi đó điểm M phải tìm là giao điểm của hai đường thẳng AB
và y = 2x − 2.
Đường thẳng AB có phương trình y = 1
2x − 1.
Suy ra M(2
3; −2
3).
Vậy min F(x; y) = 2
√
5.
Bài tập 2.17. Cho x, y là các số thực thỏa mãn điều kiện



−x + 2y − 8 ≤ 0
x + y + 2 ≥ 0
y − 2x − 4 ≥ 0
(∗)
Tìm giá trị lớn nhất và nhỏ nhất của biểu thức P = x2
+ y2
.
Lời giải.
Gọi M(x; y) là điểm trên mặt phẳng có tọa độ thỏa mãn hệ (∗).
Ta thấy tập hợp các điểm M là tam giác ABC (kể cả ba cạnh), trong
đó A(−4; 2), B(0; 4), C(−2; 0).
Ta có x2
+ y2
= OM2
.
48
Chương 2. Giá trị lớn nhất, giá trị nhỏ nhất
Do đó
max P = max
(x;y)∈ ABC
(x2
+ y2
)
= max
M∈ ABC
OM2
= max OA2
; OB2
; OC2
= max {20; 16; 4} = 20 = OA2
min P = min
M∈ ABC
OM2
= OH2
(ở đây ta kẻ OH ⊥ BC)
Lại có OH là khoảng cách từ O đến đường thẳng y − 2x − 4 = 0, do đó
OH =
|−4|
√
1 + 4
=
4
√
5
Vậy
max P = 20 ⇔
x = −4
y = 2
min P =
16
5
⇔
x = 8
5
x = 4
5
Bài tập 2.18. Cho bốn số thực x, y, z, t thỏa mãn điều kiện x + y =
6; z2
+ t2
= 1. Tìm giá trị nhỏ nhất của biểu thức
P = x2
+ y2
− 2xz − 2yt
Gợi ý: Xét hệ trục tọa độ Oxy. Khi đó điểm M(x; y) với x + y = 6
nằm trên đường thẳng x + y = 6, còn điểm N(z; t) với z2
+ t2
= 1 nằm
49
Chương 2. Giá trị lớn nhất, giá trị nhỏ nhất
trên đường tròn đơn vị x2
+ y2
= 1 tâm O, bán kính bằng 1.
Viết P dưới dạng
P = (x − z)2
+ (y − t)2 − (z2
+ t2
)
= (x − z)2
+ (y − t)2 − 1
= MN2
− 1
Đáp số:
min P = 18 − 6
√
2 ⇔
x = y =
√
2
2
z = t = 3
Bài tập 2.19. Cho x, y là các số thực không âm thỏa mãn điều kiện
2x + y ≥ 2
x + 3y ≤ 9
(∗)
Tìm giá trị lớn nhất và nhỏ nhất của biểu thức P = x2
+ y2
− 4x − 8y.
Gợi ý: Xét hệ trục Oxy với điểm M(x; y) có tọa độ thỏa mãn hệ
điều kiện của giả thiết. Khi đó, tập hợp điểm M là tứ giác ABCD với
A(1; 0), B(0; 2), C(0; 3), D(9; 0).
Biểu thức P có dạng P = (x − 2)2
+ (y − 4)2
− 20
Gọi I(2; 4), khi đó P = MI2
− 20
Đáp số:
max P = 45 ⇔ x = 9; y = 0
min P =
35
2
⇔ x =
3
2
; y =
5
2
50
Chương 2. Giá trị lớn nhất, giá trị nhỏ nhất
Bài tập 2.20. Cho tam giác ABC thay đổi luôn nội tiếp đường tròn
bán kính R cho trước. Tìm giá trị lớn nhất của biểu thức
M = a2
+ b2
+ c2
trong đó a, b, c lần lượt là độ dài ba cạnh tam giác đã cho.
Lời giải.
Ki hiệu O là tâm đường tròn ngoại tiếp tam giác ABC, G là trọng tâm
tam giác.Khi đó, từ đẳng thức:
3
−→
OG =
−→
OA +
−−→
OB +
−→
OC
ta suy ra
(3
−→
OG)2
= (
−→
OA +
−−→
OB +
−→
OC)2
=
−→
OA2
+
−−→
OB2
+
−→
OC2
+ 2(
−→
OA
−−→
OB +
−−→
OB
−→
OC +
−→
OC
−→
OA)
= 9R2
− (
−→
OA −
−−→
OB)2
+ (
−−→
OB −
−→
OC)2
+ (
−→
OC −
−→
OA)2
= 9R2
− (
−→
BA2
+
−−→
CB2
+
−→
AC2
)
= 9R2
− (a2
+ b2
+ c2
)
Do đó M = a2
+ b2
+ c2
≤ 9R2
Dấu bằng xảy ra khi 9
−→
OG2
= 0 ⇔ O ≡ G
Vậy max M = 9R2
2.2.5 Phương pháp sử dụng bất đẳng thức
Phương pháp sử dụng bất đẳng thức được áp dụng rộng rãi để tìm
giá trị lớn nhất, giá trị nhỏ nhất. Trong phạm vi toán sơ cấp, luận văn
chỉ nêu ra một số phương pháp sử dụng các bất đẳng thức quen thuộc
như bất đẳng thức Cauchy, bất đẳng thức giữa các giá trị trung bình và
một số bất đẳng thức có cách chứng minh đơn giản.
a. Cơ sở lý thuyết.
Phương pháp này còn có thể gọi là phương pháp sử dụng định nghĩa,
vì ta sử dụng trực tiếp định nghĩa giá trị lớn nhất, giá trị nhỏ nhất để
giải quyết bài toán.
Quy tắc:
51
Chương 2. Giá trị lớn nhất, giá trị nhỏ nhất
- Sử dụng các bất đẳng thức quen thuộc để đánh giá biểu thức cần tìm
giá trị lớn nhất, giá trị nhỏ nhất với các hằng số M và n;
- Chỉ ra ít nhất một bộ phận giá trị của các biến để dấu đẳng thức xảy
ra;
- Kết luận về giá trị lớn nhất, giá trị nhỏ nhất cần tìm.
Một số bất đẳng thức quen thuộc:
1. Bất đẳng thức Cauchy: Với hai bộ n số thực (a1, a2, ..., an) và
(b1, b2, ..., bn) ta luôn có
(a1b1 + a2b2 + ... + anbn)2
≤ (a2
1 + a2
2 + ... + a2
n)(b2
1 + b2
2 + ... + b2
n)
Dấu đẳng thức xảy ra khi a1
b1
= a2
b2
= ... = an
bn
. Với quy ước: nếu
ai = 0 thì bi = 0.
2. Bất đẳng thức giữa trung bình cộng và trung bình nhân: Giả sử
x1, x2, ..., xn là các số không âm. Khi đó
x1 + x2 + ... + xn
n
≥ n
√
x1x2...xn
Dấu đẳng thức xảy ra khi x1 = x2 = ... = xn
3. Bất đẳng thức Bernoulli: Với mọi số thực α > 1 ta luôn có
xα
+ α − 1 ≥ αx, x ∈ R+
Dấu đẳng thức xảy ra khi x = 1.
Chứng minh.
Xét hàm số f(x) = xα
+ α − 1 − αx, x > 0
Ta có f(1) = 0 và f (x) = αxα−1
− α = α(xα−1
− 1).
Suy ra f (x) = 0 ⇔ x = 1 và x = 1 là cực tiểu duy nhất của f(x)
trên R+
.
Vậy f(x) ≥ f(1) = 0.
4. Sử dụng tính chất của giá trị lớn nhất, giá trị nhỏ nhất của hàm số
được nêu trong định lý (2.2) và (2.3).
Cho hàm số f(x) có đạo hàm cấp một liên tục trên [a; b], có đạo
52
Chương 2. Giá trị lớn nhất, giá trị nhỏ nhất
hàm cấp hai tại mọi điểm x ∈ (a; b).Khi đó :
• Nếu f (x) ≥ 0 với mọi x ∈ (a; b), thì
f(x) ≥ f(y) + f (y)(x − y), ∀x, y ∈ [a; b]
• Nếu f (x) ≤ 0 với mọi x ∈ (a; b), thì
f(x) ≤ f(y) + f (y)(x − y), ∀x, y ∈ [a; b]
b. Các ví dụ áp dụng.
Ví dụ 2.16. Cho a, b, c là các số thực dương thỏa mãn a + b + c = 1.
Tìm giá trị nhỏ nhất của biểu thức
P =
a2
b + c
+
b2
c + a
+
c2
a + b
Lời giải.
Nhận thấy
1 = (a+b+c)2
= (
a
√
b + c
√
b + c+
b
√
c + a
√
c + a+
c
√
a + b
√
a + b)2
(∗)
Áp dụng bất đẳng thức Cauchy cho hai bộ số
(
a
√
b + c
;
b
√
c + a
;
c
√
a + b
) và (
√
b + c;
√
c + a;
√
a + b), ta có
(∗) ≤ (
a2
b + c
+
b2
c + a
+
c2
a + b
)(b + c + c + a + a + b)
⇔ 1 ≤ 2P
⇔ P ≥
1
2
Dấu đẳng thức xảy ra khi a = b = c = 1
3
Vậy min P = 1
2 ⇔ a = b = c = 1
3
Ví dụ 2.17. Cho a, b, c là độ dài ba cạnh một tam giác. Tìm giá trị nhỏ
nhất của biểu thức
P =
a
b + c
+
b
c + a
+
c
a + b
53
Chương 2. Giá trị lớn nhất, giá trị nhỏ nhất
Lời giải.
Nhận thấy
P + 3 =
a
b + c
+ 1 +
b
c + a
+ 1 +
c
a + b
+ 1
=
a + b + c
b + c
+
b + c + a
c + a
+
c + a + b
a + b
= (a + b + c)(
1
b + c
+
1
c + a
+
1
a + b
)
=
1
2
[(a + b) + (b + c) + (c + a)] (
1
b + c
+
1
c + a
+
1
a + b
)
Áp dụng bất đẳng thức Cauchy cho hai bộ số
(a + b), (b + c), (c + a)và
1
b + c
,
1
c + a
,
1
a + b
, ta có
[(a + b) + (b + c) + (c + a)] (
1
b + c
+
1
c + a
+
1
a + b
) ≥ 9
Dấu đẳng thức xảy ra khi a = b = c
Suy ra
P + 3 ≥
9
2
⇔ P ≥
3
2
Vậy minP = 3
2 ⇔ a = b = c (hay tam giác đã cho là tam giác đều).
Ví dụ 2.18. Cho x, y là các số thực thỏa mãn 2x2
+ 3y2
≤ 5. Tìm giá
trị lớn nhất, giá trị nhỏ nhất của biểu thức
A = 2x + 3y
Lời giải.
Nhận thấy
A2
= (2x + 3y)2
=
√
2(
√
2x) +
√
3(
√
3y)
2
Áp dụng bất đẳng thức Cauchy ta có
√
2(
√
2x) +
√
3(
√
3y)
2
≤ (2 + 3)(2x2
+ 3y2
)
⇔ A2
≤ 25
⇔ − 5 ≤ A ≤ 5
54
Chương 2. Giá trị lớn nhất, giá trị nhỏ nhất
Vậy
min A = −5 ⇔ x = y = −1
max A = 5 ⇔ x = y = 1
Ví dụ 2.19. (Đề thi học sinh giỏi Toán Quốc gia năm 1994)
Xét các biến số thực x, y, u, v thỏa mãn ba điều kiện:



2x2
+ 3y2
= 10
3u2
+ 8v2
= 6
4xv + 3yu ≥ 2
√
15
Tìm giá trị lớn nhất và giá trị nhỏ nhất của biểu thức S = x + y + u.
Lời giải.
Áp dụng bất đẳng thức Cauchy ta có
4xv + 3yu ≤ |4xv + 3yu| ≤ (2x2 + 3y2)(8v2 + 3u2) ≤ 2
√
15
Dấu đẳng thức xảy ra khi và chỉ khi
√
2x
2
√
2v
=
√
3y
√
3u
⇔
x
v
=
y
u
Hay xv và yu phải cùng dấu dương.
Lại có, theo giả thiết 4xv + 3yu ≥ 2
√
15
Từ đó suy ra 4xv + 3yu = 2
√
15
Theo tính chất của tỉ lệ thức và giả thiết ta có
2x2
8v2
=
3y2
3u2
=
x2
+ 3y2
8v2 + 3u2
=
10
6
Suy ra x = 2
√
5√
3
v và y =
√
5√
3
u
Khi đó
S = x + y + u =
2
√
5
√
3
v + (
√
5
√
3
+ 1)u
Áp dụng bất đẳng thức Cauchy ta được
|S| =
2
√
5
√
3
v + (
√
5
√
3
+ 1)u
≤ (
5
6
+
8 + 2
√
15
9
)(8v2 + 3u2) =
31 + 4
√
15
3
55
Chương 2. Giá trị lớn nhất, giá trị nhỏ nhất
Như vậy
−
31 + 4
√
15
3
≤ S ≤
31 + 4
√
15
3
Ta thấy:
S1 = 31+4
√
15
3 tại u1 = 16+4
√
15
23+2
√
15
; v1 = 45
92+8
√
15
và x1 = 2
√
5√
3
v1; y1 =
√
5√
3
u1.
S2 = − 31+4
√
15
3 tại u2 = −u1; v2 = −v1 và x2 = 2
√
5√
3
v2; y2 =
√
5√
3
u2.
Kết luận
max S =
31 + 4
√
15
3
min S = −
31 + 4
√
15
3
Ví dụ 2.20. Cho a, b, c là các số thực dương thỏa mãn a + b + c = 2012.
Tìm giá trị nhỏ nhất của biểu thức
M =
a3
(a + b)(b + c)
+
b3
(b + c)(c + a)
+
c3
(c + a)(a + b)
Lời giải.
Áp dụng bất đẳng thức giữa trung bình cộng và trung bình nhân, ta có:
8a3
(a + b)(b + c)
+ (a + b) + (b + c) ≥ 6a
8b3
(b + c)(c + a)
+ (b + c) + (c + a) ≥ 6b
8c3
(c + a)(a + b)
+ (c + a) + (a + b) ≥ 6c
Cộng các bất dẳng thức trên vế theo vế ta được
8M ≥ 2(a + b + c) ⇔ M ≥
1
4
(a + b + c)
Vậy min M = 503 ⇔ a = b = c = 2012
3 .
56
Chương 2. Giá trị lớn nhất, giá trị nhỏ nhất
Ví dụ 2.21. Cho a, b, c > 1. Tìm giá trị nhỏ nhất của biểu thức
M =
4a2
a − 1
+
5b2
b − 1
+
3c2
c − 1
Lời giải.
Ta có
4a2
a − 1
= 4(a + 1) +
4
a − 1
=
4
a − 1
+ 4(a − 1) + 8
Áp dụng bất đẳng thức giữa trung bình cộng và trung bình nhân cho
hai số dương 4
a−1 và 4(a − 1) ta được:
4
a − 1
+ 4(a − 1) ≥ 2
4
a − 1
.4(a − 1) = 8
Dấu đẳng thức xảy ra khi 4
a−1 = 4(a − 1) ⇔ a = 2
Suy ra
4a2
a − 1
≥ 16
Chứng minh tương tự:
5b2
b − 1
≥ 20
3c2
c − 1
≥ 12
Cộng ba bất đẳng thức trên vế theo vế ta được M ≥ 48
Vậy min M = 48 ⇔ a = b = c = 2
Ví dụ 2.22. Cho a, b, c > 0 thỏa mãn a+b+c = 1. Tìm giá trị lớn nhất
của biểu thức
M = a
2
3 + b
2
3 + c
2
3
Lời giải.
Áp dụng bất đẳng thức Bernoulli cho trường hợp x = (3t)
2
3 , ∀t > 0 và
α = 3
2, ta có
(3t)
2
3
2
3
+
1
2
≥
3
2
(3t)
2
3
⇔ 3t +
1
2
≥
3
2
.3
2
3 .t
2
3 (∗)
57
Chương 2. Giá trị lớn nhất, giá trị nhỏ nhất
Dấu đẳng thức xảy ra khi t = 1
3
Áp dụng bất đẳng thức (∗) cho các số a, b, c ta có:
3a +
1
2
≥
3
2
.3
2
3 .a
2
3
3b +
1
2
≥
3
2
.3
2
3 .b
2
3
3c +
1
2
≥
3
2
.3
2
3 .c
2
3
Cộng các bất đẳng thức trên vế theo vế:
3 +
3
2
≥
3
2
.3
2
3 .M
Suy ra
M ≤
3
3
√
9
Vậy max M = 3
3
√
9
⇔ a = b = c = 1
3
Ví dụ 2.23.
Cho



x ≥ 4
x + y ≥ 7
x + y + z ≥ 8
(∗). Tìm min x2
+ y2
+ z2
Lời giải.
Xét hàm số f(t) = t2
, t ∈ R có f (t) = 2 > 0, ∀t ∈ R
Theo định lý (2.2) ta có
f(t) ≥ f(t0) + f (t0)(t − t0); ∀t, t0 ∈ R
Áp dụng cho trường hợp t = x, y, z và t0 = 4, 3, 1, ta đươc:
x2
≥ 16 + 8(x − 4)
y2
≥ 9 + 6(y − 3)
z2
≥ 1 + 2(z − 1)
Cộng các bất đẳng thức trên vế theo vế
x2
+ y2
+ z2
≥ 26 + 2(x + y + z − 8) + 4(x + y − 7) + 2(x − 4)
58
Chương 2. Giá trị lớn nhất, giá trị nhỏ nhất
Từ hệ điều kiện (∗) ta suy ra x2
+ y2
+ z2
≥ 26.
Dấu đẳng thức xảy ra khi x = 4, y = 3, z = 1.
Vậy
min x2
+ y2
+ z2
= 26 ⇔ x = 4, y = 3, z = 1
Ví dụ 2.24. Cho x, y, z thỏa mãn 0 ≤ x, y, z ≤ 2; x + +z = 3. Tìm giá
trị lớn nhất của biểu thức
M = (1 + x2
)x
(1 + y2
)y
(1 + z2
)z
Lời giải.
Không mất tính tổng quát, giả sử x ≥ y ≥ z. Từ giả thiết ta có



x ≤ 2
x + y ≤ 3 = 2 + 1
x + y + z = 3 = 2 + 1 + 0
Xét hàm số f(x) = x ln(1 + x2
) với x ≥ 0.
Ta thấy f (x) = ln(1 + x2
) + 2x2
1+x2 ; f (x) = 2x
1+x2 + 4x
(1+x2)2 > 0, ∀x ≥ 0
Áp dụng định lý (2.2) ta suy ra f(2) ≥ f(x) + f (x)(2 − x)
Do x ≤ 2 nên f (x)(2 − x) ≥ 0, ∀0 ≤ x ≤ 2
Suy ra f(x) ≤ f(2) ⇔ x ln(1 + x2
) ≤ 2 ln 5
Dấu đẳng thức xảy ra khi x = 2 Tương tự ta có
y ln(1 + y2
) ≤ 1 ln 2
z ln(1 + z2
) ≤ 0 ln 1
Cộng các bất đẳng thức trên vế theo vế ta được
x ln(1 + x2
) + y ln(1 + y2
) + z ln(1 + z2
) ≤ 2 ln 5 + 1 ln 2 + 0 ln 1
⇔ ln (1 + x2
) + (1 + y2
) + (1 + z2
) ≤ ln 50
⇔(1 + x2
) + (1 + y2
) + (1 + z2
) ≤ 50
Vậy max M = 50 khi x = 2, y = 1, z = 0.
Ví dụ 2.25. Cho tam giác ABC không nhọn. Tìm giá trị lớn nhất của
biểu thức
M = sin A + sin B + sin C
59
Chương 2. Giá trị lớn nhất, giá trị nhỏ nhất
Lời giải.
Không mất tính tổng quát ta giả sử A ≥ B ≥ C
Từ đó suy ra
A ≥ π
2
B + C ≤ π
2
Xét hàm số f(x) = sin x, x ∈ (0; π) có f (x) = − sin x < 0, ∀x ∈ (0; π).
Áp dụng định lý (2.3) với x = A, B, C và x0 = π
2 , π
4 , π
4 ta được
sin A ≤ sin
π
2
+ cos
π
2
(A −
π
2
)
sin B ≤ sin
π
4
+ cos
π
4
(B −
π
4
)
sin C ≤ sin
π
4
+ cos
π
4
(C −
π
4
)
Cộng các bất đẳng thức trên vế theo vế
sin A + sin B + sin C ≤ sin
π
2
+ sin
π
4
+ sin
π
4
+ cos
π
4
(B + C) −
π
2
⇔ sin A + sin B + sin C ≤ sin
π
2
+ sin
π
4
+ sin
π
4
⇔M ≤ 1 +
√
2
Dấu đẳng thức xảy ra khi A = π
2 , B = π
4 , C = π
4
Vậy max M = 1 +
√
2 khi ABC là tam giác vuông cân.
Nhận xét: Các ví dụ (2.23), (2.24), (2.25) đã vận dụng bất đẳng thức
Karamata. Tuy nhiên bất đẳng thức này không được giới thiệu trong
chương trình phổ thông vì vậy ta đã chứng minh những bài toán này
hoàn toàn bằng phương pháp sơ cấp.
c. Bài tập tham khảo.
Bài tập 2.21. Cho x, y, z, t là các số thực thỏa mãn
x + y + z + t = 0
x2
+ y2
+ z2
+ t2
= 1
Tìm giá trị lớn nhất, giá trị nhỏ nhất của biểu thức P = xy+yz+zt+tx.
Lời giải.
Áp dụng bất đẳng thức Cauchy ta có
(xy + yz + zt + tx)2
≤ (x2
+ y2
+ z2
+ t2
)2
⇔ P2
≤ 1
⇔ − 1 ≤ P ≤ 1 (1)
60
Chương 2. Giá trị lớn nhất, giá trị nhỏ nhất
Mặt khác
P = (x + z)(y + t) = −(x + z)2
⇒ P ≤ 0 (2)
Từ (1) và (2) suy ra: −1 ≤ P ≤ 0
Vậy min P = −1 khi
x = z = −1
2
y = t = 1
2
max P = 0 khi
x = y = −1
2
z = t = 1
2
Bài tập 2.22. Cho a, b, c là các số thực dương thỏa mãn a+b+c = 2012.
Tìm giá trị nhỏ nhất của biểu thức
M =
a2
a + b
+
b2
b + c
+
c2
c + a
Gợi ý:Áp dụng bất đẳng thức Cauchy ta suy ra
M [(a + b) + (b + c) + (c + a)] ≥ (a + b + c)2
Đáp số.
min M =
a + b + c
2
= 1006 ⇔ a = b = c =
2012
3
Bài tập 2.23. Cho −1 ≤ x ≤ 1. Tìm giá trị nhỏ nhất của hàm số
f(x) =
4
1 − x2 + 4
√
1 + x + 4
√
1 − x
Lời giải.
Áp dụng bất đẳng thức giữa trung bình cộng và trung bình nhân, ta có:
4
1 − x2 = 4
√
1 − x 4
√
1 + x ≤
√
1 − x +
√
1 + x
2
4
√
1 + x = 4
√
1 + x
4
√
1 ≤
√
1 + x + 1
2
4
√
1 − x = 4
√
1 − x
4
√
1 ≤
√
1 − x + 1
2
Suy ra f(x) ≤ 1 +
√
1 + x +
√
1 − x
Lại có:
√
1 + x +
√
1 − x ≤
1 + x + 1
2
+
1 + x + 1
2
= 2
Vậy max
−1≤x≤1
f(x) = 3 ⇔ x = 0
61
Chương 2. Giá trị lớn nhất, giá trị nhỏ nhất
Bài tập 2.24. Cho x, y ≥ 0 và x + y = 1. Tìm min x
√
2
+ y
√
2
Gợi ý:Áp dụng bất đẳng thức Bernoulli ta suy ra
(2t)
√
2
≥
√
2(2t) −
√
2 + 1
Đáp số.
min x
√
2
+ y
√
2
= 21−
√
2
⇔ x = y =
1
2
Bài tập 2.25.
Cho a ≥ b ≥ c và



x ≥ a
x + y ≥ b
x + y + z ≥ c
Tìm giá trị lớn nhất của M = x2
+ y2
+ z2
Gợi ý: Áp dụng định lý (2.2) cho hàm f(t) = t2
Đáp số.
max M = a2
+ b2
+ c2
Bài tập 2.26. Cho x, y, y, z, t là các số thực dương thỏa mãn:



x ≤ 1
x + y ≤ 3
x + y + z ≤ 6
x + y + z + t ≤ 10
Tìm giá trị lớn nhất của
M =
1
x
+
1
y
+
1
z
+
1
t
.
Gợi ý: Áp dụng định lý (2.2) cho hàm f(t) = 1
t
Đáp số.
max M =
25
12
⇔ x = 1, y = 2, z = 3, t = 4
Bài tập 2.27. Cho tam giác ABC bất kỳ.Tìm giá trị lớn nhất của các
biểu thức
P =
sin A
2
2
√
2
+
sin B
2
2
√
3
+
sin C
2
√
6 +
√
2
62
Chương 2. Giá trị lớn nhất, giá trị nhỏ nhất
Gợi ý: Nhận thấy
4P =
sin A
2√
2
2
+
sin B
2√
3
2
+
sin C
2√
6+
√
2
2
=
sin A
2
cos π
4
+
sin B
2
cos π
6
+
sin C
2
cos π
12
(1)
Áp dụng định lý (2.3) cho hàm y = sin x ta có
sin x ≤ sin y + cos y(x − y), ∀x, y ∈ (0;
π
2
)
⇔
sin x
cos y
≤
sin y
cos y
+ (x − y), ∀x, y ∈ (0;
π
2
)
Vận dụng vào bài toán, ta được:
sin A
2
cos π
4
≤
sin π
4
cos π
4
+ (
A
2
−
π
4
)
sin B
2
cos π
6
≤
sin π
6
cos π
6
+ (
B
2
−
π
6
)
sin C
2
cos π
12
≤
sin π
12
cos π
12
+ (
C
2
−
π
12
) (2)
Từ (1) và (2) suy ra
4P ≤ 1 +
√
3
3
+ (2 −
√
3)
Dấu đẳng thức xảy xảy ra khi A = π
2 , B = π
3 , C = π
6
Đáp số.
max P = 9−2
√
3
12 ⇔ A = π
2 , B = π
3 , C = π
6
2.2.6 Một số bài tập vận dụng
Trên đây luận văn đã trình bày một số phương pháp cơ bản để tìm
giá trị lớn nhất, giá trị nhỏ nhất; các ví dụ và bài tập ứng dụng của từng
phương pháp. Tuy nhiên trong thực tế không phải lúc nào một bài toán
cũng được giải quyết bằng một phương pháp riêng lẻ, có những bài toán
63
Chương 2. Giá trị lớn nhất, giá trị nhỏ nhất
cần sự phối hợp nhiều phương pháp. Vì vậy, phần tiếp theo của luận văn
sẽ trình bày một số bài tập được giải bằng cách phối hợp nhiều phương
pháp.
Bài tập 2.28. (Đề thi học sinh giỏi Toán Quốc gia năm 2004)
Xét các số thực dương x, y, z thỏa mãn điều kiện (x + y + z)3
= 32xyz.
Hãy tìm giá trị lớn nhất, giá trị nhỏ nhất của biểu thức
P =
x4
+ y4
+ z4
(x + y + z)4
Lời giải.
Nhận thấy với n là số thực dương tùy ý ta luôn có
P(x, y, z) = P(nx + ny + nz)
Nếu x, y, z thỏa mãn giả thiết thì nx, ny, nz cũng thỏa mãn các điều kiện
đó.
Vì vậy không mất tính tổng quát, giả sử x + y + z = 4. Khi đó, kết hợp
với điều kiện đề bài, ta được xyz = 2.
Bài toán quy về việc tìm giá trị lớn nhất, giá trị nhỏ nhất của biểu thức
P =
1
256
(x4
+ y4
+ z4
)
trong đó x, y, z là các số thực dương thỏa mãn x + y + z = 4 và xyz = 2.
Đặt Q = x4
+ y4
+ z4
và t = xy + yz + zx, ta có
Q = (x2
+ y2
+ z2
)2
− 2(x2
y2
+ y2
z2
+ z2
x2
)
= (42
− 2t)2
− 2(t2
− 2xyz(x + y + z))
= 2t2
− 64t + 44
+ 32
Q = 2(t2
− 32t + 144) (1)
Từ các điều kiện đối với x, y, z ta được
y + z = 4 − x và yz = 2
x (2)
Do đó t = x(4 − x) + 2
x (3)
Áp dụng bất đẳng thức giữa trung bình cộng và trung bình nhân cho
64
Chương 2. Giá trị lớn nhất, giá trị nhỏ nhất
hai số dương y, z, từ (2) ta được
(4 − x)2
≥
8
x
⇔ x3
− 8x2
+ 16x − 8 ≥ 0
⇔ (x − 2)(x2
− 6x + 4) ≥ 0
⇔ 3 −
√
5 ≤ x ≤ 2 (vì x ∈ (0; 4))
Xét hàm số t(x) = x(4 − x) + 2
x trên đoạn 3 −
√
5; 2 ta có
t (x) =
−2(x − 1)(x2
− x − 1)
x2
Từ việc xét dấu của t (x) trên 3 −
√
5; 2 ta được
5 ≤ t ≤
5
√
5 − 1
2
Vì hàm f(t) = t2
− 32t + 144 nghịch biến trong khoảng (0; 16) và
3 −
√
5; 2 ⊂ (0; 16)
Nên ta có
min f(t) = f(
5
√
5 − 1
2
) =
383 − 165
√
5
2
max f(t) = f(5) = 9
Kết hợp với (1) ta được min Q = 183 − 165
√
5; max Q = 18
Kết luận
min P =
383 − 165
√
5
256
⇔ x = 3 −
√
5, y = z =
1 +
√
5
2
max P =
9
128
⇔ x = 2, y = z = 1
Bài tập 2.29. (Đề thi tuyển sinh Đại học, Cao đẳng khối A năm 2003)
Cho x, y, z là các số thực dương và thỏa mãn điều kiện x + y + z ≤ 1.
Tìm giá trị nhỏ nhất của biểu thức
P = x2 +
1
x2
+ y2 +
1
y2
+ z2 +
1
z2
Lời giải.
65
Chương 2. Giá trị lớn nhất, giá trị nhỏ nhất
Xét các vectơ −→u = (x; 1
x), −→v = (y; 1
y ), −→w = (z; 1
z )
Ta có
−→u + −→v + −→w = (x + y + z;
1
x
+
1
y
+
1
z
)
Theo tính chất độ dài vectơ ta có |−→u | + |−→v | + |−→w | ≥ |−→u + −→v + −→w |
Do đó
P = x2 +
1
x2
+ y2 +
1
y2
+ z2 +
1
z2
≥ (x + y + z)2 + (
1
x
+
1
y
+
1
z
)2
Dấu đẳng thức xảy ra khi −→u , −→v , −→w là các vectơ cùng chiều.
⇔
−→u = k1
−→v , k1 > 0
−→v = k2
−→w , k2 > 0
Ta thấy
(x+y+z)2
+(
1
x
+
1
y
+
1
z
)2
= 81(x+y+z)2
+(
1
x
+
1
y
+
1
z
)2
−80(x+y+z)2
Áp dụng bất đẳng thức giữa trung bình cộng và trung bình nhân ta có
81(x + y + z)2
+ (
1
x
+
1
y
+
1
z
)2
≥ 18(x + y + z)(
1
x
+
1
y
+
1
z
)
Mặt khác
(x + y + z)(
1
x
+
1
y
+
1
z
) ≥ 9
Suy ra 81(x + y + z)2
+ (1
x + 1
y + 1
z )2
≥ 162
Theo giả thiết 0 < x + y + z ≤ 1 ⇒ 80(x + y + z)2
≤ 80
Từ đó suy ra P ≥
√
82
Dấu đẳng thức xảy ra khi x = y = z = 1
3
Vậy min P =
√
82 ⇔ x = y = z = 1
3.
Bài tập 2.30. (Đề thi tuyển sinh Đại học, Cao đẳng khối A năm 2011)
Cho x ≥ y, x ≥ z và x, y, z ∈ [1; 4]. Tìm giá trị nhỏ nhất của biểu thức
P =
x
2x + 3y
+
y
y + z
+
z
z + x
Lời giải.
66
Chương 2. Giá trị lớn nhất, giá trị nhỏ nhất
Viết biểu thức P dưới dạng
P =
1
2 + 3y
x
+
1
1 + z
y
+
1
1 + x
z
Trước hết ta chứng minh bất đẳng thức: Nếu a, b > 0 và ab ≥ 1, thì ta
có
1
1 + a
+
1
1 + b
≥
2
1 +
√
ab
(∗)
Dấu đẳng thức xảy ra khi và chỉ khi a = b hoặc ab = 1
Thật vậy
(∗) ⇔ (
1
1 + a
−
1
1 +
√
ab
) + (
1
1 + b
−
1
1 +
√
ab
) ≥ 0
⇔
√
ab − a
(1 + a)(1 +
√
ab)
+
√
ab − b
(1 + b)(1 +
√
ab)
≥ 0
⇔
(
√
a −
√
b)2
(
√
ab − 1)
(1 + a)(1 + b)(1 +
√
ab)
≥ 0 (∗∗)
Do a, b > 0; ab ≥ 1 nên (∗∗) đúng, suy ra (∗) đúng.
Áp dụng (∗) với a = z
y , b = x
z . Khi đó a, b > 0 và ab = x
y ≥ 1 (do x ≥ y),
nên ta có
1
1 + z
y
+
1
1 + x
z
≥
2
1 + x
y
Dấu đẳng thức xảy ra khi và chỉ khi
z
y = x
z
z
y .x
y = 1
⇔
√
yz = z
x = y
Từ đó suy ra
P ≥
1
2 + 3y
x
+
2
1 + x
y
Dấu đẳng thức xảy ra ⇔
√
yz = z
x = y
Đặt t = x
y . Do x ≥ y và x, y ∈ [1; 4], nên suy ra
1 ≤
x
y
≤ 4 ⇒ 1 ≤ t ≤ 2
Khi đó
P ≥
1
2 + 3
t2
+
2
1 + t
hay P ≥
t2
2t2 + 3
+
2
1 + t
67
Chương 2. Giá trị lớn nhất, giá trị nhỏ nhất
Xét hàm số
f(t) =
t2
2t2 + 3
+
2
1 + t
, 1 ≤ t ≤ 2
f (t) =
(3t − 6t2
) + (3t3
− 4t4
) − 9
(2t2 + 3)2(1 + t)2
Vì t ≥ 1 nên f (t) < 0, ∀t ∈ [1; 2]
Suy ra f(t) nghịch biến trong [1; 2].
Vậy min
1≤t≤2
f(t) = f(2) = 34
33
Từ đó suy ra P ≥ 34
33
Dấu đẳng thức xảy ra khi và chỉ khi x = 4y, z = 2y
Lại có x, y, z ∈ [1; 4] nên suy ra x = 4, y = 1, z = 2
Kết luận min P = 34
33 ⇔ x = 4, y = 1, z = 2.
68
Kết luận
Sau thời gian học tập tại Khoa Toán - Cơ - Tin học, Trường Đại
học Khoa học Tự nhiên, Đại học Quốc gia Hà Nội. Được các thầy cô
trực tiếp giảng dạy và hướng dẫn đặc biệt là PGS. TS Nguyễn Đình
Sang, tôi đã hoàn thành luận văn với đề tài ” Phương pháp giải bài toán
cực trị và ứng dụng”. Luận văn đã đạt được một số kết quả sau:
1. Luận văn đã trình bày một cách hệ thống các phương pháp giải
bài toán cực trị địa phương và bài toán cực trị tuyệt đối, đưa ra một số
bài toán tổng quát với những cách giải hiệu quả, giúp kích thích tư duy
tìm tòi, sáng tạo của học sinh trong việc học tập bộ môn Toán.
2. Luận văn khai thác được một số ứng dụng cơ bản của bài toán
cực trị với nhiều ví dụ minh họa áp dụng các phương pháp đa dạng kèm
theo các bài tập tham khảo được trích từ các kì thi học sinh giỏi Toán
Quốc gia, kì thi tuyển sinh và Đại học và Cao đẳng các năm. Vì vậy bản
luận văn có thể là tài liệu tham khảo nhằm nâng cao và mở rộng kiến
thức cho học sinh bậc trung học phổ thông.
3. Một số hướng phát triển của đề tài:
• Phương pháp tìm giá trị lớn nhất, giá trị nhỏ nhất sử dụng bất đẳng
thức có thể mở rộng với nhiều bất đẳng thức khác như: bất đẳng thức
giữa trung bình cộng với trung bình nhân suy rộng, bất đẳng thức hàm,
bất đẳng thức trong dãy số, bất đẳng thức tích phân, ...
• Ứng dụng bài toán cực trị tuyệt đối vào việc giải các phương trình
và bất phương trình.
Tài liệu tham khảo
Tiếng Việt
1. Phan Huy Khải (2011),Các phương pháp giải toán Giá trị lớn nhất,
giá trị nhỏ nhất, NXB Đại học Sư Phạm.
2. Trần Đức Long, Nguyễn Đình Sang, Hoàng Quốc Toàn (2001),Giáo
trình Giải tích tập I - Phép tính vi phân của hàm một biến, NXB
Đại học Quốc Gia Hà Nội.
3. Trần Đức Long, Nguyễn Đình Sang, Nguyễn Viết Trần Tiến, Hoàng
Quốc Toàn (2001),Phép tính vi phân, NXB Đại học Quốc gia Hà
Nội.
4. Nguyễn Vũ Lương, Phạm Văn Hùng, Nguyễn Ngọc Thắng (2006),Các
bài giảng về bất đẳng thức Côsi, NXB Đại học Quốc Gia Hà Nội.
5. Nguyễn Văn Mậu (2006), Bất đẳng thức định lý và áp dụng, NXB
Giáo Dục.
6. Nguyễn Văn Mậu, Nguyễn Văn Tiến (2010),Một số chuyên đề Đại số
bồi dưỡng học sinh giỏi Trung học phổ thông, NXB Giáo Dục .
7. Trần Phương (2010), Tuyển tập các chuyên đề hàm số, NXB Hà Nội.
8. Đoàn Quỳnh, Trần Nam Dũng, Nguyến Vũ Lương, Đặng Hùng Thắng
(2010),Tài liệu chuyên toán và giải tích 12, NXB Giáo Dục.
9. Tủ sách Toán học và Tuổi trẻ (2007),Các bài thi Olympic Toán Trung
học phổ thông Việt Nam (1990 - 2006), NXB Giáo Dục.
Tiếng Nga
10. I. I. Liasko, A. K. Bolartruk, G.P. Golobax (1977),Matematitreski,
Vưsa-skola, Kieb.

More Related Content

What's hot

Hdc cttoan gdtx_tn_k10
Hdc cttoan gdtx_tn_k10Hdc cttoan gdtx_tn_k10
Hdc cttoan gdtx_tn_k10
ntquangbs
 
Thi thử toán nghèn ht 2012 lần 1 k d
Thi thử toán nghèn ht 2012 lần 1 k dThi thử toán nghèn ht 2012 lần 1 k d
Thi thử toán nghèn ht 2012 lần 1 k d
Thế Giới Tinh Hoa
 
[Toanmath.com] chuyên đề giới hạn của dãy số - nguyễn quốc tuấn
[Toanmath.com]   chuyên đề giới hạn của dãy số - nguyễn quốc tuấn[Toanmath.com]   chuyên đề giới hạn của dãy số - nguyễn quốc tuấn
[Toanmath.com] chuyên đề giới hạn của dãy số - nguyễn quốc tuấn
Quý Hoàng
 
[Math educare] bai giang phuong trinh vi phan-trinh duc tai
[Math educare] bai giang phuong trinh vi phan-trinh duc tai[Math educare] bai giang phuong trinh vi phan-trinh duc tai
[Math educare] bai giang phuong trinh vi phan-trinh duc tai
Nguyen Vietnam
 

What's hot (18)

Hdc cttoan gdtx_tn_k10
Hdc cttoan gdtx_tn_k10Hdc cttoan gdtx_tn_k10
Hdc cttoan gdtx_tn_k10
 
Luận văn: Phương pháp giải phương trình chứa ẩn dưới dấu căn
Luận văn: Phương pháp giải phương trình chứa ẩn dưới dấu cănLuận văn: Phương pháp giải phương trình chứa ẩn dưới dấu căn
Luận văn: Phương pháp giải phương trình chứa ẩn dưới dấu căn
 
Luận văn: Bài toán nội suy sinh bởi toán tử khả nghịch, HAY, 9đ
Luận văn: Bài toán nội suy sinh bởi toán tử khả nghịch, HAY, 9đLuận văn: Bài toán nội suy sinh bởi toán tử khả nghịch, HAY, 9đ
Luận văn: Bài toán nội suy sinh bởi toán tử khả nghịch, HAY, 9đ
 
Luận án: Giải bất đẳng thức biến phân trên tập điểm bất động, HAY
Luận án: Giải bất đẳng thức biến phân trên tập điểm bất động, HAYLuận án: Giải bất đẳng thức biến phân trên tập điểm bất động, HAY
Luận án: Giải bất đẳng thức biến phân trên tập điểm bất động, HAY
 
Luận văn: Giải một số phương trình tích phân kỳ dị, HAY, 9đ
Luận văn: Giải một số phương trình tích phân kỳ dị, HAY, 9đLuận văn: Giải một số phương trình tích phân kỳ dị, HAY, 9đ
Luận văn: Giải một số phương trình tích phân kỳ dị, HAY, 9đ
 
Luận văn: Sự hội tụ của các độ đo xác suất và ứng dụng, HOT, 9đ
Luận văn: Sự hội tụ của các độ đo xác suất và ứng dụng, HOT, 9đLuận văn: Sự hội tụ của các độ đo xác suất và ứng dụng, HOT, 9đ
Luận văn: Sự hội tụ của các độ đo xác suất và ứng dụng, HOT, 9đ
 
Đề tài: Thiết kế và phân tích thuật toán, HAY
Đề tài: Thiết kế và phân tích thuật toán, HAYĐề tài: Thiết kế và phân tích thuật toán, HAY
Đề tài: Thiết kế và phân tích thuật toán, HAY
 
Thi thử toán nghèn ht 2012 lần 1 k d
Thi thử toán nghèn ht 2012 lần 1 k dThi thử toán nghèn ht 2012 lần 1 k d
Thi thử toán nghèn ht 2012 lần 1 k d
 
Đề tài: Phương trình vi phân với toán tử khả nghịch phải, HOT, 9đ
Đề tài: Phương trình vi phân với toán tử khả nghịch phải, HOT, 9đĐề tài: Phương trình vi phân với toán tử khả nghịch phải, HOT, 9đ
Đề tài: Phương trình vi phân với toán tử khả nghịch phải, HOT, 9đ
 
[Toanmath.com] chuyên đề giới hạn của dãy số - nguyễn quốc tuấn
[Toanmath.com]   chuyên đề giới hạn của dãy số - nguyễn quốc tuấn[Toanmath.com]   chuyên đề giới hạn của dãy số - nguyễn quốc tuấn
[Toanmath.com] chuyên đề giới hạn của dãy số - nguyễn quốc tuấn
 
Luận văn: phương pháp nghiên cứu sự phân nhánh, HAY, 9đ
Luận văn: phương pháp nghiên cứu sự phân nhánh, HAY, 9đLuận văn: phương pháp nghiên cứu sự phân nhánh, HAY, 9đ
Luận văn: phương pháp nghiên cứu sự phân nhánh, HAY, 9đ
 
Luận văn: Phương pháp xây dựng độ đo và tích phân, HOT, 9đ
Luận văn: Phương pháp xây dựng độ đo và tích phân, HOT, 9đLuận văn: Phương pháp xây dựng độ đo và tích phân, HOT, 9đ
Luận văn: Phương pháp xây dựng độ đo và tích phân, HOT, 9đ
 
Luận văn: Mô hình xác suất trong khoa học máy tính, HAY, 9đ
Luận văn: Mô hình xác suất trong khoa học máy tính, HAY, 9đLuận văn: Mô hình xác suất trong khoa học máy tính, HAY, 9đ
Luận văn: Mô hình xác suất trong khoa học máy tính, HAY, 9đ
 
Đề tài: Lớp bất đẳng thức, bài toán cực trị với đa thức đối xứng, 9đ
Đề tài: Lớp bất đẳng thức, bài toán cực trị với đa thức đối xứng, 9đĐề tài: Lớp bất đẳng thức, bài toán cực trị với đa thức đối xứng, 9đ
Đề tài: Lớp bất đẳng thức, bài toán cực trị với đa thức đối xứng, 9đ
 
THPT hoang le kha 2013 -MVN
THPT hoang le kha 2013 -MVNTHPT hoang le kha 2013 -MVN
THPT hoang le kha 2013 -MVN
 
Luận văn: Phương trình tích phân kỳ dị với dịch chuyển và phản xạ
Luận văn: Phương trình tích phân kỳ dị với dịch chuyển và phản xạLuận văn: Phương trình tích phân kỳ dị với dịch chuyển và phản xạ
Luận văn: Phương trình tích phân kỳ dị với dịch chuyển và phản xạ
 
Luận văn thạc sĩ: Quy hoạch toàn phương, HAY, 9đ
Luận văn thạc sĩ: Quy hoạch toàn phương, HAY, 9đLuận văn thạc sĩ: Quy hoạch toàn phương, HAY, 9đ
Luận văn thạc sĩ: Quy hoạch toàn phương, HAY, 9đ
 
[Math educare] bai giang phuong trinh vi phan-trinh duc tai
[Math educare] bai giang phuong trinh vi phan-trinh duc tai[Math educare] bai giang phuong trinh vi phan-trinh duc tai
[Math educare] bai giang phuong trinh vi phan-trinh duc tai
 

Similar to Luận văn: Phương pháp giải bài toán cực trị, HAY

Cđ giải hpt không mẫu mực
Cđ giải hpt không mẫu mựcCđ giải hpt không mẫu mực
Cđ giải hpt không mẫu mực
Cảnh
 
[Vnmath.com] 13-ki-thuat-giai-phuong-trinh-ham
[Vnmath.com] 13-ki-thuat-giai-phuong-trinh-ham[Vnmath.com] 13-ki-thuat-giai-phuong-trinh-ham
[Vnmath.com] 13-ki-thuat-giai-phuong-trinh-ham
Duy Duy
 

Similar to Luận văn: Phương pháp giải bài toán cực trị, HAY (20)

Luận văn: Phép biến đổi phân tuyến tính, HAY, 9đ
Luận văn: Phép biến đổi phân tuyến tính, HAY, 9đLuận văn: Phép biến đổi phân tuyến tính, HAY, 9đ
Luận văn: Phép biến đổi phân tuyến tính, HAY, 9đ
 
Vận dụng giới hạn dãy số trong giải phương trình hàm.pdf
Vận dụng giới hạn dãy số trong giải phương trình hàm.pdfVận dụng giới hạn dãy số trong giải phương trình hàm.pdf
Vận dụng giới hạn dãy số trong giải phương trình hàm.pdf
 
Luận văn: Lớp bài toán tìm giá trị lớn nhất giá trị nhỏ nhất, HOT
Luận văn: Lớp bài toán tìm giá trị lớn nhất giá trị nhỏ nhất, HOTLuận văn: Lớp bài toán tìm giá trị lớn nhất giá trị nhỏ nhất, HOT
Luận văn: Lớp bài toán tìm giá trị lớn nhất giá trị nhỏ nhất, HOT
 
Luận văn: Một số lớp bài toán về loại phương trình hàm, HAY - Gửi miễn phí qu...
Luận văn: Một số lớp bài toán về loại phương trình hàm, HAY - Gửi miễn phí qu...Luận văn: Một số lớp bài toán về loại phương trình hàm, HAY - Gửi miễn phí qu...
Luận văn: Một số lớp bài toán về loại phương trình hàm, HAY - Gửi miễn phí qu...
 
Luận văn: Một số lớp bài toán về phương trình hàm, HAY, 9đ
Luận văn: Một số lớp bài toán về phương trình hàm, HAY, 9đLuận văn: Một số lớp bài toán về phương trình hàm, HAY, 9đ
Luận văn: Một số lớp bài toán về phương trình hàm, HAY, 9đ
 
Luận văn: Giải số phương trình vi phân đại số bằng đa bước, 9đ
Luận văn: Giải số phương trình vi phân đại số bằng đa bước, 9đLuận văn: Giải số phương trình vi phân đại số bằng đa bước, 9đ
Luận văn: Giải số phương trình vi phân đại số bằng đa bước, 9đ
 
Phương Pháp Bình Phương Nhỏ Nhất Và Ứng Dụng.doc
Phương Pháp Bình Phương Nhỏ Nhất Và Ứng Dụng.docPhương Pháp Bình Phương Nhỏ Nhất Và Ứng Dụng.doc
Phương Pháp Bình Phương Nhỏ Nhất Và Ứng Dụng.doc
 
Luận văn: Một số phương pháp giải phương trình hàm, HOT, 9đ
Luận văn: Một số phương pháp giải phương trình hàm, HOT, 9đLuận văn: Một số phương pháp giải phương trình hàm, HOT, 9đ
Luận văn: Một số phương pháp giải phương trình hàm, HOT, 9đ
 
Luận văn: Một số phương pháp giải phương trình hàm, HOT, 9đ
Luận văn: Một số phương pháp giải phương trình hàm, HOT, 9đLuận văn: Một số phương pháp giải phương trình hàm, HOT, 9đ
Luận văn: Một số phương pháp giải phương trình hàm, HOT, 9đ
 
Luận văn: Giải bài toán bất đẳng thức biến phân giả đơn điệu, 9đ
Luận văn: Giải bài toán bất đẳng thức biến phân giả đơn điệu, 9đLuận văn: Giải bài toán bất đẳng thức biến phân giả đơn điệu, 9đ
Luận văn: Giải bài toán bất đẳng thức biến phân giả đơn điệu, 9đ
 
Luận văn: Kết quả về nghiệm của phương trình Cauchy-Riemann
Luận văn: Kết quả về nghiệm của phương trình Cauchy-RiemannLuận văn: Kết quả về nghiệm của phương trình Cauchy-Riemann
Luận văn: Kết quả về nghiệm của phương trình Cauchy-Riemann
 
Cđ giải hpt không mẫu mực
Cđ giải hpt không mẫu mựcCđ giải hpt không mẫu mực
Cđ giải hpt không mẫu mực
 
Luận văn: Phương trình liên hợp và ứng dụng, HAY, 9đ
Luận văn: Phương trình liên hợp và ứng dụng, HAY, 9đLuận văn: Phương trình liên hợp và ứng dụng, HAY, 9đ
Luận văn: Phương trình liên hợp và ứng dụng, HAY, 9đ
 
Luận văn: Phương trình liên hợp và ứng dụng của nó, HOT - Gửi miễn phí qua za...
Luận văn: Phương trình liên hợp và ứng dụng của nó, HOT - Gửi miễn phí qua za...Luận văn: Phương trình liên hợp và ứng dụng của nó, HOT - Gửi miễn phí qua za...
Luận văn: Phương trình liên hợp và ứng dụng của nó, HOT - Gửi miễn phí qua za...
 
Luận văn: Sáu phương pháp giải các bài toán phổ thông, HOT
Luận văn: Sáu phương pháp giải các bài toán phổ thông, HOTLuận văn: Sáu phương pháp giải các bài toán phổ thông, HOT
Luận văn: Sáu phương pháp giải các bài toán phổ thông, HOT
 
[Vnmath.com] 13-ki-thuat-giai-phuong-trinh-ham
[Vnmath.com] 13-ki-thuat-giai-phuong-trinh-ham[Vnmath.com] 13-ki-thuat-giai-phuong-trinh-ham
[Vnmath.com] 13-ki-thuat-giai-phuong-trinh-ham
 
Tính toán nội lực và chuyển vị của dầm bằng sai phân hữu hạn, HAY
Tính toán nội lực và chuyển vị của dầm bằng sai phân hữu hạn, HAYTính toán nội lực và chuyển vị của dầm bằng sai phân hữu hạn, HAY
Tính toán nội lực và chuyển vị của dầm bằng sai phân hữu hạn, HAY
 
Luận văn: Bài toán cực trị với đa thức đối xứng ba biến, HAY - Gửi miễn phí q...
Luận văn: Bài toán cực trị với đa thức đối xứng ba biến, HAY - Gửi miễn phí q...Luận văn: Bài toán cực trị với đa thức đối xứng ba biến, HAY - Gửi miễn phí q...
Luận văn: Bài toán cực trị với đa thức đối xứng ba biến, HAY - Gửi miễn phí q...
 
Đề tài: Tính ổn định của lớp phương trình hàm với cặp biến tự do
Đề tài: Tính ổn định của lớp phương trình hàm với cặp biến tự doĐề tài: Tính ổn định của lớp phương trình hàm với cặp biến tự do
Đề tài: Tính ổn định của lớp phương trình hàm với cặp biến tự do
 
Luận văn: Giải bài toán Dirichlet đối với phương trình Elliptic, 9đ
Luận văn: Giải bài toán Dirichlet đối với phương trình Elliptic, 9đLuận văn: Giải bài toán Dirichlet đối với phương trình Elliptic, 9đ
Luận văn: Giải bài toán Dirichlet đối với phương trình Elliptic, 9đ
 

More from Dịch Vụ Viết Bài Trọn Gói ZALO 0917193864

More from Dịch Vụ Viết Bài Trọn Gói ZALO 0917193864 (20)

Yếu Tố Tự Truyện Trong Truyện Ngắn Thạch Lam Và Thanh Tịnh.doc
Yếu Tố Tự Truyện Trong Truyện Ngắn Thạch Lam Và Thanh Tịnh.docYếu Tố Tự Truyện Trong Truyện Ngắn Thạch Lam Và Thanh Tịnh.doc
Yếu Tố Tự Truyện Trong Truyện Ngắn Thạch Lam Và Thanh Tịnh.doc
 
Từ Ngữ Biểu Thị Tâm Lí – Tình Cảm Trong Ca Dao Người Việt.doc
Từ Ngữ Biểu Thị Tâm Lí – Tình Cảm Trong Ca Dao Người Việt.docTừ Ngữ Biểu Thị Tâm Lí – Tình Cảm Trong Ca Dao Người Việt.doc
Từ Ngữ Biểu Thị Tâm Lí – Tình Cảm Trong Ca Dao Người Việt.doc
 
Quản Lý Hoạt Động Dạy Học Các Môn Khoa Học Tự Nhiên Theo Chuẩn Kiến Thức Và K...
Quản Lý Hoạt Động Dạy Học Các Môn Khoa Học Tự Nhiên Theo Chuẩn Kiến Thức Và K...Quản Lý Hoạt Động Dạy Học Các Môn Khoa Học Tự Nhiên Theo Chuẩn Kiến Thức Và K...
Quản Lý Hoạt Động Dạy Học Các Môn Khoa Học Tự Nhiên Theo Chuẩn Kiến Thức Và K...
 
Quản Lý Thu Thuế Giá Trị Gia Tăng Đối Với Doanh Nghiệp Ngoài Quốc Doanh Trên ...
Quản Lý Thu Thuế Giá Trị Gia Tăng Đối Với Doanh Nghiệp Ngoài Quốc Doanh Trên ...Quản Lý Thu Thuế Giá Trị Gia Tăng Đối Với Doanh Nghiệp Ngoài Quốc Doanh Trên ...
Quản Lý Thu Thuế Giá Trị Gia Tăng Đối Với Doanh Nghiệp Ngoài Quốc Doanh Trên ...
 
Thu Hút Nguồn Nhân Lực Trình Độ Cao Vào Các Cơ Quan Hành Chính Nhà Nước Tỉnh ...
Thu Hút Nguồn Nhân Lực Trình Độ Cao Vào Các Cơ Quan Hành Chính Nhà Nước Tỉnh ...Thu Hút Nguồn Nhân Lực Trình Độ Cao Vào Các Cơ Quan Hành Chính Nhà Nước Tỉnh ...
Thu Hút Nguồn Nhân Lực Trình Độ Cao Vào Các Cơ Quan Hành Chính Nhà Nước Tỉnh ...
 
Quản Trị Rủi Ro Tín Dụng Trong Cho Vay Doanh Nghiệp Tại Ngân Hàng Thương Mại ...
Quản Trị Rủi Ro Tín Dụng Trong Cho Vay Doanh Nghiệp Tại Ngân Hàng Thương Mại ...Quản Trị Rủi Ro Tín Dụng Trong Cho Vay Doanh Nghiệp Tại Ngân Hàng Thương Mại ...
Quản Trị Rủi Ro Tín Dụng Trong Cho Vay Doanh Nghiệp Tại Ngân Hàng Thương Mại ...
 
Vaporisation Of Single And Binary Component Droplets In Heated Flowing Gas St...
Vaporisation Of Single And Binary Component Droplets In Heated Flowing Gas St...Vaporisation Of Single And Binary Component Droplets In Heated Flowing Gas St...
Vaporisation Of Single And Binary Component Droplets In Heated Flowing Gas St...
 
Quản Lý Hoạt Động Dạy Học Các Trường Thpt Trên Địa Bàn Huyện Sơn Hà Tỉnh Quản...
Quản Lý Hoạt Động Dạy Học Các Trường Thpt Trên Địa Bàn Huyện Sơn Hà Tỉnh Quản...Quản Lý Hoạt Động Dạy Học Các Trường Thpt Trên Địa Bàn Huyện Sơn Hà Tỉnh Quản...
Quản Lý Hoạt Động Dạy Học Các Trường Thpt Trên Địa Bàn Huyện Sơn Hà Tỉnh Quản...
 
Tác Giả Hàm Ẩn Trong Tiểu Thuyết Nguyễn Việt Hà.doc
Tác Giả Hàm Ẩn Trong Tiểu Thuyết Nguyễn Việt Hà.docTác Giả Hàm Ẩn Trong Tiểu Thuyết Nguyễn Việt Hà.doc
Tác Giả Hàm Ẩn Trong Tiểu Thuyết Nguyễn Việt Hà.doc
 
Quản Trị Rủi Ro Tín Dụng Trong Cho Vay Ngắn Hạn Tại Ngân Hàng Công Thƣơng Chi...
Quản Trị Rủi Ro Tín Dụng Trong Cho Vay Ngắn Hạn Tại Ngân Hàng Công Thƣơng Chi...Quản Trị Rủi Ro Tín Dụng Trong Cho Vay Ngắn Hạn Tại Ngân Hàng Công Thƣơng Chi...
Quản Trị Rủi Ro Tín Dụng Trong Cho Vay Ngắn Hạn Tại Ngân Hàng Công Thƣơng Chi...
 
Quản Lý Nhà Nước Về Nuôi Trồng Thủy Sản Nước Ngọt Trên Địa Bàn Thành Phố Hải ...
Quản Lý Nhà Nước Về Nuôi Trồng Thủy Sản Nước Ngọt Trên Địa Bàn Thành Phố Hải ...Quản Lý Nhà Nước Về Nuôi Trồng Thủy Sản Nước Ngọt Trên Địa Bàn Thành Phố Hải ...
Quản Lý Nhà Nước Về Nuôi Trồng Thủy Sản Nước Ngọt Trên Địa Bàn Thành Phố Hải ...
 
Song Song Hóa Các Thuật Toán Trên Mạng Đồ Thị.doc
Song Song Hóa Các Thuật Toán Trên Mạng Đồ Thị.docSong Song Hóa Các Thuật Toán Trên Mạng Đồ Thị.doc
Song Song Hóa Các Thuật Toán Trên Mạng Đồ Thị.doc
 
Ứng Dụng Số Phức Trong Các Bài Toán Sơ Cấp.doc
Ứng Dụng Số Phức Trong Các Bài Toán Sơ Cấp.docỨng Dụng Số Phức Trong Các Bài Toán Sơ Cấp.doc
Ứng Dụng Số Phức Trong Các Bài Toán Sơ Cấp.doc
 
Vai Trò Của Cái Bi Trong Giáo Dục Thẩm Mỹ.doc
Vai Trò Của Cái Bi Trong Giáo Dục Thẩm Mỹ.docVai Trò Của Cái Bi Trong Giáo Dục Thẩm Mỹ.doc
Vai Trò Của Cái Bi Trong Giáo Dục Thẩm Mỹ.doc
 
Quản Lý Hoạt Động Giáo Dục Ngoài Giờ Lên Lớp Ở Các Trường Thcs Huyện Chư Păh ...
Quản Lý Hoạt Động Giáo Dục Ngoài Giờ Lên Lớp Ở Các Trường Thcs Huyện Chư Păh ...Quản Lý Hoạt Động Giáo Dục Ngoài Giờ Lên Lớp Ở Các Trường Thcs Huyện Chư Păh ...
Quản Lý Hoạt Động Giáo Dục Ngoài Giờ Lên Lớp Ở Các Trường Thcs Huyện Chư Păh ...
 
Thu Hút Vốn Đầu Tư Vào Lĩnh Vực Nông Nghiệp Trên Địa Bàn Tỉnh Gia Lai.doc
Thu Hút Vốn Đầu Tư Vào Lĩnh Vực Nông Nghiệp Trên Địa Bàn Tỉnh Gia Lai.docThu Hút Vốn Đầu Tư Vào Lĩnh Vực Nông Nghiệp Trên Địa Bàn Tỉnh Gia Lai.doc
Thu Hút Vốn Đầu Tư Vào Lĩnh Vực Nông Nghiệp Trên Địa Bàn Tỉnh Gia Lai.doc
 
Quản Lý Hoạt Động Dạy Học Ngoại Ngữ Tại Các Trung Tâm Ngoại Ngữ - Tin Học Trê...
Quản Lý Hoạt Động Dạy Học Ngoại Ngữ Tại Các Trung Tâm Ngoại Ngữ - Tin Học Trê...Quản Lý Hoạt Động Dạy Học Ngoại Ngữ Tại Các Trung Tâm Ngoại Ngữ - Tin Học Trê...
Quản Lý Hoạt Động Dạy Học Ngoại Ngữ Tại Các Trung Tâm Ngoại Ngữ - Tin Học Trê...
 
Quản Trị Rủi Ro Tín Dụng Trong Cho Vay Doanh Nghiệp Tại Ngân Hàng Thƣơng Mại ...
Quản Trị Rủi Ro Tín Dụng Trong Cho Vay Doanh Nghiệp Tại Ngân Hàng Thƣơng Mại ...Quản Trị Rủi Ro Tín Dụng Trong Cho Vay Doanh Nghiệp Tại Ngân Hàng Thƣơng Mại ...
Quản Trị Rủi Ro Tín Dụng Trong Cho Vay Doanh Nghiệp Tại Ngân Hàng Thƣơng Mại ...
 
Tạo Việc Làm Cho Thanh Niên Trên Địa Bàn Quận Thanh Khê, Thành Phố Đà Nẵng.doc
Tạo Việc Làm Cho Thanh Niên Trên Địa Bàn Quận Thanh Khê, Thành Phố Đà Nẵng.docTạo Việc Làm Cho Thanh Niên Trên Địa Bàn Quận Thanh Khê, Thành Phố Đà Nẵng.doc
Tạo Việc Làm Cho Thanh Niên Trên Địa Bàn Quận Thanh Khê, Thành Phố Đà Nẵng.doc
 
Quản Trị Rủi Ro Tín Dụng Trong Cho Vay Trung Và Dài Hạn Tại Ngân Hàng Thương ...
Quản Trị Rủi Ro Tín Dụng Trong Cho Vay Trung Và Dài Hạn Tại Ngân Hàng Thương ...Quản Trị Rủi Ro Tín Dụng Trong Cho Vay Trung Và Dài Hạn Tại Ngân Hàng Thương ...
Quản Trị Rủi Ro Tín Dụng Trong Cho Vay Trung Và Dài Hạn Tại Ngân Hàng Thương ...
 

Recently uploaded

26 Truyện Ngắn Sơn Nam (Sơn Nam) thuviensach.vn.pdf
26 Truyện Ngắn Sơn Nam (Sơn Nam) thuviensach.vn.pdf26 Truyện Ngắn Sơn Nam (Sơn Nam) thuviensach.vn.pdf
26 Truyện Ngắn Sơn Nam (Sơn Nam) thuviensach.vn.pdf
ltbdieu
 
bài tập lớn môn kiến trúc máy tính và hệ điều hành
bài tập lớn môn kiến trúc máy tính và hệ điều hànhbài tập lớn môn kiến trúc máy tính và hệ điều hành
bài tập lớn môn kiến trúc máy tính và hệ điều hành
dangdinhkien2k4
 
xemsomenh.com-Vòng Tràng Sinh - Cách An 12 Sao Và Ý Nghĩa Từng Sao.pdf
xemsomenh.com-Vòng Tràng Sinh - Cách An 12 Sao Và Ý Nghĩa Từng Sao.pdfxemsomenh.com-Vòng Tràng Sinh - Cách An 12 Sao Và Ý Nghĩa Từng Sao.pdf
xemsomenh.com-Vòng Tràng Sinh - Cách An 12 Sao Và Ý Nghĩa Từng Sao.pdf
Xem Số Mệnh
 
Bài tập nhóm Kỹ Năng Gỉai Quyết Tranh Chấp Lao Động (1).pptx
Bài tập nhóm Kỹ Năng Gỉai Quyết Tranh Chấp Lao Động (1).pptxBài tập nhóm Kỹ Năng Gỉai Quyết Tranh Chấp Lao Động (1).pptx
Bài tập nhóm Kỹ Năng Gỉai Quyết Tranh Chấp Lao Động (1).pptx
DungxPeach
 
C6. Van de dan toc va ton giao ....pdf . Chu nghia xa hoi
C6. Van de dan toc va ton giao ....pdf . Chu nghia xa hoiC6. Van de dan toc va ton giao ....pdf . Chu nghia xa hoi
C6. Van de dan toc va ton giao ....pdf . Chu nghia xa hoi
dnghia2002
 

Recently uploaded (20)

30 ĐỀ PHÁT TRIỂN THEO CẤU TRÚC ĐỀ MINH HỌA BGD NGÀY 22-3-2024 KỲ THI TỐT NGHI...
30 ĐỀ PHÁT TRIỂN THEO CẤU TRÚC ĐỀ MINH HỌA BGD NGÀY 22-3-2024 KỲ THI TỐT NGHI...30 ĐỀ PHÁT TRIỂN THEO CẤU TRÚC ĐỀ MINH HỌA BGD NGÀY 22-3-2024 KỲ THI TỐT NGHI...
30 ĐỀ PHÁT TRIỂN THEO CẤU TRÚC ĐỀ MINH HỌA BGD NGÀY 22-3-2024 KỲ THI TỐT NGHI...
 
26 Truyện Ngắn Sơn Nam (Sơn Nam) thuviensach.vn.pdf
26 Truyện Ngắn Sơn Nam (Sơn Nam) thuviensach.vn.pdf26 Truyện Ngắn Sơn Nam (Sơn Nam) thuviensach.vn.pdf
26 Truyện Ngắn Sơn Nam (Sơn Nam) thuviensach.vn.pdf
 
Bài giảng môn Truyền thông đa phương tiện
Bài giảng môn Truyền thông đa phương tiệnBài giảng môn Truyền thông đa phương tiện
Bài giảng môn Truyền thông đa phương tiện
 
TUYỂN TẬP 50 ĐỀ LUYỆN THI TUYỂN SINH LỚP 10 THPT MÔN TOÁN NĂM 2024 CÓ LỜI GIẢ...
TUYỂN TẬP 50 ĐỀ LUYỆN THI TUYỂN SINH LỚP 10 THPT MÔN TOÁN NĂM 2024 CÓ LỜI GIẢ...TUYỂN TẬP 50 ĐỀ LUYỆN THI TUYỂN SINH LỚP 10 THPT MÔN TOÁN NĂM 2024 CÓ LỜI GIẢ...
TUYỂN TẬP 50 ĐỀ LUYỆN THI TUYỂN SINH LỚP 10 THPT MÔN TOÁN NĂM 2024 CÓ LỜI GIẢ...
 
Giới thiệu Dự án Sản Phụ Khoa - Y Học Cộng Đồng
Giới thiệu Dự án Sản Phụ Khoa - Y Học Cộng ĐồngGiới thiệu Dự án Sản Phụ Khoa - Y Học Cộng Đồng
Giới thiệu Dự án Sản Phụ Khoa - Y Học Cộng Đồng
 
30 ĐỀ PHÁT TRIỂN THEO CẤU TRÚC ĐỀ MINH HỌA BGD NGÀY 22-3-2024 KỲ THI TỐT NGHI...
30 ĐỀ PHÁT TRIỂN THEO CẤU TRÚC ĐỀ MINH HỌA BGD NGÀY 22-3-2024 KỲ THI TỐT NGHI...30 ĐỀ PHÁT TRIỂN THEO CẤU TRÚC ĐỀ MINH HỌA BGD NGÀY 22-3-2024 KỲ THI TỐT NGHI...
30 ĐỀ PHÁT TRIỂN THEO CẤU TRÚC ĐỀ MINH HỌA BGD NGÀY 22-3-2024 KỲ THI TỐT NGHI...
 
xemsomenh.com-Vòng Lộc Tồn - Vòng Bác Sĩ và Cách An Trong Vòng Lộc Tồn.pdf
xemsomenh.com-Vòng Lộc Tồn - Vòng Bác Sĩ và Cách An Trong Vòng Lộc Tồn.pdfxemsomenh.com-Vòng Lộc Tồn - Vòng Bác Sĩ và Cách An Trong Vòng Lộc Tồn.pdf
xemsomenh.com-Vòng Lộc Tồn - Vòng Bác Sĩ và Cách An Trong Vòng Lộc Tồn.pdf
 
Giáo trình nhập môn lập trình - Đặng Bình Phương
Giáo trình nhập môn lập trình - Đặng Bình PhươngGiáo trình nhập môn lập trình - Đặng Bình Phương
Giáo trình nhập môn lập trình - Đặng Bình Phương
 
bài tập lớn môn kiến trúc máy tính và hệ điều hành
bài tập lớn môn kiến trúc máy tính và hệ điều hànhbài tập lớn môn kiến trúc máy tính và hệ điều hành
bài tập lớn môn kiến trúc máy tính và hệ điều hành
 
Đề thi tin học HK2 lớp 3 Chân Trời Sáng Tạo
Đề thi tin học HK2 lớp 3 Chân Trời Sáng TạoĐề thi tin học HK2 lớp 3 Chân Trời Sáng Tạo
Đề thi tin học HK2 lớp 3 Chân Trời Sáng Tạo
 
xemsomenh.com-Vòng Tràng Sinh - Cách An 12 Sao Và Ý Nghĩa Từng Sao.pdf
xemsomenh.com-Vòng Tràng Sinh - Cách An 12 Sao Và Ý Nghĩa Từng Sao.pdfxemsomenh.com-Vòng Tràng Sinh - Cách An 12 Sao Và Ý Nghĩa Từng Sao.pdf
xemsomenh.com-Vòng Tràng Sinh - Cách An 12 Sao Và Ý Nghĩa Từng Sao.pdf
 
Bài tập nhóm Kỹ Năng Gỉai Quyết Tranh Chấp Lao Động (1).pptx
Bài tập nhóm Kỹ Năng Gỉai Quyết Tranh Chấp Lao Động (1).pptxBài tập nhóm Kỹ Năng Gỉai Quyết Tranh Chấp Lao Động (1).pptx
Bài tập nhóm Kỹ Năng Gỉai Quyết Tranh Chấp Lao Động (1).pptx
 
BỘ LUYỆN NGHE VÀO 10 TIẾNG ANH DẠNG TRẮC NGHIỆM 4 CÂU TRẢ LỜI - CÓ FILE NGHE.pdf
BỘ LUYỆN NGHE VÀO 10 TIẾNG ANH DẠNG TRẮC NGHIỆM 4 CÂU TRẢ LỜI - CÓ FILE NGHE.pdfBỘ LUYỆN NGHE VÀO 10 TIẾNG ANH DẠNG TRẮC NGHIỆM 4 CÂU TRẢ LỜI - CÓ FILE NGHE.pdf
BỘ LUYỆN NGHE VÀO 10 TIẾNG ANH DẠNG TRẮC NGHIỆM 4 CÂU TRẢ LỜI - CÓ FILE NGHE.pdf
 
TÀI LIỆU BỒI DƯỠNG HỌC SINH GIỎI LÝ LUẬN VĂN HỌC NĂM HỌC 2023-2024 - MÔN NGỮ ...
TÀI LIỆU BỒI DƯỠNG HỌC SINH GIỎI LÝ LUẬN VĂN HỌC NĂM HỌC 2023-2024 - MÔN NGỮ ...TÀI LIỆU BỒI DƯỠNG HỌC SINH GIỎI LÝ LUẬN VĂN HỌC NĂM HỌC 2023-2024 - MÔN NGỮ ...
TÀI LIỆU BỒI DƯỠNG HỌC SINH GIỎI LÝ LUẬN VĂN HỌC NĂM HỌC 2023-2024 - MÔN NGỮ ...
 
powerpoint mẫu họp phụ huynh cuối kì 2 học sinh lớp 7 bgs
powerpoint mẫu họp phụ huynh cuối kì 2 học sinh lớp 7 bgspowerpoint mẫu họp phụ huynh cuối kì 2 học sinh lớp 7 bgs
powerpoint mẫu họp phụ huynh cuối kì 2 học sinh lớp 7 bgs
 
C6. Van de dan toc va ton giao ....pdf . Chu nghia xa hoi
C6. Van de dan toc va ton giao ....pdf . Chu nghia xa hoiC6. Van de dan toc va ton giao ....pdf . Chu nghia xa hoi
C6. Van de dan toc va ton giao ....pdf . Chu nghia xa hoi
 
Trắc nghiệm CHƯƠNG 5 môn Chủ nghĩa xã hội
Trắc nghiệm CHƯƠNG 5 môn Chủ nghĩa xã hộiTrắc nghiệm CHƯƠNG 5 môn Chủ nghĩa xã hội
Trắc nghiệm CHƯƠNG 5 môn Chủ nghĩa xã hội
 
Danh sách sinh viên tốt nghiệp Đại học - Cao đẳng Trường Đại học Phú Yên năm ...
Danh sách sinh viên tốt nghiệp Đại học - Cao đẳng Trường Đại học Phú Yên năm ...Danh sách sinh viên tốt nghiệp Đại học - Cao đẳng Trường Đại học Phú Yên năm ...
Danh sách sinh viên tốt nghiệp Đại học - Cao đẳng Trường Đại học Phú Yên năm ...
 
ĐỀ CHÍNH THỨC KỲ THI TUYỂN SINH VÀO LỚP 10 THPT CÁC TỈNH THÀNH NĂM HỌC 2020 –...
ĐỀ CHÍNH THỨC KỲ THI TUYỂN SINH VÀO LỚP 10 THPT CÁC TỈNH THÀNH NĂM HỌC 2020 –...ĐỀ CHÍNH THỨC KỲ THI TUYỂN SINH VÀO LỚP 10 THPT CÁC TỈNH THÀNH NĂM HỌC 2020 –...
ĐỀ CHÍNH THỨC KỲ THI TUYỂN SINH VÀO LỚP 10 THPT CÁC TỈNH THÀNH NĂM HỌC 2020 –...
 
TÀI LIỆU BỒI DƯỠNG HỌC SINH GIỎI KỸ NĂNG VIẾT ĐOẠN VĂN NGHỊ LUẬN XÃ HỘI 200 C...
TÀI LIỆU BỒI DƯỠNG HỌC SINH GIỎI KỸ NĂNG VIẾT ĐOẠN VĂN NGHỊ LUẬN XÃ HỘI 200 C...TÀI LIỆU BỒI DƯỠNG HỌC SINH GIỎI KỸ NĂNG VIẾT ĐOẠN VĂN NGHỊ LUẬN XÃ HỘI 200 C...
TÀI LIỆU BỒI DƯỠNG HỌC SINH GIỎI KỸ NĂNG VIẾT ĐOẠN VĂN NGHỊ LUẬN XÃ HỘI 200 C...
 

Luận văn: Phương pháp giải bài toán cực trị, HAY

  • 1. ĐẠI HỌC QUỐC GIA HÀ NỘI TRƯỜNG ĐẠI HỌC KHOA HỌC TỰ NHIÊN HÀ NỘI VŨ THỊ HẢI THANH PHƯƠNG PHÁP GIẢI BÀI TOÁN CỰC TRỊ VÀ ỨNG DỤNG LUẬN VĂN THẠC SĨ KHOA HỌC Chuyên ngành: Phương pháp toán sơ cấp Mã số: 60 46 40 NGƯỜI HƯỚNG DẪN KHOA HỌC PGS.TS Nguyễn Đình Sang Hà Nội - 2012
  • 2. Mục lục Mục lục . . . . . . . . . . . . . . . . . . . . . . . . . . . . . i Lời nói đầu . . . . . . . . . . . . . . . . . . . . . . . . . . . iii Lời cảm ơn . . . . . . . . . . . . . . . . . . . . . . . . . . . . iv Bảng kí hiệu . . . . . . . . . . . . . . . . . . . . . . . . . . v 1 Cực trị hàm số 1 1.1 Kiến thức chuẩn bị . . . . . . . . . . . . . . . . . . . . . 1 1.2 Các phương pháp tìm cực trị . . . . . . . . . . . . . . . . 2 1.2.1 Áp dụng điều kiện cần, điều kiện đủ . . . . . . . 2 1.2.2 Phương pháp sử dụng bất đẳng thức . . . . . . . 9 1.3 Một số bài toán tổng quát và ứng dụng . . . . . . . . . . 11 1.3.1 Bài toán tổng quát . . . . . . . . . . . . . . . . . 11 1.3.2 Bài tập tham khảo . . . . . . . . . . . . . . . . . 14 2 Giá trị lớn nhất, giá trị nhỏ nhất 19 2.1 Các khái niệm cơ bản . . . . . . . . . . . . . . . . . . . . 19 2.1.1 Giá trị lớn nhất, giá trị nhỏ nhất của hàm số . . . 19 2.1.2 Giá trị lớn nhất, giá trị nhỏ nhất của một tập hợp 20 2.1.3 Một số tính chất của giá trị lớn nhất, giá trị nhỏ nhất . . . . . . . . . . . . . . . . . . . . . . . . . 22 2.1.4 Một số định lý về giá trị lớn nhất, giá trị nhỏ nhất 24 2.2 Các phương pháp tìm giá trị lớn nhất, giá trị nhỏ nhất . 26 2.2.1 Phương pháp sử dụng đạo hàm . . . . . . . . . . 26 2.2.2 Phương pháp tập giá trị . . . . . . . . . . . . . . 31 2.2.3 Phương pháp lượng giác . . . . . . . . . . . . . . 37 2.2.4 Phương pháp hình học . . . . . . . . . . . . . . . 44 i
  • 3. MỤC LỤC 2.2.5 Phương pháp sử dụng bất đẳng thức . . . . . . . 51 2.2.6 Một số bài tập vận dụng . . . . . . . . . . . . . . 63 Kết luận . . . . . . . . . . . . . . . . . . . . . . . . . . . . . 69 Tài liệu tham khảo . . . . . . . . . . . . . . . . . . . . . . . 70 ii
  • 4. Lời nói đầu Bài toán cực trị địa phương và cực trị tuyệt đối là những bài toán rất quan trọng trong giải tích toán học và có nhiều ứng dụng khác nhau trong toán học cũng như trong nhiều ngành khoa học khác như: Kinh tế, Khoa học công nghệ, ...v.v. Để giải bài toán cực trị, có nhiều phương pháp khác nhau. Mục đích của luận văn là giới thiệu các phương pháp giải dạng toán này, cho bình luận về các phương pháp đó đồng thời đưa ra một số ứng dụng. Những ứng dụng của bài toán cực trị có rất nhiều, nhưng vì giới hạn trong phương pháp toán sơ cấp và hạn chế trong một luận văn thạc sĩ nên bản luận văn chỉ nêu ra một số ứng dụng cơ bản. Bản luận văn gồm 2 chương: Chương 1: Cực trị hàm số. Trình bày bài toán cực trị địa phương, đưa ra điều kiện cần, điều kiện đủ để có cực trị. Cho những ví dụ không thỏa mãn điều kiện đủ nhưng vẫn có cực trị. Trình bày các phương pháp khác nhau để giải bài toán cực trị, tổng quát hóa một số bài toán về cực trị với mong muốn đưa ra cách giải nhanh gọn cho các bài toán dạng này. Chương 2: Giá trị lớn nhất, giá trị nhỏ nhất. Phần đầu của chương trình bày định nghĩa giá trị lớn nhất, giá trị nhỏ nhất của hàm số trên một tập, điều kiện đủ để tồn tại giá trị lớn nhất, giá trị nhỏ nhất của hàm một biến và các tính chất của giá trị lớn nhất, giá trị nhỏ nhất. Trong phạm vi chương trình phổ thông, hàm số nhiều biến không được nghiên cứu. Vì vậy để tìm giá trị lớn nhất, giá nhỏ nhất của hàm nhiều biến, ta phải quy về bài toán tìm giá trị lớn nhất, giá trị nhỏ nhất của một tập hợp số. Phần tiếp theo luận văn trình bày một số phương pháp khác nhau để giải bài toán tìm giá trị lớn nhất, giá trị nhỏ nhất trong đó dành nhiều thời gian cho phương pháp bất đẳng thức. Phần cuối chương là một số bài toán vận dụng phối hợp nhiều phương pháp.
  • 5. Lời cảm ơn Hoàn thành được luận văn này, ngoài sự nỗ lực của bản thân, tôi đã nhận được sự chỉ bảo, giúp đỡ từ nhiều phía của các thầy, cô giáo, gia đình và bạn bè. Tôi xin bày tỏ lòng biết ơn sâu sắc tới người thầy kính mến PGS.TS. Nguyễn Đình Sang, người đã trực tiếp truyền thụ kiến thức, quyết định hướng nghiên cứu và tận tình hướng dẫn cho tôi hoàn thành bản luận văn. Tôi xin chân thành cảm ơn các thầy, cô giáo khoa Toán - Cơ - Tin học, Trường Đại học Khoa học tự nhiên - Đại học Quốc gia Hà Nội, những người đã trực tiếp giảng dạy và giúp đỡ tôi trong quá trình học tập tại trường cùng toàn thể bạn bè và người thân đã đóng góp ý kiến, giúp đỡ, động viên tôi trong quá trình học tập, nghiên cứu và hoàn thành luận văn này. Ứng dụng của bài toán cực trị có rất nhiều, nhưng vì giới hạn trong phương pháp toán sơ cấp và hạn chế trong một luận văn thạc sĩ nên bản luận văn mới chỉ trình bày được một phần nào đó. Do thời gian có hạn và năng lực có phần hạn chế nên chắc chắn luận văn không tránh khỏi những thiếu sót. Kính mong nhận được ý kiến đóng góp của các thầy cô và bạn bè đồng nghiệp để bản luận văn được hoàn chỉnh hơn. Xin chân thành cảm ơn. Hà Nội, ngày 20 tháng 11 năm 2012 Học viên Vũ Thị Hải Thanh
  • 6. Bảng kí hiệu N tập các số tự nhiên N∗ tập các số tự nhiên khác không Z tập các số nguyên Z+ tập số nguyên không âm Z∗ + tập số nguyên dương R tập số thực R∗ tập số thực khác không R+ tập số thực không âm R∗ + tập số thực dương C tập số phức [a; b] = {x ∈ R|a ≤ x ≤ b} (a; b) = {x ∈ R|a < x < b} (a; b] = {x ∈ R|a < x ≤ b} v
  • 7. Chương 1 Cực trị hàm số Trong chương này, chúng ta sẽ trình bày khái niệm về cực trị hàm số. Điều kiện để có cực trị hàm số, đưa ra một số ví dụ minh họa điều kiện cần, điều kiện đủ cũng như giới thiệu các phương pháp tìm cực trị kèm theo các ví dụ và bài tập. 1.1 Kiến thức chuẩn bị Định nghĩa 1.1. Cho khoảng (a; b) ⊂ R và hàm số f : (a; b) → R. Ta nói rằng, hàm f đạt cực đại địa phương(tương ứng cực tiểu địa phương) tại x0 ∈ (a; b) nếu: ∃δ sao cho (x0 − δ; x0 + δ) ⊂ (a; b) và f(x0) ≥ f(x) (tương ứng f(x0) ≤ f(x)), với mọi x ∈ (x0 − δ; x0 + δ) và f không phải là một hằng số trong một lân cận nào đó của x0. Điểm x0 mà tại đó hàm đạt cực đại địa phương hoặc cực tiểu địa phương được gọi chung là điểm cực trị của hàm số. Định lý 1.1. (Định lý Fermat - Điều kiện cần để hàm số có cực trị) Cho khoảng (a; b) ⊂ R và hàm số f : (a; b) → R. Nếu điểm c ∈ (a; b) là điểm cực trị của hàm số f và nếu tồn tại f (c) thì f (c) = 0. Điểm x0 mà tại đó f (x0) = 0 hoặc đạo hàm không xác định được gọi là điểm dừng của hàm f. Nhận xét: Nếu hàm f : (a; b) → R là hàm khả vi trên (a; b) thì những điểm cực trị của f phải nằm trong số các điểm dừng của f. Định lý 1.2. (Điều kiện đủ để hàm số đạt cực trị) Giả sử hàm số f liên tục trên (a; b) chứa điểm x0 và có đạo hàm trên 1
  • 8. Chương 1. Cực trị hàm số các khoảng (a; x0) và (x0; b). - Nếu f (x) đổi dấu từ âm sang dương khi x qua điểm x0 thì hàm số đạt cực tiểu tại điểm x0. - Nếu f (x) đổi dấu từ dương sang âm khi x qua điểm x0 thì hàm số đạt cực đại tại điểm x0. Định lý 1.3. Giả sử hàm số f(x) xác định trên khoảng (a; b), x0 là một điểm dừng của f(x). Hàm f(x) khả vi cấp 1 và cấp 2 tại x0. Khi đó: - Nếu f (x0) < 0 thì hàm số f đạt cực đại tại x0. - Nếu f (x0) > 0 thì hàm số f đạt cực tiểu tại x0. 1.2 Các phương pháp tìm cực trị 1.2.1 Áp dụng điều kiện cần, điều kiện đủ Dựa vào điều kiện cần, điều kiện đủ để hàm số đạt cực trị, ta xây dựng các quy tắc tìm cực trị của hàm số f(x) liên tục trên khoảng (a; b) sau đây: Quy tắc 1. - Tìm f (x) ; - Tìm các điểm xi, (i = 1, 2, 3, ...) mà tại đó f có đạo hàm bằng 0 hoặc hàm số liên tục nhưng không có đạo hàm; - Xét dấu f (x).Nếu f (x) đổi dấu khi x qua điểm xi thì hàm số đạt cực trị tại xi. Ví dụ 1.1. Tìm cực trị của hàm số: y = 3 x(1 − x)2 Lời giải. Hàm y xác định và liên tục trên R. Với mọi x = 0 và x = 1 y = 1 − 3x 3 3 x2(1 − x) 2
  • 9. Chương 1. Cực trị hàm số y = 0 ⇔ x = 1 3 Lập bảng biến thiên của hàm y: x y y −∞ 0 1 3 1 +∞ + + 0 − + −∞−∞ 3 √ 4 3 3 √ 4 3 00 +∞+∞ 0 Từ bảng biến thiên ta thấy: Hàm số đạt cực đại tại x = 1 3, giá trị cực đại của hàm số là y(1 3) = 3 √ 4 3 . Hàm số đạt cực tiểu tại x = 1, giá trị cực tiểu của hàm số là y(1) = 0. Chú ý: Khi qua điểm x = 0 đạo hàm y không đổi dấu nên hàm số đã cho không có cực trị tại điểm x = 0. Ví dụ 1.2. Tìm cực trị của hàm số: y = −x2 + 2x + 3 Lời giải. Hàm y xác định trên R. Ta có y = −x2 + 2x + 3 = (|−x2 + 2x + 3|)2 ⇔ y = (−x2 + 2x + 3)(−2x + 2) |−x2 + 2x + 3| = f(x) |−x2 + 2x + 3| Xét f(x) = (−x2 + 2x + 3)(−2x + 2) = 0 ⇔ x = ±1 x = 3 Lập bảng biến thiên của hàm y x y y −∞ −1 1 3 +∞ − + 0 − + +∞+∞ 00 44 00 +∞+∞ 3
  • 10. Chương 1. Cực trị hàm số Từ bảng biến thiên ta suy ra Giá trị cực đại của hàm số y(1) = 4 Giá trị cực tiểu của hàm số y(−1) = 0; y(3) = 0 Ví dụ 1.3. Tìm cực trị của hai hàm số sau: f(x) = xe−1 x với x = 0 0 với x = 0 g(x) = e− 1 x2 với x = 0 0 với x = 0 Lời giải. Ta có: f (x) = e− 1 x2 + 2 x2 e− 1 x2 , ∀x = 0 Nhận thấy f (x) > 0, ∀x = 0 Mặt khác, do lim x→0− xe−1 x = −∞ Nên hàm f(x) không liên tục tại x = 0. Từ đó suy ra hàm f(x) không có cực trị. Hàm g(x) liên tục với mọi x,vì lim x→0 e− 1 x2 = 0. Ta thấy với mọi x = 0 g (x) = 2 x3 e− 1 x2 Lập bảng biến thiên của hàm g(x): x g (x) g(x) −∞ 0 +∞ − + +∞+∞ 00 +∞+∞ 4
  • 11. Chương 1. Cực trị hàm số Từ bảng biến thiên suy ra, giá trị cực tiểu của hàm số là g(0) = 0. Nhận xét: Hai hàm f(x) và g(x) đều có đạo hàm không xác định tại điểm x = 0. Nhưng khi qua điểm x = 0: hàm g(x) có đạo hàm đổi dấu nên g(x) mới có cực trị, còn hàm f(x) thì đạo hàm không đổi dấu nên không tồn tại cực trị. Ví dụ 1.4. Tìm cực trị của hàm số: y = (1 + x + x2 2! + ... + xn n! )e−x , n ∈ N∗ . Lời giải. Hàm số đã cho xác định trên R. Ta có y = − xn n! e−x , n ∈ N∗ • Với n = 2k, k ∈ N Khi đó y = − xn n! e−x < 0, ∀x ∈ R Do đó hàm số không có cực trị. • Với n = 2k + 1, k ∈ N Ta có y = 0 ⇔ x = 0 Lập bảng biến thiên của hàm y : x y y −∞ 0 +∞ + 0 − −∞−∞ 00 −∞−∞ Vậy giá trị cực đại của hàm y là y(0) = 0. Ví dụ 1.5. Tìm cực trị của hàm số sau: f(x) = 2 − x2 (2 + sin 1 x) với x = 0 2 với x = 0 5
  • 12. Chương 1. Cực trị hàm số Lời giải. Hàm số đã cho xác định và liên tục trên R vì lim x→0 f(x) = 2. Nhận thấy: f(x) − f(0) = −x2 (2 + sin 1 x) < 0, ∀x = 0 Mặt khác f (x) = −2x(2 + sin 1 x ) + cos 1 x Với xk = 1 kπ , k ∈ Z, ta có: cos 1 xk = 1 với k chẵn −1 với k lẻ Từ đó suy ra: f (x) > 0 với k chẵn f (x) < 0 với k lẻ. Như vậy f (x) đổi dấu trong khoảng (0; 1 kπ ) Tuy vậy ta vẫn kết luận được giá trị cực đại của hàm f(x) là f(0) = 2. Nhận xét: Như vậy không phải hàm số nào cũng có đạo hàm không đổi dấu về một lân cận ở phía phải (hay phía trái) của điểm cực trị. Hàm số trong ví dụ (1.5) không thỏa mãn điều kiện đủ nhưng vẫn có cực trị. Các hàm sơ cấp thường không có tình trạng này. Quy tắc 2. - Tìm f (x) ; - Tìm các nghiệm xi, (i = 1, 2, 3, ...) của phương trình f (x) = 0; - Tìm f (x) và tính f (xi) : Nếu f (xi) > 0 thì hàm số đạt cực tiểu tại điểm xi. Nếu f (xi) < 0 thì hàm số đạt cực đại tại điểm xi. Ví dụ 1.6. Tìm cực trị của hàm số: y = x − sin2x + 2 Lời giải. Hàm số đã cho xác định trên R. Ta có y = 1 − 2 cos 2x; y = 0 ⇔ x = ± π 6 + kπ (k ∈ Z) 6
  • 13. Chương 1. Cực trị hàm số • Vì y = (π 6 + kπ) = 4 sin(π 3 + k2π) > 0 Nên hàm số đạt cưc tiểu tại x = π 6 + kπ và giá trị cực tiểu của hàm số là y(π 6 + kπ) = π 6 − √ 3 2 + 2 + kπ, k ∈ Z. • Vì y (−π 6 + kπ) = 4sin(π 3 + k2π) < 0 Nên hàm số đạt cực đại tại x = −π 6 + kπ và giá trị cực đại của hàm số là y(−π 6 + kπ) = −π 6 + √ 3 2 + 2 + kπ, k ∈ Z. Ví dụ 1.7. Tìm cực trị của hàm số: y = ex + e−x 2 Lời giải. Hàm số đã cho xác định trên R. Ta có y = ex − e−x 2 y = 0 ⇔ ex = e−x ⇔ x = 0 Lại có y = ex + e−x 2 ⇒ y (0) = 1 > 0 Như vậy y (0) = 0; y (0) > 0 Vậy giá trị cực tiểu của hàm số là y(0) = 1 Ví dụ 1.8. Tìm cực trị của hàm số: y = x2 + x + 1 + x2 − x + 1 Lời giải. Hàm số đã cho xác định trên R. Ta có y = 2x + 1 2 √ x2 + x + 1 + 2x − 1 2 √ x2 − x + 1 y = 0 ⇔ (2x + 1) x2 − x + 1 − (1 − 2x) x2 + x + 1 = 0 ⇔ 2x[( x2 − x + 1 + x2 + x + 1) − 1] = 0 ⇔ x = 0√ x2 − x + 1 + √ x2 + x + 1 = 1 7
  • 14. Chương 1. Cực trị hàm số Do x2 − x + 1+ x2 + x + 1 = (x + 1 2 )2 + 3 4 + (x − 1 2 )2 + 3 4 ≥ 2 √ 3 2 > 1, ∀x Nên y = 0 ⇔ x = 0 Mặt khác y = 4 √ x2 + x + 1 − (2x+1)2 √ x2+x+1 4(x2 + x + 1) + 4 √ x2 − x + 1 − (2x−1)2 √ x2−x+1 4(x2 − x + 1) Suy ra y (0) = 3 2 > 0 Vậy hàm số đạt cực tiểu tại x = 0 và giá trị cực tiểu của hàm số là y(0) = 2. Ví dụ 1.9. Cho hàm số y = (x − m)3 − 3x. Tìm m để hàm số đạt cực đại tại điểm có hoành độ x = 0. Lời giải. Hàm số đã cho xác định trên R. Ta có y = 3(x − m)2 − 3, y = 6(x − m). Hàm số đạt cực đại tại điểm có hoành độ x = 0 khi: y (0) = 0 y (0) < 0 ⇔ 3m2 − 3 = 0 −6m < 0 ⇔    m = −1 m = 1 m > 0 ⇔ m = 1 Vậy m = 1 hàm số đã cho đạt cực đại tại điểm x = 0. Ví dụ 1.10. Cho hàm số y = −2x + k √ x2 + 1. Tìm k để hàm số có cực tiểu. Lời giải. Hàm số đã cho xác định và liên tục với mọi x ∈ R. Ta có y = −2 + kx √ x2 + 1 = kx − 2 √ x2 + 1 √ x2 + 1 8
  • 15. Chương 1. Cực trị hàm số y = k (x2 + 1) √ x2 + 1 Do đó y và y xác định và liên tục với mọi x ∈ R Xét các trường hợp: • Nếu k = 0 thì y = −2x, nên hàm số không có cực tri. • Nếu k < 0 thì y < 0, ∀x ∈ R, nên hàm số hoặc không có cực trị, hoặc chỉ có cực đại. • Nếu k > 0 thì y > 0, ∀x ∈ R, nên hàm số đã cho có cực tiểu khi và chỉ khi phương trình y = 0 có nghiệm. y = 0 ⇔ 2 x2 + 1 = kx ⇔ kx ≥ 0 4(x2 + 1) = k2 x2 ⇔ x ≥ 0 (1) (k2 − 4)x2 = 4 (2) Phương trình y = 0 có nghiệm khi và chỉ khi (2) có nghiệm. Suy ra k2 − 4 > 0 hay k > 2. Vậy k > 2 hàm số đã cho có cực tiểu. Nhận xét: Quy tắc 2 tìm cực trị hàm số được áp dụng trong trường hợp hàm số chứa tham số hoặc khó xét dấu đạo hàm bậc nhất. 1.2.2 Phương pháp sử dụng bất đẳng thức Khi tìm cực trị của các hàm số không tính được đạo hàm hoặc tính được đạo hàm nhưng việc tìm nghiệm của phương trình y = 0 gặp nhiều khó khăn, ta có thể sử dụng phương pháp bất đẳng thức để tìm cực trị của hàm số. Nội dung của phương pháp như sau: Cho hàm số y = f(x) xác định trên D. + Nếu ta tìm được giá trị x0 mà f(x) ≥ f(x0), ∀x0 ∈ (x0 −δ, x0 +δ) với (x0 − δ, x0 + δ) ⊂ D, ∀δ > 0 và f không phải là hằng số trong một lân cận của x0 thì hàm số đạt cực tiểu tại x0. + Nếu ta tìm được giá trị x1 mà f(x) ≤ f(x1), ∀x1 ∈ (x1 −δ, x1 +δ) với (x1 − δ, x1 + δ) ⊂ D, ∀δ > 0 và f không phải là hằng số trong một lân cận của x1 thì hàm số đạt cực đại tại x1. 9
  • 16. Chương 1. Cực trị hàm số Sau đây ta sẽ xét một số ví dụ vận dụng phương pháp bất đẳng thức để tìm cực trị hàm số. Ví dụ 1.11. Tìm cực trị của hàm số: y = √ sin x − 4 √ cos x Trong trường hợp này, nếu sử dụng phương pháp xét dấu đạo hàm thì sẽ phức tạp hơn rất nhiều. Mặc dù đây là hàm lượng giác nhưng ta cũng không vận dụng được quy tắc 2 cho ví dụ này, do đó ta cần vận dụng một phương pháp khác. Lời giải. Với mọi x thuộc tập xác định của hàm số, ta luôn có: 0 ≤ √ sin x ≤ 1 −1 ≤ − 4 √ cos x ≤ 0 Suy ra −1 ≤ y = √ sin x − 4 √ cos x ≤ 1 Vậy hàm số đạt cực đại tại x = π 2 + k2π, k ∈ Z, giá trị cực đại của hàm số là y = 1. Hàm số đạt cực tiểu tại x = k2π, giá trị cực tiểu của hàm số là y = −1. Ví dụ 1.12. Xét hàm số: f(x, y) = (x2 + y2 )e−(x2 +y2 ) Tìm cực trị của hàm số f. Đây là một hàm số 2 biến, trong phạm vi chương trình phổ thông ta không sử dụng được hai quy tắc tìm cực trị trong trường hợp này. Ví dụ này được giải bằng phương pháp bất đẳng thức như sau. Lời giải. Hàm số đã cho xác định trên R. Ta thấy f(x; y) ≥ 0, ∀(x; y) f(0; 0) = 0 Suy ra f(x; y) ≥ f(0; 0), ∀(x; y) Vậy hàm số đạt cực tiểu tại một điểm duy nhất khi x = y = 0, và giá trị cực tiểu của hàm số là f(0; 0) = 0. 10
  • 17. Chương 1. Cực trị hàm số Ví dụ 1.13. Tìm cực tiểu của hàm số f(x) = x3 + 2(1 + x3 + 1) + x3 + 2(1 − x3 + 1) với x ≥ 0 Lời giải. Hàm số đã cho xác định với mọi x ≥ 0 Ta có f(x) = ( x3 + 1 + 1)2 + ( x3 + 1 − 1)2 = x3 + 1 + 1 + x3 + 1 − 1 ≥ 2 x3 + 1 Suy ra f(x) ≥ 2 Dấu bằng xảy ra khi x = 0 Vậy giá trị cực tiểu của hàm số f(0) = 2 1.3 Một số bài toán tổng quát và ứng dụng 1.3.1 Bài toán tổng quát Bài toán 1. Cho hàm số f(x) > 0, ∀x ∈ TXĐ và hàm F(x) = cf2 (x), với c > 0 bất kỳ. Chứng minh rằng F(x) và f(x) có cùng các điểm cực trị. Chứng minh. • Giả sử x0 là điểm cực đại của hàm f(x), tức là 0 < f(x) < f(x0), x ∈ {0 < |x − x0| < δ} Từ đó suy ra cf2 (x) < cf2 (x0), với c > 0 Hay F(x) < F(x0), x ∈ {0 < |x − x0| < δ} Vậy x0 cũng là điểm cực đại của hàm F(x). Tương tự với trường hợp x1 là điểm cực tiểu. • Giả sử x0 là điểm cực đại của hàm F(x), tức là 0 < F(x) < F(x0), x ∈ {0 < |x − x0| < δ} Lại có F(x) = cf2 (x), với c > 0, f(x) > 0, x ∈ {0 < |x − x0| < δ} Từ đó suy ra f(x) < f(x0), x ∈ {0 < |x − x0| < δ} Vậy x0 cũng là điểm cực đại của hàm f(x). 11
  • 18. Chương 1. Cực trị hàm số Tương tự với trường hợp x1 là điểm cực tiểu. Kết luận: f(x) và F(x) có cùng cực trị. Nhận xét: Tổng quát hóa bái toán trên, ta được kết quả sau đây: Cho hàm số f(x) > 0, ∀x ∈ TXĐ và hàm F(x) = cf2n (x), với c > 0 bất kỳ và n ∈ N∗ . Khi đó hai hàm F(x) và f(x) có cùng các điểm cực trị. Ví dụ 1.14. Tìm cực trị của hàm số: f(x) = 4 x2 + x + 1 Lời giải. Hàm số đã cho xác định trên R. Đặt F(x) = f4 (x) = x2 + x + 1 ⇒ F (x) = 2x + 1 F (x) = 0 ⇔ x = − 1 2 Lập bảng biến thiên của hàm F(x): x F (x) F(x) −∞ −1 2 +∞ − 0 + +∞+∞ 3 4 3 4 +∞+∞ Từ bảng biến thiên ta thấy hàm F(x) đạt cực tiểu tại x = −1 2 Lại có: f(x) > 0, ∀x ∈ R và hàm F(x) = f4 (x). Nên hàm F(x) và f(x) có cùng các điểm cực trị. Do đó hàm f(x) cũng đạt cực tiểu tại x = −1 2, giá trị cực tiểu của hàm f(x) là f(−1 2) = 4 3 4. Bài toán 2. Cho hàm số f(x) xác định và liên tục trên D ⊂ R và ϕ(x) là một hàm đồng biến, liên tục với mọi x ∈ R. Chứng minh rằng hàm f(x) có cùng cực trị với hàm ϕ[f(x)]. Chứng minh. Giả sử hàm f(x) đạt cực đại tại điểm x0, tức là f(x) ≤ f(x0), ∀x ∈ D. 12
  • 19. Chương 1. Cực trị hàm số Vì ϕ(x) là hàm đồng biến với mọi x ∈ R. Nên ϕ[f(x)] ≤ ϕ[f(x0)], ∀x ∈ D Như vậy hàm ϕ[f(x)] cũng đạt cực đại tại điểm x0 Chứng minh tương tự với trường hợp cực tiểu. Kết luận: hàm f(x) và ϕ[f(x)] có cùng các điểm cực trị. Ví dụ 1.15. Tìm cực trị của hàm số: y = 2 x x2+1 Lời giải. Hàm số đã cho xác định với mọi x ∈ R. Vì hàm y = log2 x là hàm đồng biến với x > 0, nên hàm y = 2 x x2+1 có cùng các điểm cực trị với hàm f(x) = log2 2 x x2+1 = x x2+1. Ta có f (x) = 1−x2 (x2+1)2 . f (x) = 0 ⇔ x = ±1 Lập bảng biến thiên của hàm f(x): x f (x) f(x) −∞ −1 1 +∞ − 0 + 0 − +∞+∞ −1 2−1 2 1 2 1 2 −∞−∞ Từ bảng biến thiên ta thấy, hàm f(x) đạt cực tiểu tại x = −1 và đạt cực đại tại x = 1. Do đó hàm y cũng đạt cực tiểu tại x = −1, giá trị cực tiểu của hàm y là y(−1) = 1√ 2 . Hàm y đạt cực đại tại x = 1, giá trị cực đại của hàm y là y(1) = √ 2. Bài toán 3. Cho hàm số ϕ(x) là hàm liên tục, đồng biến với mọi x ∈ R và f(X) là hàm số đạt cực tiểu tại X0 = ϕ(x0). Chứng minh rằng hàm f[ϕ(x)] cũng đạt cực tiểu tại x0. Chứng minh. Giả sử hàm f(X) đạt cực tiểu tại điểm X0 = ϕ(x0), tức là f(X) ≥ f(X0), ∀x ∈ (X0 − δ; X0 + δ) với δ > 0 13
  • 20. Chương 1. Cực trị hàm số Giả sử ϕ(x0 − α) = X0 − δ , ϕ(x0 + α) = X0 + δ, . Nên ϕ[f(x)] ≤ ϕ[f(x0)], ∀x ∈ D Như vậy hàm ϕ[f(x)] cũng đạt cực tiểu tại điểm x0 Chứng minh tương tự với trường hợp cực đại. Kết luận: hàm f(x) và ϕ[f(x)] có cùng các điểm cực trị. Ví dụ 1.16. Tìm cực trị của hàm số: y = sin ln x Lời giải. Vì hàm y = sin X là hàm tuần hoàn nên ta chỉ cần xét trong −π 2 ; π 2 Nhận thấy: hàm y = sin X đạt cực đại tại X = π 2 . Áp dụng kết quả bài toán 3 ta suy ra hàm y = sin ln x đạt cực đại tại ln x = π 2 ⇔ x = e π 2 . Tương tự: y = sin X đạt cực tiểu tại X = −π 2 . Suy ra hàm y = sin ln x đạt cực tiểu tại ln x = −π 2 ⇔ x = e−π 2 . 1.3.2 Bài tập tham khảo Bài tập 1.1. Tìm cực trị của hàm số y = (x − 1)e x2−5x+6 x−1 Lời giải. Hàm số dã cho xác định với mọi x = 1 Ta có y = e x2−5x+6 x−1 + x2 − 2x − 1 x − 1 e x2−5x+6 x−1 y = 0 ⇔ x2 − x − 2 x − 1 e x2−5x+6 x−1 = 0 ⇔ (x + 1)(x − 2) x − 1 e x2−5x+6 x−1 = 0 ⇔ x = −1 x = 2 Lập bảng biến thiên của hàm số y 14
  • 21. Chương 1. Cực trị hàm số x y y −∞ −1 1 2 +∞ − 0 + − 0 + 00 −2e−6 −2e−6 0 +∞ 11 +∞+∞ Từ bảng biến thiên ta suy ra: Giá trị cực tiểu của hàm số y(−1) = −2e−6 , y(2) = 1 Bài tập 1.2. Tìm cực trị của các hàm số sau: a) y = |x| e−|x−1| b) y = |x| 1√ 2 |1 − x|1− 1√ 2 c) y = 1 2(cos x + |cos x|) Lời giải. a) Xét các trường hợp: • Với x < 0, ta có y = −x e1−x y = −1 − x e1−x ; y = 0 ⇔ x = −1 Lập bảng biến thiên của hàm y trong (−∞; 0) x y y −∞ −1 0 + 0 − −∞−∞ 1 e2 1 e2 00 Do đó hàm số đạt cực đại tại x = −1, giá trị cực đại của hàm số là y(−1) = 1 e2 . • Với 0 ≤ x < 1, ta có y = x e1−x ; y = x + 1 e1−x 15
  • 22. Chương 1. Cực trị hàm số Nhận thấy y > 0, ∀x [0; 1) Suy ra y ≥ y(0), ∀x [0; 1) Vậy hàm số đạt cực tiểu tại x = 0, giá trị cực tiểu của hàm số là y(0) = 0. • Với x ≥ 1, ta có y = x ex−1 y = 1 − x ex−1 ; y = 0 ⇔ x = 1 Lập bảng biến thiên của hàm y trong (1; +∞) x y y 1 +∞ 0 + 11 +∞+∞ Do đó hàm số đạt cực đại tại x = 1, giá trị cực đại của hàm số là y(1) = 1. b) Chứng minh tương tự phần a), ta được kết quả • Tại x = 0 hàm số đạt giá trị cực tiểu y = 0 • Tại x = 1 hàm số đạt giá trị cực tiểu y = 1 • Tại x = 1 + 1√ 2 hàm số đạt giá trị cực tiểu y = (1 + 1 √ 2 ) 1√ 2 ( 1 √ 2 )1− 1√ 2 c) Giá trị cực tiểu của hàm số là y = 0, tại điểm x = π 2 . Giá trị cực đại của hàm số là y = 1, tại điểm x = 0 Bài tập 1.3. Cho f(x) = xn + (a − x)n với a > 0 và n ∈ Z, n ≥ 2 Tìm cực trị của hàm f(x). Lời giải. Hàm số dã cho xác định với mọi x ∈ R Ta có f (x) = nxn−1 − n(a − x)n−1 = n xn−1 − (a − x)n−1 16
  • 23. Chương 1. Cực trị hàm số f (x) = 0 ⇔ xn−1 = (a − x)n−1 (∗) • Nếu n chẵn, khi đó (∗) ⇔ x = a − x ⇔ x = a 2 • Nếu n lẻ, khi đó (∗) ⇔ x = a − x x = x − a ⇔ 2x = a a = 0 ⇔ x = a 2 Từ đó suy ra f (x) = 0 ⇔ x = a 2. Lại có f (x) =n(n − 1) xn−2 + (a − x)n−2 ⇒ f ( a 2 ) =n(n − 1) ( a 2 )n−2 + ( a 2 )n−2 =2n(n − 1)( a 2 )n−2 > 0, a > 0 2 ≤ n ∈ Z . Do f (a 2) = 0 và f (a 2) > 0 nên giá trị cực tiểu của hàm số f(a 2) = 2(a 2)n . Bài tập 1.4. Cho m, n là các số tự nhiên. Tìm cực trị của hàm số y = xm (1 − x)n Gợi ý: • Tại x = 0: khi m chẵn, hàm số đạt cực tiểu. Giá trị cực tiểu của hàm số là: y(0) = 0 Khi m lẻ, hàm số không có cực trị. • Tại x = 1: khi n chẵn, hàm số đạt cực tiểu. Giá trị cực tiểu của hàm số là: y(1) = 0 Khi n lẻ, hàm số không có cực trị. • Tại x = m m+n hàm số đạt cực đại, giá trị cực đại của hàm số là: y( m m + n ) = mm nn (m + n)m+n Bài tập 1.5. Tìm cực trị của các hàm số sau: a)f(x) = e−π2 x2 ( √ 2 + sin π2 x2 ) với x = 0 0 với x = 0 17
  • 24. Chương 1. Cực trị hàm số b)g(x) = e− 1 |x| ( √ 2 + cos 1 x) với x = 0 0 với x = 0 Lời giải. a) Hàm số đã cho xác định và liên tục trên R, vì lim x→0 f(x) = 0 Do f (x) = 2π2 x3 e−π2 x2 ( √ 2 + sin π2 x2 − cos π2 x2 ) Nên f (x) > 0 với x > 0 f (x) < 0 với x < 0 Lại có f(x) − f(0) = e−π2 x2 ( √ 2 + sin π2 x2 ) > 0, ∀x = 0 Từ đó suy ra giá trị cực tiểu của hàm số là f(0) = 0 b) Chứng minh tương tự phần a), ta có: Giá trị cực tiểu của hàm g(x) là g(0) = 0. 18
  • 25. Chương 2 Giá trị lớn nhất, giá trị nhỏ nhất 2.1 Các khái niệm cơ bản Trong phần này ta sẽ nêu ra một số khái niệm cơ bản về giá trị lớn nhất và giá trị nhỏ nhất của hàm số một biến, hàm số nhiều biến và của một tập hợp. 2.1.1 Giá trị lớn nhất, giá trị nhỏ nhất của hàm số Định nghĩa 2.1. Cho hàm số y = f(x) xác định trên D ⊂ R. Khi đó: • Số M được gọi là giá trị lớn nhất của hàm số f(x) trong miền D nếu đồng thời xảy ra hai điều kiện: 1) f(x) ≤ M với mọi x ∈ D; 2) Tồn tại x1 ∈ D sao cho f(x1) = M. Ký hiệu: M = max x∈D f(x). • Số m được gọi là giá trị nhỏ nhất của hàm số f(x) trong miền D nếu đồng thời xảy ra hai điều kiện: 1) f(x) ≥ m với mọi x ∈ D; 2) Tồn tại x2 ∈ D sao cho f(x2) = m. Ký hiệu: m = min x∈D f(x). Định nghĩa 2.2. Cho hàm số n biến F = f(x1, x2, ..., xn) : D ⊂ Rn → R 19
  • 26. Chương 2. Giá trị lớn nhất, giá trị nhỏ nhất Khi đó: • Số M được gọi là giá trị lớn nhất của hàm số F trong miền D, ký hiệu M = max D F nếu đồng thời xảy ra hai điều kiện: 1) f(x1, x2, ..., xn) ≤ M với mọi (x1, x2, ..., xn) ∈ D; 2) Tồn tại (x0 1, x0 2, ..., x0 n) ∈ D sao cho f(x0 1, x0 2, ..., x0 n) = M. • Số m được gọi là giá trị nhỏ nhất của hàm số F trong miền D, ký hiệu m = min D F nếu đồng thời xảy ra hai điều kiện: 1) f(x1, x2, ..., xn) ≥ m với mọi (x1, x2, ..., xn) ∈ D; 2) Tồn tại (x1 1, x1 2, ..., x1 n) ∈ D sao cho f(x1 1, x1 2, ..., x1 n) = m. 2.1.2 Giá trị lớn nhất, giá trị nhỏ nhất của một tập hợp Trong một số bài toán việc tìm giá trị lớn nhất, nhỏ nhất của một hàm số được đưa về tìm giá trị lớn nhất, giá trị nhỏ nhất của một tập hợp số. Định nghĩa 2.3. Cho tập A ⊂ R • Một số thực α được gọi là giá trị nhỏ nhất của tập A nếu nó đồng thời thỏa mãn hai điều kiện: 1) α ≤ a với mọi a ∈ A; 2) α ∈ A. Khi đó ký hiệu: α = min A Ta phân tích rõ thành hai điều kiện để khi chứng minh α = minA thì phải chứng minh đủ cả hai điều kiện đó. • Một số thực β được gọi là giá trị lớn nhất của tập A nếu nó đồng thời thỏa mãn hai điều kiện: 1) β ≥ a với mọi a ∈ A; 2) β ∈ A. Khi đó ký hiệu: β = max A 20
  • 27. Chương 2. Giá trị lớn nhất, giá trị nhỏ nhất Ví dụ 2.1. Cho A = {c ∈ R|c = (3 − x)(4 − y)(2x + 3y), 0 ≤ x ≤ 3, 0 ≤ y ≤ 4} Tìm max A. Lời giải. Do 0 ≤ x ≤ 3 0 ≤ y ≤ 4 ⇔ 0 ≤ 3 − x ≤ 3 0 ≤ 4 − y ≤ 4 Áp dụng bất đẳng thức giữa trung bình cộng và trung bình nhân,ta có c = 1 6 (6−2x)(12−3y)(2x+3y) ≤ 1 6 ( 6 − 2x + 12 − 3y + 2x + 3y 3 )3 = 36 Dấu bằng xảy ra khi và chỉ khi 6 − 2x = 12 − 3y 6 − 2x = 2x + 3y ⇔ 2x − 3y = −6 4x + 3y = 6 ⇔ x = 0 y = 2 Suy ra c ≤ 36, ∀c ∈ A c1 = (3 − 0)(4 − 2)(0 + 6) = 36 ∈ A Vậy max A = 36 Ví dụ 2.2. Cho A = {a0, a1, ..., a12}, trong đó các aj là hệ số trong khai triển (1 + 2x)12 = a0 + a1x + a2x2 + ... + a12x12 Tìm max A, min A. Lời giải. Ta biết rằng theo khai triển nhị thức Newton ak = Ck 122k với 0 ≤ k ≤ 12 Giải bất phương trình ak ≤ ak+1 ⇔ Ck 122k ≤ Ck+1 12 2k+1 ⇔ k ≤ 7 Như vậy: ak ≤ a8, ∀k ≤ 7 ak ≥ a8, ∀k = 9, 10, 11, 12 Vậy max A = a8 = C8 1228 . Mặt khác a0 = 1, a12 = 212 . Do đó min A = a0 = 1. 21
  • 28. Chương 2. Giá trị lớn nhất, giá trị nhỏ nhất 2.1.3 Một số tính chất của giá trị lớn nhất, giá trị nhỏ nhất Tính chất 1. Nếu hàm số y = f(x) có đạo hàm và có hữu hạn các điểm dừng x1, x2, ..., xn trong (a; b) thì max [a;b] = max {f(x1); f(x2); ...; f(xn); f(a); f(b)} min [a;b] = min {f(x1); f(x2); ...; f(xn); f(a); f(b)} Tính chất 2. Cho hàm số f(x) xác định trên miền D (i) Nếu hàm f(x) đồng biến trong [a; b] ⊆ D thì: min x∈[a;b] f(x) = f(a) max x∈[a;b] f(x) = f(b) (ii) Nếu hàm f(x) nghịch biến trong [a; b] ⊆ D thì: min x∈[a;b] f(x) = f(b) max x∈[a;b] f(x) = f(a) Tính chất 3. Giả sử hàm số f(x) xác định trên D và A, B là các tập con của D, trong đó A ⊆ B. Giả sử tồn tại max x∈A f(x), max x∈B f(x), min x∈A f(x), min x∈B f(x). Khi đó ta có: max x∈A f(x) ≤ max x∈B f(x) min x∈A f(x) ≥ min x∈B f(x) Tính chất 4. Giả sử hàm số f(x) xác định trên miền D và D = D1 ∪D2. Nếu tồn tại các giá trị: m1 = min x∈D1 f(x); m2 = min x∈D2 f(x) M1 = max x∈D1 f(x); M2 = max x∈D2 f(x) Thì: min x∈D f(x) = min {m1; m2} max x∈D f(x) = max {M1; M2} 22
  • 29. Chương 2. Giá trị lớn nhất, giá trị nhỏ nhất Tính chất 5. Giả sử hàm số f(x) xác định trên D và tồn tại min x∈D f(x), max x∈D f(x). Khi đó ta có: max x∈D f(x) = − min x∈D (−f(x)) min x∈D f(x) = − max x∈D (−f(x)) Nếu f(x) > 0 với mọi x ∈ D thì: max x∈D ( 1 f(x) ) = 1 min x∈D f(x) min x∈D ( 1 f(x) ) = 1 max x∈D f(x) Tính chất 6. Giả sử hàm số f(x) xác định trên miền D. Khi đó với mọi n nguyên dương, ta có: max x∈D f(x) = 2n+1 max x∈D (f2n+1(x)) min x∈D f(x) = 2n+1 min x∈D (f2n+1(x)) Nếu f(x) ≥ 0, ∀x ∈ D. Khi đó với mọi n nguyên dương, ta có: max x∈D f(x) = 2n max x∈D (f2n(x)) min x∈D f(x) = 2n min x∈D (f2n(x)) Tính chất 7. Giả sử f(x) là hàm số xác định trên D và tồn tại max x∈D f(x), min x∈D f(x). Khi đó ta có: max x∈D |f(x)| = max max x∈D f(x) ; min x∈D f(x) min x∈D f(x) =    0 nếu f(x) = 0 min max x∈D f(x) ; min x∈D f(x) nếu f(x) = 0 Tính chất 8. Cho các hàm số f1(x), f2(x), ..., fn(x) cùng xác định trên miền D. Đặt f(x) = f1(x) + f2(x) + ... + fn(x). Giả sử tồn tại max x∈D f(x), min x∈D f(x) 23
  • 30. Chương 2. Giá trị lớn nhất, giá trị nhỏ nhất và max x∈D fi(x), min x∈D fi(x) với mọi i = 1, n. Khi đó ta có: max x∈D f(x) ≤ max x∈D f1(x) + max x∈D f2(x) + ... + max x∈D fn(x) (1) min x∈D f(x) ≥ min x∈D f1(x) + min x∈D f2(x) + ... + min x∈D fn(x) (2) Dấu bằng trong (1) xảy ra khi và chỉ khi tồn tại x0 ∈ D sao cho max x∈D fi(x) = fi(x0), ∀i = 1, n. Dấu bằng trong (2) xảy ra khi và chỉ khi tồn tại x1 ∈ D sao cho min x∈D fi(x) = fi(x1), ∀i = 1, n. 2.1.4 Một số định lý về giá trị lớn nhất, giá trị nhỏ nhất Định lý 2.1. (Điều kiện đủ để tồn tại giá trị lớn nhất, giá trị nhỏ nhất) Nếu hàm số f(x) liên tục trên [a; b] thì f(x) đạt giá trị lớn nhất, giá trị nhỏ nhất trên đoạn đó. Định lý 2.2. Cho hàm số f(x) có đạo hàm cấp một liên tục trên [a; b], có đạo hàm cấp hai tại mọi điểm x ∈ (a; b) sao cho f (x) ≥ 0 với mọi x ∈ (a; b). Khi đó, với mọi x, y ∈ [a; b] ta luôn có: f(x) ≥ f(y) + f (y)(x − y) (2.2) Hay f(x) = max a≤y≤b (f(y) + f (y)(x − y)) Chứng minh. 1) Xét x = y thì dấu đẳng thức của (2.2) xảy ra. 2) Xét x > y thì (2.2) ⇔ f(x) − f(y) x − y ≥ f (y) (1) Theo nguyên lý Lagrange suy ra: ∃y0 ∈ (x; y) sao cho f(x) − f(y) x − y = f (y0) Vậy (1) có dạng f (y0) ≥ f (y) với y < y0 < x (2) Ta thấy (2) là hiển nhiên vì f (x) ≥ 0 nên f (x) là hàm đơn điệu tăng. 24
  • 31. Chương 2. Giá trị lớn nhất, giá trị nhỏ nhất 3) Xét x < y thì (2.2) ⇔ f(x) − f(y) x − y ≤ f (y) (3) Theo nguyên lý Lagrange suy ra: ∃y1 ∈ (x; y) sao cho f(x) − f(y) x − y = f (y1) Vậy (3) có dạng f (y1) ≤ f (y) với y < y1 < x (4) Ta thấy (4) là hiển nhiên vì f (x) ≥ 0 nên f (x) là hàm đơn điệu tăng. 2 Khi f (x) ≤ 0, chứng minh tương tự ta có định lý sau đây: Định lý 2.3. Cho hàm số f(x) có đạo hàm cấp một liên tục trên [a; b], có đạo hàm cấp hai tại mọi điểm x ∈ (a; b) sao cho f (x) ≤ 0 với mọi x ∈ (a; b). Khi đó, với mọi x, y ∈ [a; b] ta luôn có: f(x) ≤ f(y) + f (y)(x − y) (2.3) Hay f(x) = min a≤y≤b (f(y) + f (y)(x − y)) Ví dụ 2.3. Cho tam giác ABC bất kỳ.Tìm giá trị lớn nhất của các biểu thức M = cos A 2 + cos B 2 + cos C 2 Lời giải. Xét hàm số f(x) = cos x, x ∈ (0; π 2 ) có f (x) = − cos x < 0, ∀x ∈ (0; π 2 ). Theo định lý (2.3) với x = A 2 và y = π 6 ta được cos A 2 ≤ cos π 6 − sin π 6 ( A 2 − π 6 ) Dấu đẳng thức xảy ra khi A 2 = π 6 Chứng minh tương tự, ta có: cos B 2 ≤ cos π 6 − sin π 6 ( B 2 − π 6 ) cos C 2 ≤ cos π 6 − sin π 6 ( C 2 − π 6 ) 25
  • 32. Chương 2. Giá trị lớn nhất, giá trị nhỏ nhất Cộng các bất đẳng thức trên vế theo vế M ≤ 3 cos π 6 − sin π 6 ( A + B + C 2 − π 2 ) Dấu đẳng thức xảy ra khi A 2 = B 2 = C 2 = π 6 Vậy max M = 3 √ 3 2 khi ABC là tam giác đều. 2.2 Các phương pháp tìm giá trị lớn nhất, giá trị nhỏ nhất Để tìm giá trị lớn nhất, giá trị nhỏ nhất của hàm số ta có thế sử dụng nhiều phương pháp. Trong thực tế, chúng ta có thể bắt gặp những bài toán chỉ có thể giải bằng một phương pháp, nhưng cũng có những bài toán có thể giải bằng nhiều phương pháp hay những bài toán cần phối hợp nhiều phương pháp mới giải quyết được.Điều đó nói lên tính đa dạng, phong phú của các phương pháp giải toán nói chung và phương pháp tìm giá trị lớn nhất, giá trị nhỏ nhất nói riêng. Vì vậy, việc nắm vững các phương pháp sẽ giúp người làm toán giải quyết được nhiều dạng bài tập. Trong phần này, luận văn sẽ trình bày một số phương pháp tìm giá trị lớn nhất, giá trị nhỏ nhất và các ví dụ áp dụng. 2.2.1 Phương pháp sử dụng đạo hàm a. Cơ sở lý thuyết. Dựa vào tính chất 1 của giá trị lớn nhất, giá trị nhỏ nhất, ta xây dựng quy tắc tìm giá trị lớn nhất, giá trị nhỏ nhất của hàm số sử dụng đạo hàm như sau: Quy tắc: 1) Tìm các điểm dừng của hàm số đã cho; Điểm dừng là điểm thuộc tập xác định của hàm số mà tại đó đạo hàm của hàm số bằng không hoặc không xác định. 2) Tính các giá trị của hàm số tại các điểm dừng và hai đầu mút; 3) So sánh các giá trị tìm được, rồi kết luận giá trị lớn nhất, giá trị nhỏ nhất của hàm số. 26
  • 33. Chương 2. Giá trị lớn nhất, giá trị nhỏ nhất b. Các ví dụ áp dụng. Ví dụ 2.4. (Đề thi tuyển sinh Đại học và Cao đẳng khối B năm 2004) Tìm giá trị lớn nhất, giá trị nhỏ nhất của hàm số: f(x) = ln2 (x) x trên 1; e3 Lời giải. Nhận thấy f(x) xác định và liên tục với mọi x ∈ 1; e3 . Ta có f (x) = lnx(2 − lnx) x2 f (x) = 0 ⇔ x = e2 Lập bảng biến thiên của hàm f(x): x f (x) f 1 e2 e3 + 0 − 00 4 e2 4 e2 9 e3 9 e3 Từ bảng biến thiên ta suy ra: max x∈[1;e3] f(x) = f(e2 ) = 4 e2 min x∈[1;e3] f(x) = f(1) = 0 Ví dụ 2.5. Tìm giá trị lớn nhất, giá trị nhỏ nhất (nếu có) của hàm số: y = 1 cos x + 1 sin x với x ∈ 0; π 2 Lời giải. Nhận thấy: y = 1 cos x + 1 sin x = sin x + cos x sin x cos x Đặt sin x + cos x = t. Suy ra t ∈ 1; √ 2 . Khi đó hàm số đã cho có dạng: g(t) = 2t t2 − 1 27
  • 34. Chương 2. Giá trị lớn nhất, giá trị nhỏ nhất Nhận xét: hàm g(t) xác định và liên tục với mọi t ∈ 1; √ 2 . Ta có g (t) = − 2 + 2t2 (t2 − 1)2 < 0, ∀t ∈ 1; √ 2 Lập bảng biến thiên của hàm g(t): x g (x) g(x) 1 √ 2 − +∞ 2 √ 22 √ 2 Từ bảng biến thiên ta suy ra min t∈(1; √ 2] g(t) = 2 √ 2; và không tồn tại max t∈(1; √ 2] g(t). Vậy không tồn tại max x∈(0;π 2 ) f(x); min x∈(0;π 2 ) f(x) = 2 √ 2 ⇔ x = π 2 . Ví dụ 2.6. Cho x, y là các số thực không âm, thỏa mãn x + y = 1. Tìm giá trị lớn nhất và nhỏ nhất của biểu thức: M = 22x + 2y Lời giải. Từ giả thiết x + y = 1, ta suy ra y = 1 − x Khi đó biểu thức M có dạng: M = 22x + 21−x = 22x + 2 2x Đặt 2x = t ⇒ 1 ≤ t ≤ 2 Ta có: M = f(t) = t2 + 2 t , với 1 ≤ t ≤ 2 f (t) = 2t − 2 t2 = 2t3 −2 t2 xác định và liên tục trên [1; 2] f (t) = 0 ⇔ t = 1 Lập bảng biến thiên của hàm f(t) trong [1; 2] 28
  • 35. Chương 2. Giá trị lớn nhất, giá trị nhỏ nhất t f (t) f(t) 1 2 0 + 33 55 Từ bảng biến thiên ta thấy max 1≤t≤2 f(t) = 5 ; min 1≤t≤2 f(t) = 3 Kết luận: max M = 5 ⇔ x + y = 1 2x = 2 ⇔ x = 1 y = 0 min M = 3 ⇔ x + y = 1 2x = 1 ⇔ x = 0 y = 1 c. Bài tập tham khảo. Bài tập 2.1. (Đề thi học sinh giỏi Toán Quốc gia năm 1991) Chứng minh rằng sin x + 2 cos x 2 cos x + 2 sin x 2 ≥ 2 + √ 2 2 với 0 ≤ x ≤ π 2 Lời giải. Xét sự biến thiên của hàm số f(x) = sin x + 2 cos x 2 cos x + 2 sin x 2 trong 0; π 2 Nhận thấy với x ∈ 0; π 2 thì cos x ≥ 0 và sin x 2 ≥ 0 nên cos x+2 sin x 2 > 0 Đạo hàm f (x) có tử thức (cos x − sin x 2 )(cos x + 2 sin x 2 ) − (sin x + 2 cos x 2 )(− sin x + cos x 2 ) = sin x. cos x 2 + sin x 2 . cos x − 1 = sin(x + x 2 ) − 1 ≤ 0 29
  • 36. Chương 2. Giá trị lớn nhất, giá trị nhỏ nhất Vậy hàm số f(x) nghịch biến trong 0; π 2 Do đó f(x) ≥ f(π 2 ) = 2+ √ 2 2 Bài tập 2.2. Tìm giá trị lớn nhất, giá trị nhỏ nhất của hàm số y = 2 cos2 x + |cos x| + 1 |cos x| + 1 trong đoạn [0; 2] Gợi ý: Đặt |cos x| = t, t ∈ [0; 1] Đáp số: max y = 2 ⇔ x = kπ min y = 1 ⇔ x = π 2 + kπ (k ∈ Z) Bài tập 2.3. Tìm giá trị lớn nhất, giá trị nhỏ nhất của hàm số y = x3 + 3x2 − 72x + 90 trong đoạn [−5; 5] Gợi ý: Đặt f(x) = x3 + 3x2 − 72x + 90 Lập bảng biến thiên, tìm giá trị lớn nhất và nhỏ nhất của hàm f(x) rồi dựa vào tính chất 7 để suy ra giá trị lớn nhất và nhỏ nhất của hàm y. Đáp số: max y = f(−5) = 400 min y = 0 Bài tập 2.4. Cho x; y là các số thực không âm, thỏa mãn x + y = 1. Tìm giá trị lớn nhất và nhỏ nhất của biểu thức: P = x y + 1 + y x + 1 Đáp số: min P = 2 3 ⇔ x = 1 2 y = 1 2 max P = 1 ⇔ x = 0; y = 1 x = 1; y = 2 Bài tập 2.5. Cho xyz = 1 và 1 4 ≤ x ≤ 1; y, z ≥ 1. Tìm giá trị nhỏ nhất của biểu thức P = 1 1 + x + 1 1 + y + 1 1 + z 30
  • 37. Chương 2. Giá trị lớn nhất, giá trị nhỏ nhất Gợi ý: Đặt t = √ xy; 1 ≤ t ≤ 2 Khi đó P = f(t) = 2t (t2 + 1)2 − 2 (1 + t)2 Lập bảng biến thiên, tìm giá trị nhỏ nhất của hàm f(t) ta được min P = min 1≤t≤2 f(t) = 22 15 khi x = 1 4 ; y = z = 2. 2.2.2 Phương pháp tập giá trị a. Cơ sở lý thuyết. Định nghĩa 2.4. Cho hàm số y = f(x) xác định trên tập D ⊂ R. Khi x biến thiên trên tập D thì tương ứng y biến thiên trên tập T ⊂ R. Khi đó tập T được gọi là tập giá trị của hàm số tương ứng với tập xác định D của nó. Nhận xét: Giả sử hàm số f(x) xác định và liên tục trong miền D và phương trình f(x) = y có nghiệm x ∈ D khi và chỉ khi y ∈ T. Khi đó: Nếu T = [a; b] thì min x∈D f(x) = a; max x∈D f(x) = b Nếu T = (a; b] thì max x∈D = b Nếu T = [a; b) thì min x∈D = a Sau đây ta sẽ xét một số bài toán tìm giá trị lớn nhất, nhỏ nhất của hàm số sử dụng phương pháp tập giá trị. b. Các ví dụ áp dụng. Ví dụ 2.7. Tìm giá trị lớn nhất, giá trị nhỏ nhất của hàm số: f(x) = x2 − 2x + 2 x2 + 2x + 2 Lời giải. Vì x2 + 2x + 2 > 0 với mọi x ∈ R nên hàm số đã cho xác định và liên tục với mọi x ∈ R. Xét phương trình ẩn x, tham số y x2 − 2x + 2 x2 + 2x + 2 = y (2.7) 31
  • 38. Chương 2. Giá trị lớn nhất, giá trị nhỏ nhất Ta có (2.7) ⇔ (y − 1)x2 + 2(y + 1)x + 2(y − 1) = 0 1) Xét y = 1 ⇒ x = 0 2) Xét y = 0. Để phương trình có nghiệm: ∆ = (y + 1)2 − 2(y − 1)2 ≥ 0 ⇔ −y2 + 6y − 1 ≥ 0 ⇔ 3 − 2 √ 2 ≤ y ≤ 3 + 2 √ 2 So sánh với giá trị y = 1 ta được max f(x) = 3 + 2 √ 2 ⇔ x = − √ 2 min f(x) = 3 − 2 √ 2 ⇔ x = √ 2 Ví dụ 2.8. Tìm giá trị lớn nhất, giá trị nhỏ nhất của hàm số: y = sin 2x + 3 cos2 x − sin2 x Lời giải. Viết hàm số đã cho dưới dạng: y = − sin2 x + 2 sin x cos x + 3 cos2 x sin2 x + cos2 x 1) Xét cos x = 0 ⇒ y = −1 2) Xét cos x = 0. Viết y dưới dạng: y = −t2 + 2t + 3 t2 + 1 , với t = tan x (2.8) Ta có (2.8) ⇔ (y + 1)t2 − 2t + y − 3 = 0 Cần xác định y để phương trình trên có nghiệm. Ta thấy ∆ = 1 − (y + 1)(y − 3) ≥ 0 ⇔ −y2 + 2y + 4 ≥ 0 ⇔ 1 − √ 5 ≤ y ≤ 1 + √ 5 32
  • 39. Chương 2. Giá trị lớn nhất, giá trị nhỏ nhất Vậy max y = 1 + √ 5 ⇔ x = arctan 1 2 + √ 5 min y = 1 − √ 5 ⇔ x = arctan 1 2 − √ 5 Ví dụ 2.9. Cho 2x2 + y2 + xy ≥ 1. Tìm giá trị nhỏ nhất của biểu thức: M = x2 + y2 Lời giải. Đặt 2x2 + y2 + xy = a, a ≥ 1 Khi đó: M a = x2 + y2 2x2 + y2 + xy 1) Nếu y = 0thì M a = 1 2 2) Nếu y = 0 thì M a = t2 + 1 2t2 + t + 1 , t = x y Cần xác định giá trị của M a sao cho phương trình M a = t2 + 1 2t2 + t + 1 có nghiệm. ⇔ (2 M a − 1)t2 + M a t + M a − 1 = 0 có nghiệm ⇔ ∆ = ( M a )2 − 4(2 M a − 1)( M a − 1) ≥ 0 ⇔ M ≥ 6 − 2 √ 2 7 a ≥ 6 − 2 √ 2 7 = M0 Vậy min M = 6−2 √ 2 7 khi x y = − M0 2(2M0−1) 2x2 + y2 + xy = 1 ⇔    x = M1y y = ± √ 2(1−2M0) √ 2−7M0+7M2 0 với M1 = − M0 2(2M0 − 1) 33
  • 40. Chương 2. Giá trị lớn nhất, giá trị nhỏ nhất c.Bài tập tham khảo. Bài tập 2.6. Tìm giá trị lớn nhất, giá trị nhỏ nhất của hàm số f(x) = 2 sin x − 3 cos x + 1 sin x + 2 cos x − 3 Đáp số: max f(x) = 2 ⇔ x = 2π min f(x) = − 3 2 ⇔ x = π 2 Bài tập 2.7. Tìm giá trị lớn nhất, giá trị nhỏ nhất của hàm số f(x) = x2 cos α − 2x + cos α x2 − 2x cos α + 1 (α là một hằng số thực) Đáp số: max f(x) = 1 ⇔ x = −1 min f(x) = −1 ⇔ x = 1 Bài tập 2.8. Tìm giá trị nhỏ nhất của hàm số f(x) = x + x2 + 1 x với x > 0 Lời giải. Hàm số đã cho xác định và liên tục với mọi x > 0. Xét phương trình ẩn x, tham số m x + x2 + 1 x = m (∗) Phương trình (∗) có nghiệm x > 0 khi và chỉ khi 0 < x ≤ m x2 + 1 x = (m − x)2 ⇔ 0 < x ≤ m 2mx2 − m2 x + 1 = 0 (∗∗) Ta thấy (∗∗) có nghiệm khi và chỉ khi m4 − 8m ≥ 0 ⇔ m ≥ 2 34
  • 41. Chương 2. Giá trị lớn nhất, giá trị nhỏ nhất Lại có: P = 1 2m > 0; S = m2 2m = m 2 > 0 Do đó (∗∗) có hai nghiệm dương x1 ≥ x2. Vì x1 + x2 = m 2 nên m > x1 ≥ x2 Vậy m ≥ 2, phương trình (∗) có nghiệm x > 0 Suy ra f(x) ≥ 2, ∀x > 0 Kết luận: min x>0 f(x) = 2 ⇔ x = 1 2. Bài tập 2.9. (Đề thi tuyển sinh Đại học, Cao đẳng khối B năm 2008) Giả sử x, y là hai số thực thỏa mãn x2 + y2 = 1. Tìm giá trị lớn nhất, nhỏ nhất của biểu thức P = 2(x2 + 6xy) 1 + 2xy + 2y2 Lời giải. Do x2 + y2 = 1 nên ta có P = 2(x2 + 6xy) x2 + 2xy + 3y2 • Nếu y = 0 ⇒ x = 1 Khi đóP = 2 • Nếu y = 0, chia cả tử thức và mẫu thức của P cho y2 = 0 ta được P = 2 (x y )2 + 6x y (x y )2 + 2x y + 3 Đặt x y = t, t ∈ R Bài toán quy về tìm giá trị lớn nhất, nhỏ nhất của hàm số f(t) = 2t2 + 12t t2 + 2t + 3 , t ∈ R Xét phương trình ẩn t, tham số một 2t2 + 12t t2 + 2t + 3 = m (1) Vì t2 + 2t + 3 > 0, ∀t nên (1) ⇔ (m − 2)t2 + 2(m − 6)t + 3m = 0 (2) 35
  • 42. Chương 2. Giá trị lớn nhất, giá trị nhỏ nhất +) Nếu m = 2: (2) ⇔ t = 3 4 +) Nếu m = 2: (2) có nghiệm khi và chỉ khi m2 + 3m − 18 ≤ 0 ⇔ −6 ≤ m ≤ 3 Vậy max f(t) = 3 ⇔ t = 3 min f(t) = −6 ⇔ t = − 3 2 Kết luận: max P = 3 ⇔ x = 3√ 10 ; y = 1√ 10 x = − 3√ 10 ; y = − 1√ 10 min P = −6 ⇔ x = − 3√ 13 ; y = 2√ 13 x = 3√ 13 ; y = − 2√ 13 Bài tập 2.10. Tìm giá trị lớn nhất, nhỏ nhất của hàm số: y = 3 √ x + 3 + 4 √ 1 − x + 1 4 √ x + 3 + 3 √ 1 − x + 1 Gợi ý: Từ điều kiện − 3 ≤ x ≤ 1 và do ( √ x + 3)2 + ( √ 1 − x)2 = 4, đặt √ x + 3 = 2.2t 1+t2 √ 1 − x = 2(1−t2 ) 1+t2 0 ≤ t ≤ 1 Khi đó y = −7t2 + 12t + 9 −5t2 + 16t + 7 Đáp số: max y = 9 7 ⇔ x = −3 min y = 7 9 ⇔ x = 1 36
  • 43. Chương 2. Giá trị lớn nhất, giá trị nhỏ nhất 2.2.3 Phương pháp lượng giác Một số bài toán tìm giá trị lớn nhất, nhỏ nhất có thể giải bằng phương pháp lượng giác. Bằng các phép biến đổi lượng giác, đánh giá lượng giác ta đưa biểu thức và điều kiện của bài toán về dạng lượng giác. a.Cơ sở lý thuyết. Định lý 2.4. Nếu đại lượng x biến thiên trên miền D thì luôn đặt được x = ϕ(t) với t ∈ D1. Trong đó: ϕ(t) là một hàm số lượng giác nào đó; miền D1 được chọn sao cho ánh xạ ϕ : D1 → D là song ánh. Quy tắc. - Đặt x = ϕ(t). Chuyển điều kiện x ∈ D về điều kiện tương đương t ∈ D1; - Chuyển bài toán đại số đã cho về bài toán lượng giác mới; - Giải bài toán lượng giác mới thu được; - Kết luận cho bài toán đại số ban đầu. b.Ví dụ áp dụng. Ví dụ 2.10. Tìm giá trị lớn nhất, giá trị nhỏ nhất của hàm số y = 1 + x4 (1 + x2)2 Lời giải. Hàm số đã cho xác định với mọi x ∈ R. Đặt x = tan t với t ∈ (−π 2 ; π 2 ). Khi đó: y = 1 + tan4 t (1 + tan2 t)2 = sin4 t + cos4 t = 1 − 1 2 sin2 2t Suy ra 1 2 ≤ y ≤ 1 Vậy min y = 1 2 ⇔ x = 1 max y = 1 ⇔ x = 0 37
  • 44. Chương 2. Giá trị lớn nhất, giá trị nhỏ nhất Ví dụ 2.11. (Đề thi tuyển sinh Đại học, Cao đẳng khối D năm 2008) Cho x ≥ 0; y ≥ 0. Tìm giá trị lớn nhất, giá trị nhỏ nhất của biểu thức P = (x − y)(1 − xy) (1 + x)2(1 + y)2 Lời giải. Biến đổi biểu thức P về dạng P = x (1 + x)2 − y (1 + y)2 Do x ≥ 0; y ≥ 0. Đặt x = tan2 α, 0 ≤ α < π 2 y = tan2 β, 0 ≤ β < π 2 Khi đó P = tan2 α (1 + tan2 α)2 − tan2 β (1 + tan2 β)2 = sin2 α cos2 α − sin2 β cos2 β = 1 4 sin2 2α − 1 4 sin2 2β Suy ra −1 4 ≤ P ≤ 1 4, ∀α, β ∈ 0; π 2 Vậy max P = 1 4 ⇔ x = 1; y = 0 min P = − 1 4 ⇔ x = 0; y = 1 Ví dụ 2.12. Cho x2 + y2 = 1. Tìm giá trị lớn nhất, giá trị nhỏ nhất của biểu thức M = 16(x5 + y5 ) − 20(x3 + y3 ) + 5(x + y) Lời giải. 38
  • 45. Chương 2. Giá trị lớn nhất, giá trị nhỏ nhất Do x2 + y2 = 1, nên ta có thể đặt x = sin α y = cos α α ∈ [0; 2π] Ta có sin 5α = sin 3α cos 2α + sin 2α cos 3α = (3 sin α − 4 sin3 α)(1 − 2 sin2 α) + 2 sin α cos α(4 cos3 α − 3 cos α) = (3 sin α − 4 sin3 α)(1 − 2 sin2 α) + 2 sin α cos2 α(4 cos2 α − 3) = 16 sin5 α − 20 sin3 α + 5 sin α Biến đổi tương tự ta được cos 5α = 16 cos5 α − 20 cos3 α + 5 cos α Suy ra M = sin5 α + cos5 α = √ 2 cos(5α − π 4 ) Vậy max M = √ 2 ⇔ x = sin π 20 ; y = cos π 20 min M = − √ 2 ⇔ x = √ 2 2 ; y = √ 2 2 Nhận xét: Một số dấu hiệu nhận biết bài toán có thể giải bằng phương pháp lượng giác: 1) Tập biến thiên D của một biến số là tập giá trị của một hàm lượng giác nào đó. 2) Trong giả thiết có một bộ phận tương tự với công thức lượng giác nào đó: • Bộ phận 1 + x2 tương tự với 1 + tan2 t = 1 cos2 t. • Bộ phận 4x3 − 3x tương tự với 4 cos3 t − 3 cos t = cos 3t. • Bộ phận 2x2 − 1 tương tự với 2 cos2 t − 1 = cos t • Bộ phận 2x 1−x2 tương tự với tan 2t = 2 tan t 1−tan2 t • Bộ phận 2x 1+x2 tương tự với sin 2t = 2 tan t 1+tan2 t • Bộ phận x+y 1−xy tương tự với tan(α + β) = tan α+tan β 1−tan α tan β 39
  • 46. Chương 2. Giá trị lớn nhất, giá trị nhỏ nhất c.Bài tập tham khảo. Bài tập 2.11. Cho x, y, z ∈ [0; 1]. Tìm giá trị lớn nhất của biểu thức M = √ xyz + (1 − x)(1 − y)(1 − z) Lời giải. Theo giả thiết x, y, z ∈ [0; 1], do đó ta đặt x = sin2 A, y = sin2 B, z = sin2 C vớiA, B, C ∈ 0; π 2 Suy ra 0 ≤ sin A ≤ 1; 0 ≤ sin B ≤ 1; 0 ≤ sin C ≤ 1; 0 ≤ cos A ≤ 1; 0 ≤ cos B ≤ 1; 0 ≤ cos C ≤ 1 Khi đó M = sin2 A sin2 B sin2 C + √ cos2 A cos2 B cos2 C = sin A sin B sin C + cos A cos B cos C (1) Nhận thấy: +) sin A sin B sin C ≤ sin A sin B (2) Dấu bằng xảy ra ⇔ sin C = 1 sin A sin B = 0 ⇔   z = 1 x = 0 y = 0 (∗) +) cos A cos B cos C ≤ cos A cos B (3) Dấu bằng xảy ra ⇔ cos C = 1 cos A cos B = 0 ⇔   z = 0 x = 1 y = 1 (∗∗) Từ (1), (2), (3) suy ra M ≤ sin A sin B + cos A cos B ⇒ M ≤ cos(A − B) (4) Vì cos(A − B) ≤ 1 nên M ≤ 1 Kết luận min M = 1 ⇔ x = y = z = 1 x = y = z = 0 40
  • 47. Chương 2. Giá trị lớn nhất, giá trị nhỏ nhất Bài tập 2.12. Cho x, y, z là các số thực dương và thỏa mãn điều kiện x + y + z = xyz. Tìm giá trị lớn nhất của biểu thức P = 1 √ 1 + x2 + 1 1 + y2 + 1 √ 1 + z2 Lời giải. Từ giả thiết x + y + z = xyz ta suy ra 1 xy + 1 yz + 1 zx = 1 Do x, y, z là các số thực dương, nên đặt x = cot A; y = cot B; z = cot C với A, B, C ∈ 0; π 2 . Khi đó ta có: 1 xy + 1 yz + 1 zx = 1 ⇔ tan A tan B + tan B tan C + tan C tan A = 1 ⇔ tan A(tan B + tan C) = 1 − tan B tan C ⇔ tan A = 1 − tan B tan C tan B + tan C = cot(B + C) ⇔ A = B + C = π 2 Do đó P có dạng P = 1 √ 1 + cot2 A + 1 √ 1 + cot2 B + 1 √ 1 + cot2 C = sin A + sin B + sin C Mặt khác sin A + sin B + sin C 3 ≤ sin A + B + C 3 = sin π 6 = 1 2 Suy ra sin A + sin B + sin C ≤ 3 2 Vậy min P = 3 2 ⇔ A = B = C = π 6 ⇔ x = y = z = √ 3 41
  • 48. Chương 2. Giá trị lớn nhất, giá trị nhỏ nhất Bài tập 2.13. (Đề thi học sinh giỏi Toán Quốc gia năm 1999) Cho a, b, c > 0 và abc + a + c = b. Tìm giá trị lớn nhất của biểu thức M = 2 a2 + 1 − 2 b2 + 1 + 3 c2 + 1 Lời giải. Từ giả thiết ta có ac + a b + c b = 1 (1) Vì a, b, c > 0 nên đặt a = tan A 2 , 1 b = tan B 2 , c = tan C 2 với A, B, C ∈ 0; π 2 Khi đó (1) có dạng: tan A 2 tan C 2 + tan A 2 tan B 2 + tan B 2 tan C 2 = 1 Suy ra A 2 + B 2 + C 2 = π 2 hay A + B + C = π Ta có: 1 + a2 = 1 + tan2 A 2 = 1 cos2 A 2 1 + b2 = 1 + cot2 B 2 = 1 sin2 B 2 1 + c2 = 1 + tan2 C 2 = 1 cos2 C 2 Vậy P = 2 cos2 A 2 − 2 sin2 B 2 + 3 cos2 C 2 = 1 + cos A − (1 − cos B) + 3 cos2 C 2 = 2 cos A + B 2 cos A − B 2 + 3 cos2 C 2 = −3 sin2 C 2 + 2 sin C 2 cos A − B 2 + 3 = 3 − 3(sin2 C 2 − 2 3 sin C 2 cos A − B 2 ) = 3 − 3 (sin C 2 − 1 3 sin C 2 cos A − B 2 )2 − 1 9 cos2 A − B 2 = 3 + 1 3 cos2 A − B 2 − 3(sin C 2 − 1 3 cos A − B 2 )2 ≤ 10 3 42
  • 49. Chương 2. Giá trị lớn nhất, giá trị nhỏ nhất Dấu bằng xảy ra ⇔ cos A−B 2 = 1 sin C 2 = 1 3 Kết luận max P = 10 3 ⇔ x = 1√ 2 ; y = √ 2; z = 1 2 √ 2 . Bài tập 2.14. Cho |a| ≥ 1. Tìm giá trị lớn nhất và giá trị nhỏ nhất của biểu thức P = 5 − 12 √ a2 − 1 a2 Lời giải. Đặt x = 1 cos α với α ∈ 0; π 2 ∪ π; 3π 2 Khi đó 5 − 12 √ a2 − 1 a2 = (5 − tan α) cos2 α = 5 cos2 α − 12 sin α cos α = 5 2 (1 + cos 2α) − 6 sin 2α = 5 2 + 13 2 ( 5 13 cos 2α − 12 13 sin 2α) = 5 2 + 13 2 cos(2α + β) với cos β = 5 13 , sin β = 12 13 Vì −1 ≤ cos(2α + β) ≤ 1 nên −4 ≤ 5 − 12 √ a2 − 1 a2 ≤ 9 Vậy max P = 9 ⇔ a = 1 cos β 2 , min y = 2 ⇔ a = 1 cos π−β 2 với cos β = 5 13 , sin β = 12 13 Bài tập 2.15. Cho x ∈ [−1; 1] và n ≥ 2. Tìm giá trị lớn nhất và giá trị nhỏ nhất của y = (1 + x)n + (1 − x)n Gợi ý: Đặt x = cos 2t, 0 ≤ t ≤ π 2 Đáp số: max y = 2n ⇔ x = 1 min y = 2 ⇔ x = 0 43
  • 50. Chương 2. Giá trị lớn nhất, giá trị nhỏ nhất 2.2.4 Phương pháp hình học a. Cơ sở lý thuyết. Khi bài toán tìm giá trị lớn nhất, giá trị nhỏ nhất có tiềm ẩn các yếu tố hình học, ta có thể sử dụng các đánh giá hình học để giải quyết bài toán. Một số đánh giá hình học quen thuộc: 1) Với các vectơ −→u , −→v bất kì ta luôn có • |−→u + −→v | ≤ |−→u | + |−→v | Dấu bằng xảy ra khi −→u , −→v cùng hướng. • |−→u .−→v | ≤ |−→u | |−→v | Dấu bằng xảy ra khi −→u , −→v cùng phương. • −→u 2 ≥ 0 Dấu bằng xảy ra khi −→u = −→ 0 2) Với ba điểm A, B, C bất kì ta luôn có AB + BC ≥ AC Dấu bằng xảy ra khi A, B, C thẳng hàng theo thứ tự trên. 3) Cho điểm M nằm ngoài đường thẳng d cho trước. Khi đó, độ dài đường vuông góc hạ từ M xuống đường thẳng d luôn ngắn hơn mọi đường xiên kẻ từ M xuống d. b.Các ví dụ áp dụng. Ví dụ 2.13. Tìm giá trị nhỏ nhất của hàm số f(x) = x2 − x + 1 + x2 − √ 3x + 1, với x ∈ R Lời giải. Hàm số đã cho xác định với mọi x ∈ R Ta có f(x) = (x − 1 2 )2 + ( √ 3 2 )2 + (x − √ 3 2 )2 + ( 1 2 )2 Xét hệ trục Oxy, với: A(1 2; √ 3 2 ), B( √ 3 2 ; −1 2) và C(x; 0), x ∈ R Khi đó f(x) = CA + CB. 44
  • 51. Chương 2. Giá trị lớn nhất, giá trị nhỏ nhất Lại có CA + CB ≥ AB = ( √ 3 2 − 1 2 )2 + (− 1 2 − √ 3 2 )2 = √ 2 Như vậy f(x) ≥ √ 2, ∀x ∈ R Vậy min f(x) = √ 2 ⇔ x = √ 3 − 1 Nhận xét: Ví dụ trên xuất phát từ một bài toán đại số đã được quy về bài toán hình học: tìm trên trục Ox một điểm C sao cho tổng khoảng cách CA + CB là nhỏ nhất.Từ đây ta suy ra: Nếu A, B nằm về hai phía của trục Ox, thì điểm C là giao điểm của đường thẳng AB với trục Ox. Nếu A, B nằm cùng phía với trục Ox, thì điểm C là giao điểm của đường thẳng AB với trục Ox, trong đó B là điểm đối xứng của B qua Ox. Ví dụ 2.14. Tìm giá trị lớn nhất, giá trị nhỏ nhất của hàm số f(x) = cos2 x − 2 cos x + 3 + cos2 x + 4 cos x + 6 Lời giải. Ta có f(x) = (1 − cos x)2 + 2 + (2 + cos x)2 + 2 45
  • 52. Chương 2. Giá trị lớn nhất, giá trị nhỏ nhất Trong mặt phẳng Oxy xét điểm M( √ 2; 1 − cos x) và N(2 √ 2; 3). Khi đó f(x) = ( √ 2)2 + (1 − cos x)2 + (2 √ 2 − √ 2)2 + [3 − (1 − cos x)]2 = OM + MN Do 0 ≤ 1−cos x ≤ 2 nên M chạy trên đoạn AB với A( √ 2; 0) và B( √ 2; 2). Từ bất đẳng thức OM + MN ≥ ON, ta suy ra f(x) ≥ (2 √ 2)2 + 32 ⇔ f(x) ≥ √ 17 Dấu bằng xảy ra khi và chỉ khi M ≡ C, với C là giao điểm của ON và AB. Khi đó: cos x = − 1 2 ⇔ x = ± 2π 3 + k2π, k ∈ Z Mặt khác: OM + MN ≤ max(OA + AN; OB + BN) = max( √ 2 + √ 11; √ 6 + √ 3) Do đó f(x) ≤ √ 2 + √ 11 Dấu bằng xảy ra khi và chỉ khi M ≡ A ⇔ cos x = 1 Vậy min f(x) = √ 17 ⇔ x = ± 2π 3 + k2π, k ∈ Z max f(x) = √ 2 + √ 11 ⇔ x = 2kπ, k ∈ Z 46
  • 53. Chương 2. Giá trị lớn nhất, giá trị nhỏ nhất Ví dụ 2.15. Cho các số thực dương a, b, c thỏa mãn ab + bc + ca = abc. Tìm giá trị nhỏ nhất của biểu thức P = √ b2 + 2a2 ab + √ c2 + 2b2 bc + √ a2 + 2c2 ca Lời giải. Biến đổi P về dạng P = 1 a2 + 2 b2 + 1 b2 + 2 c2 + 1 c2 + 2 a2 Đặt −→u (1 a; √ 2 b ), −→v (1 b ; √ 2 c ), −→w (1 c ; √ 2 a ). Khi đó −→u + −→v + −→w = ( 1 a + 1 b + 1 c ; √ 2 a + √ 2 b + √ 2 c ) Mặt khác, từ giả thiết ab + bc + ca = abc suy ra 1 a + 1 b + 1 c = 1, nên −→u + −→v + −→w = (1; √ 2) ⇒ |−→u + −→v + −→w | = √ 1 + 2 = √ 3 Nhận thấy P = |−→u | + |−→v | + |−→w | ≥ |−→u + −→v + −→w | Suy ra P ≥ √ 3. Dấu bằng xảy ra khi a = b = c = 3 Vậy min P = √ 3 ⇔ a = b = c = 3. c. Bài tập tham khảo. Bài tập 2.16. (Đề thi học sinh giỏi Toán Toàn Quốc năm 1998) Tìm giá trị giá trị nhỏ nhất của biểu thức F(x; y) = (x + 1)2 + (y − 1)2+ (x − 1)2 + (y + 1)2+ (x + 2)2 + (y + 2)2 Trong đó x, y là các số thực. Gợi ý: Trong hệ trục Oxy lấy các điểm A(0; −1), B(0; 3). Bài toán trở thành: Tìm giá trị nhỏ nhất của tổng khoảng cách từ điểm M(x; y) trên đường thẳng y = 2x − 2 đến hai điểm A và B, nghĩa là tìm giá trị nhỏ nhất của F(x; y) = F(M) = MA + MB 47
  • 54. Chương 2. Giá trị lớn nhất, giá trị nhỏ nhất Giải bài toán này bằng hình học: Lấy điểm B đối xứng với điểm B qua đường thẳng y = 2x − 2, được B (4; 1). Khi đó điểm M phải tìm là giao điểm của hai đường thẳng AB và y = 2x − 2. Đường thẳng AB có phương trình y = 1 2x − 1. Suy ra M(2 3; −2 3). Vậy min F(x; y) = 2 √ 5. Bài tập 2.17. Cho x, y là các số thực thỏa mãn điều kiện    −x + 2y − 8 ≤ 0 x + y + 2 ≥ 0 y − 2x − 4 ≥ 0 (∗) Tìm giá trị lớn nhất và nhỏ nhất của biểu thức P = x2 + y2 . Lời giải. Gọi M(x; y) là điểm trên mặt phẳng có tọa độ thỏa mãn hệ (∗). Ta thấy tập hợp các điểm M là tam giác ABC (kể cả ba cạnh), trong đó A(−4; 2), B(0; 4), C(−2; 0). Ta có x2 + y2 = OM2 . 48
  • 55. Chương 2. Giá trị lớn nhất, giá trị nhỏ nhất Do đó max P = max (x;y)∈ ABC (x2 + y2 ) = max M∈ ABC OM2 = max OA2 ; OB2 ; OC2 = max {20; 16; 4} = 20 = OA2 min P = min M∈ ABC OM2 = OH2 (ở đây ta kẻ OH ⊥ BC) Lại có OH là khoảng cách từ O đến đường thẳng y − 2x − 4 = 0, do đó OH = |−4| √ 1 + 4 = 4 √ 5 Vậy max P = 20 ⇔ x = −4 y = 2 min P = 16 5 ⇔ x = 8 5 x = 4 5 Bài tập 2.18. Cho bốn số thực x, y, z, t thỏa mãn điều kiện x + y = 6; z2 + t2 = 1. Tìm giá trị nhỏ nhất của biểu thức P = x2 + y2 − 2xz − 2yt Gợi ý: Xét hệ trục tọa độ Oxy. Khi đó điểm M(x; y) với x + y = 6 nằm trên đường thẳng x + y = 6, còn điểm N(z; t) với z2 + t2 = 1 nằm 49
  • 56. Chương 2. Giá trị lớn nhất, giá trị nhỏ nhất trên đường tròn đơn vị x2 + y2 = 1 tâm O, bán kính bằng 1. Viết P dưới dạng P = (x − z)2 + (y − t)2 − (z2 + t2 ) = (x − z)2 + (y − t)2 − 1 = MN2 − 1 Đáp số: min P = 18 − 6 √ 2 ⇔ x = y = √ 2 2 z = t = 3 Bài tập 2.19. Cho x, y là các số thực không âm thỏa mãn điều kiện 2x + y ≥ 2 x + 3y ≤ 9 (∗) Tìm giá trị lớn nhất và nhỏ nhất của biểu thức P = x2 + y2 − 4x − 8y. Gợi ý: Xét hệ trục Oxy với điểm M(x; y) có tọa độ thỏa mãn hệ điều kiện của giả thiết. Khi đó, tập hợp điểm M là tứ giác ABCD với A(1; 0), B(0; 2), C(0; 3), D(9; 0). Biểu thức P có dạng P = (x − 2)2 + (y − 4)2 − 20 Gọi I(2; 4), khi đó P = MI2 − 20 Đáp số: max P = 45 ⇔ x = 9; y = 0 min P = 35 2 ⇔ x = 3 2 ; y = 5 2 50
  • 57. Chương 2. Giá trị lớn nhất, giá trị nhỏ nhất Bài tập 2.20. Cho tam giác ABC thay đổi luôn nội tiếp đường tròn bán kính R cho trước. Tìm giá trị lớn nhất của biểu thức M = a2 + b2 + c2 trong đó a, b, c lần lượt là độ dài ba cạnh tam giác đã cho. Lời giải. Ki hiệu O là tâm đường tròn ngoại tiếp tam giác ABC, G là trọng tâm tam giác.Khi đó, từ đẳng thức: 3 −→ OG = −→ OA + −−→ OB + −→ OC ta suy ra (3 −→ OG)2 = ( −→ OA + −−→ OB + −→ OC)2 = −→ OA2 + −−→ OB2 + −→ OC2 + 2( −→ OA −−→ OB + −−→ OB −→ OC + −→ OC −→ OA) = 9R2 − ( −→ OA − −−→ OB)2 + ( −−→ OB − −→ OC)2 + ( −→ OC − −→ OA)2 = 9R2 − ( −→ BA2 + −−→ CB2 + −→ AC2 ) = 9R2 − (a2 + b2 + c2 ) Do đó M = a2 + b2 + c2 ≤ 9R2 Dấu bằng xảy ra khi 9 −→ OG2 = 0 ⇔ O ≡ G Vậy max M = 9R2 2.2.5 Phương pháp sử dụng bất đẳng thức Phương pháp sử dụng bất đẳng thức được áp dụng rộng rãi để tìm giá trị lớn nhất, giá trị nhỏ nhất. Trong phạm vi toán sơ cấp, luận văn chỉ nêu ra một số phương pháp sử dụng các bất đẳng thức quen thuộc như bất đẳng thức Cauchy, bất đẳng thức giữa các giá trị trung bình và một số bất đẳng thức có cách chứng minh đơn giản. a. Cơ sở lý thuyết. Phương pháp này còn có thể gọi là phương pháp sử dụng định nghĩa, vì ta sử dụng trực tiếp định nghĩa giá trị lớn nhất, giá trị nhỏ nhất để giải quyết bài toán. Quy tắc: 51
  • 58. Chương 2. Giá trị lớn nhất, giá trị nhỏ nhất - Sử dụng các bất đẳng thức quen thuộc để đánh giá biểu thức cần tìm giá trị lớn nhất, giá trị nhỏ nhất với các hằng số M và n; - Chỉ ra ít nhất một bộ phận giá trị của các biến để dấu đẳng thức xảy ra; - Kết luận về giá trị lớn nhất, giá trị nhỏ nhất cần tìm. Một số bất đẳng thức quen thuộc: 1. Bất đẳng thức Cauchy: Với hai bộ n số thực (a1, a2, ..., an) và (b1, b2, ..., bn) ta luôn có (a1b1 + a2b2 + ... + anbn)2 ≤ (a2 1 + a2 2 + ... + a2 n)(b2 1 + b2 2 + ... + b2 n) Dấu đẳng thức xảy ra khi a1 b1 = a2 b2 = ... = an bn . Với quy ước: nếu ai = 0 thì bi = 0. 2. Bất đẳng thức giữa trung bình cộng và trung bình nhân: Giả sử x1, x2, ..., xn là các số không âm. Khi đó x1 + x2 + ... + xn n ≥ n √ x1x2...xn Dấu đẳng thức xảy ra khi x1 = x2 = ... = xn 3. Bất đẳng thức Bernoulli: Với mọi số thực α > 1 ta luôn có xα + α − 1 ≥ αx, x ∈ R+ Dấu đẳng thức xảy ra khi x = 1. Chứng minh. Xét hàm số f(x) = xα + α − 1 − αx, x > 0 Ta có f(1) = 0 và f (x) = αxα−1 − α = α(xα−1 − 1). Suy ra f (x) = 0 ⇔ x = 1 và x = 1 là cực tiểu duy nhất của f(x) trên R+ . Vậy f(x) ≥ f(1) = 0. 4. Sử dụng tính chất của giá trị lớn nhất, giá trị nhỏ nhất của hàm số được nêu trong định lý (2.2) và (2.3). Cho hàm số f(x) có đạo hàm cấp một liên tục trên [a; b], có đạo 52
  • 59. Chương 2. Giá trị lớn nhất, giá trị nhỏ nhất hàm cấp hai tại mọi điểm x ∈ (a; b).Khi đó : • Nếu f (x) ≥ 0 với mọi x ∈ (a; b), thì f(x) ≥ f(y) + f (y)(x − y), ∀x, y ∈ [a; b] • Nếu f (x) ≤ 0 với mọi x ∈ (a; b), thì f(x) ≤ f(y) + f (y)(x − y), ∀x, y ∈ [a; b] b. Các ví dụ áp dụng. Ví dụ 2.16. Cho a, b, c là các số thực dương thỏa mãn a + b + c = 1. Tìm giá trị nhỏ nhất của biểu thức P = a2 b + c + b2 c + a + c2 a + b Lời giải. Nhận thấy 1 = (a+b+c)2 = ( a √ b + c √ b + c+ b √ c + a √ c + a+ c √ a + b √ a + b)2 (∗) Áp dụng bất đẳng thức Cauchy cho hai bộ số ( a √ b + c ; b √ c + a ; c √ a + b ) và ( √ b + c; √ c + a; √ a + b), ta có (∗) ≤ ( a2 b + c + b2 c + a + c2 a + b )(b + c + c + a + a + b) ⇔ 1 ≤ 2P ⇔ P ≥ 1 2 Dấu đẳng thức xảy ra khi a = b = c = 1 3 Vậy min P = 1 2 ⇔ a = b = c = 1 3 Ví dụ 2.17. Cho a, b, c là độ dài ba cạnh một tam giác. Tìm giá trị nhỏ nhất của biểu thức P = a b + c + b c + a + c a + b 53
  • 60. Chương 2. Giá trị lớn nhất, giá trị nhỏ nhất Lời giải. Nhận thấy P + 3 = a b + c + 1 + b c + a + 1 + c a + b + 1 = a + b + c b + c + b + c + a c + a + c + a + b a + b = (a + b + c)( 1 b + c + 1 c + a + 1 a + b ) = 1 2 [(a + b) + (b + c) + (c + a)] ( 1 b + c + 1 c + a + 1 a + b ) Áp dụng bất đẳng thức Cauchy cho hai bộ số (a + b), (b + c), (c + a)và 1 b + c , 1 c + a , 1 a + b , ta có [(a + b) + (b + c) + (c + a)] ( 1 b + c + 1 c + a + 1 a + b ) ≥ 9 Dấu đẳng thức xảy ra khi a = b = c Suy ra P + 3 ≥ 9 2 ⇔ P ≥ 3 2 Vậy minP = 3 2 ⇔ a = b = c (hay tam giác đã cho là tam giác đều). Ví dụ 2.18. Cho x, y là các số thực thỏa mãn 2x2 + 3y2 ≤ 5. Tìm giá trị lớn nhất, giá trị nhỏ nhất của biểu thức A = 2x + 3y Lời giải. Nhận thấy A2 = (2x + 3y)2 = √ 2( √ 2x) + √ 3( √ 3y) 2 Áp dụng bất đẳng thức Cauchy ta có √ 2( √ 2x) + √ 3( √ 3y) 2 ≤ (2 + 3)(2x2 + 3y2 ) ⇔ A2 ≤ 25 ⇔ − 5 ≤ A ≤ 5 54
  • 61. Chương 2. Giá trị lớn nhất, giá trị nhỏ nhất Vậy min A = −5 ⇔ x = y = −1 max A = 5 ⇔ x = y = 1 Ví dụ 2.19. (Đề thi học sinh giỏi Toán Quốc gia năm 1994) Xét các biến số thực x, y, u, v thỏa mãn ba điều kiện:    2x2 + 3y2 = 10 3u2 + 8v2 = 6 4xv + 3yu ≥ 2 √ 15 Tìm giá trị lớn nhất và giá trị nhỏ nhất của biểu thức S = x + y + u. Lời giải. Áp dụng bất đẳng thức Cauchy ta có 4xv + 3yu ≤ |4xv + 3yu| ≤ (2x2 + 3y2)(8v2 + 3u2) ≤ 2 √ 15 Dấu đẳng thức xảy ra khi và chỉ khi √ 2x 2 √ 2v = √ 3y √ 3u ⇔ x v = y u Hay xv và yu phải cùng dấu dương. Lại có, theo giả thiết 4xv + 3yu ≥ 2 √ 15 Từ đó suy ra 4xv + 3yu = 2 √ 15 Theo tính chất của tỉ lệ thức và giả thiết ta có 2x2 8v2 = 3y2 3u2 = x2 + 3y2 8v2 + 3u2 = 10 6 Suy ra x = 2 √ 5√ 3 v và y = √ 5√ 3 u Khi đó S = x + y + u = 2 √ 5 √ 3 v + ( √ 5 √ 3 + 1)u Áp dụng bất đẳng thức Cauchy ta được |S| = 2 √ 5 √ 3 v + ( √ 5 √ 3 + 1)u ≤ ( 5 6 + 8 + 2 √ 15 9 )(8v2 + 3u2) = 31 + 4 √ 15 3 55
  • 62. Chương 2. Giá trị lớn nhất, giá trị nhỏ nhất Như vậy − 31 + 4 √ 15 3 ≤ S ≤ 31 + 4 √ 15 3 Ta thấy: S1 = 31+4 √ 15 3 tại u1 = 16+4 √ 15 23+2 √ 15 ; v1 = 45 92+8 √ 15 và x1 = 2 √ 5√ 3 v1; y1 = √ 5√ 3 u1. S2 = − 31+4 √ 15 3 tại u2 = −u1; v2 = −v1 và x2 = 2 √ 5√ 3 v2; y2 = √ 5√ 3 u2. Kết luận max S = 31 + 4 √ 15 3 min S = − 31 + 4 √ 15 3 Ví dụ 2.20. Cho a, b, c là các số thực dương thỏa mãn a + b + c = 2012. Tìm giá trị nhỏ nhất của biểu thức M = a3 (a + b)(b + c) + b3 (b + c)(c + a) + c3 (c + a)(a + b) Lời giải. Áp dụng bất đẳng thức giữa trung bình cộng và trung bình nhân, ta có: 8a3 (a + b)(b + c) + (a + b) + (b + c) ≥ 6a 8b3 (b + c)(c + a) + (b + c) + (c + a) ≥ 6b 8c3 (c + a)(a + b) + (c + a) + (a + b) ≥ 6c Cộng các bất dẳng thức trên vế theo vế ta được 8M ≥ 2(a + b + c) ⇔ M ≥ 1 4 (a + b + c) Vậy min M = 503 ⇔ a = b = c = 2012 3 . 56
  • 63. Chương 2. Giá trị lớn nhất, giá trị nhỏ nhất Ví dụ 2.21. Cho a, b, c > 1. Tìm giá trị nhỏ nhất của biểu thức M = 4a2 a − 1 + 5b2 b − 1 + 3c2 c − 1 Lời giải. Ta có 4a2 a − 1 = 4(a + 1) + 4 a − 1 = 4 a − 1 + 4(a − 1) + 8 Áp dụng bất đẳng thức giữa trung bình cộng và trung bình nhân cho hai số dương 4 a−1 và 4(a − 1) ta được: 4 a − 1 + 4(a − 1) ≥ 2 4 a − 1 .4(a − 1) = 8 Dấu đẳng thức xảy ra khi 4 a−1 = 4(a − 1) ⇔ a = 2 Suy ra 4a2 a − 1 ≥ 16 Chứng minh tương tự: 5b2 b − 1 ≥ 20 3c2 c − 1 ≥ 12 Cộng ba bất đẳng thức trên vế theo vế ta được M ≥ 48 Vậy min M = 48 ⇔ a = b = c = 2 Ví dụ 2.22. Cho a, b, c > 0 thỏa mãn a+b+c = 1. Tìm giá trị lớn nhất của biểu thức M = a 2 3 + b 2 3 + c 2 3 Lời giải. Áp dụng bất đẳng thức Bernoulli cho trường hợp x = (3t) 2 3 , ∀t > 0 và α = 3 2, ta có (3t) 2 3 2 3 + 1 2 ≥ 3 2 (3t) 2 3 ⇔ 3t + 1 2 ≥ 3 2 .3 2 3 .t 2 3 (∗) 57
  • 64. Chương 2. Giá trị lớn nhất, giá trị nhỏ nhất Dấu đẳng thức xảy ra khi t = 1 3 Áp dụng bất đẳng thức (∗) cho các số a, b, c ta có: 3a + 1 2 ≥ 3 2 .3 2 3 .a 2 3 3b + 1 2 ≥ 3 2 .3 2 3 .b 2 3 3c + 1 2 ≥ 3 2 .3 2 3 .c 2 3 Cộng các bất đẳng thức trên vế theo vế: 3 + 3 2 ≥ 3 2 .3 2 3 .M Suy ra M ≤ 3 3 √ 9 Vậy max M = 3 3 √ 9 ⇔ a = b = c = 1 3 Ví dụ 2.23. Cho    x ≥ 4 x + y ≥ 7 x + y + z ≥ 8 (∗). Tìm min x2 + y2 + z2 Lời giải. Xét hàm số f(t) = t2 , t ∈ R có f (t) = 2 > 0, ∀t ∈ R Theo định lý (2.2) ta có f(t) ≥ f(t0) + f (t0)(t − t0); ∀t, t0 ∈ R Áp dụng cho trường hợp t = x, y, z và t0 = 4, 3, 1, ta đươc: x2 ≥ 16 + 8(x − 4) y2 ≥ 9 + 6(y − 3) z2 ≥ 1 + 2(z − 1) Cộng các bất đẳng thức trên vế theo vế x2 + y2 + z2 ≥ 26 + 2(x + y + z − 8) + 4(x + y − 7) + 2(x − 4) 58
  • 65. Chương 2. Giá trị lớn nhất, giá trị nhỏ nhất Từ hệ điều kiện (∗) ta suy ra x2 + y2 + z2 ≥ 26. Dấu đẳng thức xảy ra khi x = 4, y = 3, z = 1. Vậy min x2 + y2 + z2 = 26 ⇔ x = 4, y = 3, z = 1 Ví dụ 2.24. Cho x, y, z thỏa mãn 0 ≤ x, y, z ≤ 2; x + +z = 3. Tìm giá trị lớn nhất của biểu thức M = (1 + x2 )x (1 + y2 )y (1 + z2 )z Lời giải. Không mất tính tổng quát, giả sử x ≥ y ≥ z. Từ giả thiết ta có    x ≤ 2 x + y ≤ 3 = 2 + 1 x + y + z = 3 = 2 + 1 + 0 Xét hàm số f(x) = x ln(1 + x2 ) với x ≥ 0. Ta thấy f (x) = ln(1 + x2 ) + 2x2 1+x2 ; f (x) = 2x 1+x2 + 4x (1+x2)2 > 0, ∀x ≥ 0 Áp dụng định lý (2.2) ta suy ra f(2) ≥ f(x) + f (x)(2 − x) Do x ≤ 2 nên f (x)(2 − x) ≥ 0, ∀0 ≤ x ≤ 2 Suy ra f(x) ≤ f(2) ⇔ x ln(1 + x2 ) ≤ 2 ln 5 Dấu đẳng thức xảy ra khi x = 2 Tương tự ta có y ln(1 + y2 ) ≤ 1 ln 2 z ln(1 + z2 ) ≤ 0 ln 1 Cộng các bất đẳng thức trên vế theo vế ta được x ln(1 + x2 ) + y ln(1 + y2 ) + z ln(1 + z2 ) ≤ 2 ln 5 + 1 ln 2 + 0 ln 1 ⇔ ln (1 + x2 ) + (1 + y2 ) + (1 + z2 ) ≤ ln 50 ⇔(1 + x2 ) + (1 + y2 ) + (1 + z2 ) ≤ 50 Vậy max M = 50 khi x = 2, y = 1, z = 0. Ví dụ 2.25. Cho tam giác ABC không nhọn. Tìm giá trị lớn nhất của biểu thức M = sin A + sin B + sin C 59
  • 66. Chương 2. Giá trị lớn nhất, giá trị nhỏ nhất Lời giải. Không mất tính tổng quát ta giả sử A ≥ B ≥ C Từ đó suy ra A ≥ π 2 B + C ≤ π 2 Xét hàm số f(x) = sin x, x ∈ (0; π) có f (x) = − sin x < 0, ∀x ∈ (0; π). Áp dụng định lý (2.3) với x = A, B, C và x0 = π 2 , π 4 , π 4 ta được sin A ≤ sin π 2 + cos π 2 (A − π 2 ) sin B ≤ sin π 4 + cos π 4 (B − π 4 ) sin C ≤ sin π 4 + cos π 4 (C − π 4 ) Cộng các bất đẳng thức trên vế theo vế sin A + sin B + sin C ≤ sin π 2 + sin π 4 + sin π 4 + cos π 4 (B + C) − π 2 ⇔ sin A + sin B + sin C ≤ sin π 2 + sin π 4 + sin π 4 ⇔M ≤ 1 + √ 2 Dấu đẳng thức xảy ra khi A = π 2 , B = π 4 , C = π 4 Vậy max M = 1 + √ 2 khi ABC là tam giác vuông cân. Nhận xét: Các ví dụ (2.23), (2.24), (2.25) đã vận dụng bất đẳng thức Karamata. Tuy nhiên bất đẳng thức này không được giới thiệu trong chương trình phổ thông vì vậy ta đã chứng minh những bài toán này hoàn toàn bằng phương pháp sơ cấp. c. Bài tập tham khảo. Bài tập 2.21. Cho x, y, z, t là các số thực thỏa mãn x + y + z + t = 0 x2 + y2 + z2 + t2 = 1 Tìm giá trị lớn nhất, giá trị nhỏ nhất của biểu thức P = xy+yz+zt+tx. Lời giải. Áp dụng bất đẳng thức Cauchy ta có (xy + yz + zt + tx)2 ≤ (x2 + y2 + z2 + t2 )2 ⇔ P2 ≤ 1 ⇔ − 1 ≤ P ≤ 1 (1) 60
  • 67. Chương 2. Giá trị lớn nhất, giá trị nhỏ nhất Mặt khác P = (x + z)(y + t) = −(x + z)2 ⇒ P ≤ 0 (2) Từ (1) và (2) suy ra: −1 ≤ P ≤ 0 Vậy min P = −1 khi x = z = −1 2 y = t = 1 2 max P = 0 khi x = y = −1 2 z = t = 1 2 Bài tập 2.22. Cho a, b, c là các số thực dương thỏa mãn a+b+c = 2012. Tìm giá trị nhỏ nhất của biểu thức M = a2 a + b + b2 b + c + c2 c + a Gợi ý:Áp dụng bất đẳng thức Cauchy ta suy ra M [(a + b) + (b + c) + (c + a)] ≥ (a + b + c)2 Đáp số. min M = a + b + c 2 = 1006 ⇔ a = b = c = 2012 3 Bài tập 2.23. Cho −1 ≤ x ≤ 1. Tìm giá trị nhỏ nhất của hàm số f(x) = 4 1 − x2 + 4 √ 1 + x + 4 √ 1 − x Lời giải. Áp dụng bất đẳng thức giữa trung bình cộng và trung bình nhân, ta có: 4 1 − x2 = 4 √ 1 − x 4 √ 1 + x ≤ √ 1 − x + √ 1 + x 2 4 √ 1 + x = 4 √ 1 + x 4 √ 1 ≤ √ 1 + x + 1 2 4 √ 1 − x = 4 √ 1 − x 4 √ 1 ≤ √ 1 − x + 1 2 Suy ra f(x) ≤ 1 + √ 1 + x + √ 1 − x Lại có: √ 1 + x + √ 1 − x ≤ 1 + x + 1 2 + 1 + x + 1 2 = 2 Vậy max −1≤x≤1 f(x) = 3 ⇔ x = 0 61
  • 68. Chương 2. Giá trị lớn nhất, giá trị nhỏ nhất Bài tập 2.24. Cho x, y ≥ 0 và x + y = 1. Tìm min x √ 2 + y √ 2 Gợi ý:Áp dụng bất đẳng thức Bernoulli ta suy ra (2t) √ 2 ≥ √ 2(2t) − √ 2 + 1 Đáp số. min x √ 2 + y √ 2 = 21− √ 2 ⇔ x = y = 1 2 Bài tập 2.25. Cho a ≥ b ≥ c và    x ≥ a x + y ≥ b x + y + z ≥ c Tìm giá trị lớn nhất của M = x2 + y2 + z2 Gợi ý: Áp dụng định lý (2.2) cho hàm f(t) = t2 Đáp số. max M = a2 + b2 + c2 Bài tập 2.26. Cho x, y, y, z, t là các số thực dương thỏa mãn:    x ≤ 1 x + y ≤ 3 x + y + z ≤ 6 x + y + z + t ≤ 10 Tìm giá trị lớn nhất của M = 1 x + 1 y + 1 z + 1 t . Gợi ý: Áp dụng định lý (2.2) cho hàm f(t) = 1 t Đáp số. max M = 25 12 ⇔ x = 1, y = 2, z = 3, t = 4 Bài tập 2.27. Cho tam giác ABC bất kỳ.Tìm giá trị lớn nhất của các biểu thức P = sin A 2 2 √ 2 + sin B 2 2 √ 3 + sin C 2 √ 6 + √ 2 62
  • 69. Chương 2. Giá trị lớn nhất, giá trị nhỏ nhất Gợi ý: Nhận thấy 4P = sin A 2√ 2 2 + sin B 2√ 3 2 + sin C 2√ 6+ √ 2 2 = sin A 2 cos π 4 + sin B 2 cos π 6 + sin C 2 cos π 12 (1) Áp dụng định lý (2.3) cho hàm y = sin x ta có sin x ≤ sin y + cos y(x − y), ∀x, y ∈ (0; π 2 ) ⇔ sin x cos y ≤ sin y cos y + (x − y), ∀x, y ∈ (0; π 2 ) Vận dụng vào bài toán, ta được: sin A 2 cos π 4 ≤ sin π 4 cos π 4 + ( A 2 − π 4 ) sin B 2 cos π 6 ≤ sin π 6 cos π 6 + ( B 2 − π 6 ) sin C 2 cos π 12 ≤ sin π 12 cos π 12 + ( C 2 − π 12 ) (2) Từ (1) và (2) suy ra 4P ≤ 1 + √ 3 3 + (2 − √ 3) Dấu đẳng thức xảy xảy ra khi A = π 2 , B = π 3 , C = π 6 Đáp số. max P = 9−2 √ 3 12 ⇔ A = π 2 , B = π 3 , C = π 6 2.2.6 Một số bài tập vận dụng Trên đây luận văn đã trình bày một số phương pháp cơ bản để tìm giá trị lớn nhất, giá trị nhỏ nhất; các ví dụ và bài tập ứng dụng của từng phương pháp. Tuy nhiên trong thực tế không phải lúc nào một bài toán cũng được giải quyết bằng một phương pháp riêng lẻ, có những bài toán 63
  • 70. Chương 2. Giá trị lớn nhất, giá trị nhỏ nhất cần sự phối hợp nhiều phương pháp. Vì vậy, phần tiếp theo của luận văn sẽ trình bày một số bài tập được giải bằng cách phối hợp nhiều phương pháp. Bài tập 2.28. (Đề thi học sinh giỏi Toán Quốc gia năm 2004) Xét các số thực dương x, y, z thỏa mãn điều kiện (x + y + z)3 = 32xyz. Hãy tìm giá trị lớn nhất, giá trị nhỏ nhất của biểu thức P = x4 + y4 + z4 (x + y + z)4 Lời giải. Nhận thấy với n là số thực dương tùy ý ta luôn có P(x, y, z) = P(nx + ny + nz) Nếu x, y, z thỏa mãn giả thiết thì nx, ny, nz cũng thỏa mãn các điều kiện đó. Vì vậy không mất tính tổng quát, giả sử x + y + z = 4. Khi đó, kết hợp với điều kiện đề bài, ta được xyz = 2. Bài toán quy về việc tìm giá trị lớn nhất, giá trị nhỏ nhất của biểu thức P = 1 256 (x4 + y4 + z4 ) trong đó x, y, z là các số thực dương thỏa mãn x + y + z = 4 và xyz = 2. Đặt Q = x4 + y4 + z4 và t = xy + yz + zx, ta có Q = (x2 + y2 + z2 )2 − 2(x2 y2 + y2 z2 + z2 x2 ) = (42 − 2t)2 − 2(t2 − 2xyz(x + y + z)) = 2t2 − 64t + 44 + 32 Q = 2(t2 − 32t + 144) (1) Từ các điều kiện đối với x, y, z ta được y + z = 4 − x và yz = 2 x (2) Do đó t = x(4 − x) + 2 x (3) Áp dụng bất đẳng thức giữa trung bình cộng và trung bình nhân cho 64
  • 71. Chương 2. Giá trị lớn nhất, giá trị nhỏ nhất hai số dương y, z, từ (2) ta được (4 − x)2 ≥ 8 x ⇔ x3 − 8x2 + 16x − 8 ≥ 0 ⇔ (x − 2)(x2 − 6x + 4) ≥ 0 ⇔ 3 − √ 5 ≤ x ≤ 2 (vì x ∈ (0; 4)) Xét hàm số t(x) = x(4 − x) + 2 x trên đoạn 3 − √ 5; 2 ta có t (x) = −2(x − 1)(x2 − x − 1) x2 Từ việc xét dấu của t (x) trên 3 − √ 5; 2 ta được 5 ≤ t ≤ 5 √ 5 − 1 2 Vì hàm f(t) = t2 − 32t + 144 nghịch biến trong khoảng (0; 16) và 3 − √ 5; 2 ⊂ (0; 16) Nên ta có min f(t) = f( 5 √ 5 − 1 2 ) = 383 − 165 √ 5 2 max f(t) = f(5) = 9 Kết hợp với (1) ta được min Q = 183 − 165 √ 5; max Q = 18 Kết luận min P = 383 − 165 √ 5 256 ⇔ x = 3 − √ 5, y = z = 1 + √ 5 2 max P = 9 128 ⇔ x = 2, y = z = 1 Bài tập 2.29. (Đề thi tuyển sinh Đại học, Cao đẳng khối A năm 2003) Cho x, y, z là các số thực dương và thỏa mãn điều kiện x + y + z ≤ 1. Tìm giá trị nhỏ nhất của biểu thức P = x2 + 1 x2 + y2 + 1 y2 + z2 + 1 z2 Lời giải. 65
  • 72. Chương 2. Giá trị lớn nhất, giá trị nhỏ nhất Xét các vectơ −→u = (x; 1 x), −→v = (y; 1 y ), −→w = (z; 1 z ) Ta có −→u + −→v + −→w = (x + y + z; 1 x + 1 y + 1 z ) Theo tính chất độ dài vectơ ta có |−→u | + |−→v | + |−→w | ≥ |−→u + −→v + −→w | Do đó P = x2 + 1 x2 + y2 + 1 y2 + z2 + 1 z2 ≥ (x + y + z)2 + ( 1 x + 1 y + 1 z )2 Dấu đẳng thức xảy ra khi −→u , −→v , −→w là các vectơ cùng chiều. ⇔ −→u = k1 −→v , k1 > 0 −→v = k2 −→w , k2 > 0 Ta thấy (x+y+z)2 +( 1 x + 1 y + 1 z )2 = 81(x+y+z)2 +( 1 x + 1 y + 1 z )2 −80(x+y+z)2 Áp dụng bất đẳng thức giữa trung bình cộng và trung bình nhân ta có 81(x + y + z)2 + ( 1 x + 1 y + 1 z )2 ≥ 18(x + y + z)( 1 x + 1 y + 1 z ) Mặt khác (x + y + z)( 1 x + 1 y + 1 z ) ≥ 9 Suy ra 81(x + y + z)2 + (1 x + 1 y + 1 z )2 ≥ 162 Theo giả thiết 0 < x + y + z ≤ 1 ⇒ 80(x + y + z)2 ≤ 80 Từ đó suy ra P ≥ √ 82 Dấu đẳng thức xảy ra khi x = y = z = 1 3 Vậy min P = √ 82 ⇔ x = y = z = 1 3. Bài tập 2.30. (Đề thi tuyển sinh Đại học, Cao đẳng khối A năm 2011) Cho x ≥ y, x ≥ z và x, y, z ∈ [1; 4]. Tìm giá trị nhỏ nhất của biểu thức P = x 2x + 3y + y y + z + z z + x Lời giải. 66
  • 73. Chương 2. Giá trị lớn nhất, giá trị nhỏ nhất Viết biểu thức P dưới dạng P = 1 2 + 3y x + 1 1 + z y + 1 1 + x z Trước hết ta chứng minh bất đẳng thức: Nếu a, b > 0 và ab ≥ 1, thì ta có 1 1 + a + 1 1 + b ≥ 2 1 + √ ab (∗) Dấu đẳng thức xảy ra khi và chỉ khi a = b hoặc ab = 1 Thật vậy (∗) ⇔ ( 1 1 + a − 1 1 + √ ab ) + ( 1 1 + b − 1 1 + √ ab ) ≥ 0 ⇔ √ ab − a (1 + a)(1 + √ ab) + √ ab − b (1 + b)(1 + √ ab) ≥ 0 ⇔ ( √ a − √ b)2 ( √ ab − 1) (1 + a)(1 + b)(1 + √ ab) ≥ 0 (∗∗) Do a, b > 0; ab ≥ 1 nên (∗∗) đúng, suy ra (∗) đúng. Áp dụng (∗) với a = z y , b = x z . Khi đó a, b > 0 và ab = x y ≥ 1 (do x ≥ y), nên ta có 1 1 + z y + 1 1 + x z ≥ 2 1 + x y Dấu đẳng thức xảy ra khi và chỉ khi z y = x z z y .x y = 1 ⇔ √ yz = z x = y Từ đó suy ra P ≥ 1 2 + 3y x + 2 1 + x y Dấu đẳng thức xảy ra ⇔ √ yz = z x = y Đặt t = x y . Do x ≥ y và x, y ∈ [1; 4], nên suy ra 1 ≤ x y ≤ 4 ⇒ 1 ≤ t ≤ 2 Khi đó P ≥ 1 2 + 3 t2 + 2 1 + t hay P ≥ t2 2t2 + 3 + 2 1 + t 67
  • 74. Chương 2. Giá trị lớn nhất, giá trị nhỏ nhất Xét hàm số f(t) = t2 2t2 + 3 + 2 1 + t , 1 ≤ t ≤ 2 f (t) = (3t − 6t2 ) + (3t3 − 4t4 ) − 9 (2t2 + 3)2(1 + t)2 Vì t ≥ 1 nên f (t) < 0, ∀t ∈ [1; 2] Suy ra f(t) nghịch biến trong [1; 2]. Vậy min 1≤t≤2 f(t) = f(2) = 34 33 Từ đó suy ra P ≥ 34 33 Dấu đẳng thức xảy ra khi và chỉ khi x = 4y, z = 2y Lại có x, y, z ∈ [1; 4] nên suy ra x = 4, y = 1, z = 2 Kết luận min P = 34 33 ⇔ x = 4, y = 1, z = 2. 68
  • 75. Kết luận Sau thời gian học tập tại Khoa Toán - Cơ - Tin học, Trường Đại học Khoa học Tự nhiên, Đại học Quốc gia Hà Nội. Được các thầy cô trực tiếp giảng dạy và hướng dẫn đặc biệt là PGS. TS Nguyễn Đình Sang, tôi đã hoàn thành luận văn với đề tài ” Phương pháp giải bài toán cực trị và ứng dụng”. Luận văn đã đạt được một số kết quả sau: 1. Luận văn đã trình bày một cách hệ thống các phương pháp giải bài toán cực trị địa phương và bài toán cực trị tuyệt đối, đưa ra một số bài toán tổng quát với những cách giải hiệu quả, giúp kích thích tư duy tìm tòi, sáng tạo của học sinh trong việc học tập bộ môn Toán. 2. Luận văn khai thác được một số ứng dụng cơ bản của bài toán cực trị với nhiều ví dụ minh họa áp dụng các phương pháp đa dạng kèm theo các bài tập tham khảo được trích từ các kì thi học sinh giỏi Toán Quốc gia, kì thi tuyển sinh và Đại học và Cao đẳng các năm. Vì vậy bản luận văn có thể là tài liệu tham khảo nhằm nâng cao và mở rộng kiến thức cho học sinh bậc trung học phổ thông. 3. Một số hướng phát triển của đề tài: • Phương pháp tìm giá trị lớn nhất, giá trị nhỏ nhất sử dụng bất đẳng thức có thể mở rộng với nhiều bất đẳng thức khác như: bất đẳng thức giữa trung bình cộng với trung bình nhân suy rộng, bất đẳng thức hàm, bất đẳng thức trong dãy số, bất đẳng thức tích phân, ... • Ứng dụng bài toán cực trị tuyệt đối vào việc giải các phương trình và bất phương trình.
  • 76. Tài liệu tham khảo Tiếng Việt 1. Phan Huy Khải (2011),Các phương pháp giải toán Giá trị lớn nhất, giá trị nhỏ nhất, NXB Đại học Sư Phạm. 2. Trần Đức Long, Nguyễn Đình Sang, Hoàng Quốc Toàn (2001),Giáo trình Giải tích tập I - Phép tính vi phân của hàm một biến, NXB Đại học Quốc Gia Hà Nội. 3. Trần Đức Long, Nguyễn Đình Sang, Nguyễn Viết Trần Tiến, Hoàng Quốc Toàn (2001),Phép tính vi phân, NXB Đại học Quốc gia Hà Nội. 4. Nguyễn Vũ Lương, Phạm Văn Hùng, Nguyễn Ngọc Thắng (2006),Các bài giảng về bất đẳng thức Côsi, NXB Đại học Quốc Gia Hà Nội. 5. Nguyễn Văn Mậu (2006), Bất đẳng thức định lý và áp dụng, NXB Giáo Dục. 6. Nguyễn Văn Mậu, Nguyễn Văn Tiến (2010),Một số chuyên đề Đại số bồi dưỡng học sinh giỏi Trung học phổ thông, NXB Giáo Dục . 7. Trần Phương (2010), Tuyển tập các chuyên đề hàm số, NXB Hà Nội. 8. Đoàn Quỳnh, Trần Nam Dũng, Nguyến Vũ Lương, Đặng Hùng Thắng (2010),Tài liệu chuyên toán và giải tích 12, NXB Giáo Dục. 9. Tủ sách Toán học và Tuổi trẻ (2007),Các bài thi Olympic Toán Trung học phổ thông Việt Nam (1990 - 2006), NXB Giáo Dục. Tiếng Nga 10. I. I. Liasko, A. K. Bolartruk, G.P. Golobax (1977),Matematitreski, Vưsa-skola, Kieb.